SlideShare ist ein Scribd-Unternehmen logo
1 von 283
Downloaden Sie, um offline zu lesen
TRẦN ANH TUẤN
TRƯỜNG ĐẠI HỌC THƯƠNG MẠI
Các chuyên đề
LUYỆN THI ĐẠI HỌC
HÀ NỘI - 2011
WWW.VNMATH.COM
Mục lục
I Đại số - Lượng giác - Giải tích 9
Chương 1 Phương trình, bất phương trình, hệ đại số 11
1.1. Phương trình, bất phương trình đa thức . . . . . . . . . . . . . . . . . . . . . . . . . . . . . . . . . . . . . 11
1.1.1. Phương trình, bất phương trình bậc hai . . . . . . . . . . . . . . . . . . . . . . . . . . . . . . . . . 11
1.1.2. Phương trình trình bậc ba . . . . . . . . . . . . . . . . . . . . . . . . . . . . . . . . . . . . . . . . 13
1.1.3. Phương trình, bất phương trình bậc bốn . . . . . . . . . . . . . . . . . . . . . . . . . . . . . . . . 13
1.2. Phương trình, bất phương trình chứa giá trị tuyệt đối . . . . . . . . . . . . . . . . . . . . . . . . . . . . . . 14
1.3. Phương trình, bất phương trình chứa căn . . . . . . . . . . . . . . . . . . . . . . . . . . . . . . . . . . . . 16
Vấn đề 1 : Phương trình, bất phương trình cơ bản . . . . . . . . . . . . . . . . . . . . . . . . . . . . . . . 16
Vấn đề 2 : Phương pháp đặt ẩn phụ . . . . . . . . . . . . . . . . . . . . . . . . . . . . . . . . . . . . . . 17
Vấn đề 3 : Phương pháp nhân liên hợp . . . . . . . . . . . . . . . . . . . . . . . . . . . . . . . . . . . . . 19
Vấn đề 4 : Phương pháp đánh giá . . . . . . . . . . . . . . . . . . . . . . . . . . . . . . . . . . . . . . . 19
Vấn đề 5 : Phương trình, bất phương trình có tham số . . . . . . . . . . . . . . . . . . . . . . . . . . . . . 20
1.4. Hệ phương trình . . . . . . . . . . . . . . . . . . . . . . . . . . . . . . . . . . . . . . . . . . . . . . . . . 23
1.4.1. Phương pháp thế . . . . . . . . . . . . . . . . . . . . . . . . . . . . . . . . . . . . . . . . . . . . 23
1.4.2. Phương pháp phân tích thành nhân tử hoặc coi một phương trình là phương trình bậc hai (ba) theo
một ẩn . . . . . . . . . . . . . . . . . . . . . . . . . . . . . . . . . . . . . . . . . . . . . . . . . 24
1.4.3. Phương pháp đặt ẩn phụ . . . . . . . . . . . . . . . . . . . . . . . . . . . . . . . . . . . . . . . . 24
1.4.4. Phương pháp hàm số . . . . . . . . . . . . . . . . . . . . . . . . . . . . . . . . . . . . . . . . . . 27
1.4.5. Phương pháp đánh giá . . . . . . . . . . . . . . . . . . . . . . . . . . . . . . . . . . . . . . . . . 27
1.5. Số nghiệm của phương trình, hệ phương trình . . . . . . . . . . . . . . . . . . . . . . . . . . . . . . . . . 28
Vấn đề 1 : Chứng minh phương trình có nghiệm duy nhất . . . . . . . . . . . . . . . . . . . . . . . . . . . 28
Vấn đề 2 : Chứng minh phương trình có đúng hai nghiệm phân biệt . . . . . . . . . . . . . . . . . . . . . . 28
Vấn đề 3 : Chứng minh phương trình có đúng ba nghiệm phân biệt . . . . . . . . . . . . . . . . . . . . . . 29
1.6. Phương trình, bất phương trình, hệ đại số trong các kì thi tuyển sinh ĐH . . . . . . . . . . . . . . . . . . . 29
1.7. Bài tập tổng hợp . . . . . . . . . . . . . . . . . . . . . . . . . . . . . . . . . . . . . . . . . . . . . . . . . 31
Chương 2 Bất đẳng thức 37
2.1. Phương pháp sử dụng bất đẳng thức Cauchy . . . . . . . . . . . . . . . . . . . . . . . . . . . . . . . . . . 37
2.1.1. Bất đẳng thức Cauchy - So sánh giữa tổng và tích . . . . . . . . . . . . . . . . . . . . . . . . . . . 37
2.1.2. Một số hệ quả trực tiếp . . . . . . . . . . . . . . . . . . . . . . . . . . . . . . . . . . . . . . . . . 37
2.1.3. Bài tập đề nghị . . . . . . . . . . . . . . . . . . . . . . . . . . . . . . . . . . . . . . . . . . . . . 37
2.2. Bất đẳng thức hình học . . . . . . . . . . . . . . . . . . . . . . . . . . . . . . . . . . . . . . . . . . . . . 42
2.3. Phương pháp sử dụng điều kiện có nghiệm của phương trình hoặc hệ phương trình . . . . . . . . . . . . . . 44
3
WWW.VNMATH.COM
2.4. Bất đẳng thức trong các kì thi tuyển sinh ĐH . . . . . . . . . . . . . . . . . . . . . . . . . . . . . . . . . . 44
2.5. Bài tập tổng hợp . . . . . . . . . . . . . . . . . . . . . . . . . . . . . . . . . . . . . . . . . . . . . . . . . 46
Chương 3 Lượng giác 51
3.1. Phương trình cơ bản . . . . . . . . . . . . . . . . . . . . . . . . . . . . . . . . . . . . . . . . . . . . . . . 51
3.2. Phương trình dạng a sin x + b cos x = c . . . . . . . . . . . . . . . . . . . . . . . . . . . . . . . . . . . . . 52
3.3. Phương pháp đặt ẩn phụ . . . . . . . . . . . . . . . . . . . . . . . . . . . . . . . . . . . . . . . . . . . . . 53
3.4. Đưa phương trình về dạng tích . . . . . . . . . . . . . . . . . . . . . . . . . . . . . . . . . . . . . . . . . 60
3.5. Phương pháp đánh giá và phương pháp hàm số . . . . . . . . . . . . . . . . . . . . . . . . . . . . . . . . . 62
3.6. Giá trị lớn nhất và nhỏ nhất của biểu thức lượng giác . . . . . . . . . . . . . . . . . . . . . . . . . . . . . 63
3.7. Lượng giác trong các kì thi tuyển sinh ĐH . . . . . . . . . . . . . . . . . . . . . . . . . . . . . . . . . . . 63
3.8. Bài tập tổng hợp . . . . . . . . . . . . . . . . . . . . . . . . . . . . . . . . . . . . . . . . . . . . . . . . . 64
Chương 4 Tổ hợp 69
4.1. Các quy tắc đếm. Tổ hợp, chỉnh hợp, hoán vị . . . . . . . . . . . . . . . . . . . . . . . . . . . . . . . . . . 69
4.2. Giải phương trình, bất phương trình, hệ . . . . . . . . . . . . . . . . . . . . . . . . . . . . . . . . . . . . 74
4.3. Hệ số của xk trong khai triển . . . . . . . . . . . . . . . . . . . . . . . . . . . . . . . . . . . . . . . . . . 76
4.4. Hệ số của xk trong khai triển nhị thức (a + b)n . . . . . . . . . . . . . . . . . . . . . . . . . . . . . . . . . 76
4.5. Hệ số của xk trong khai triển (a + b)n(c + d)m . . . . . . . . . . . . . . . . . . . . . . . . . . . . . . . . . 77
4.6. Hệ số của xk trong khai triển (a + b + c)n . . . . . . . . . . . . . . . . . . . . . . . . . . . . . . . . . . . . 77
4.7. Tính tổng các hệ số tổ hợp :
nÈ
k=0
akCk
n . . . . . . . . . . . . . . . . . . . . . . . . . . . . . . . . . . . . . . 77
4.8. Phương pháp cơ bản với ak chỉ là hàm số mũ theo biến k . . . . . . . . . . . . . . . . . . . . . . . . . . . 77
4.9. Phương pháp đạo hàm với ak là tích hàm số mũ và đa thức theo k . . . . . . . . . . . . . . . . . . . . . . . 78
4.10. Phương pháp tích phân với ak là tích hàm số mũ và phân thức theo k . . . . . . . . . . . . . . . . . . . . . 79
4.11. Bài tập tổng hợp . . . . . . . . . . . . . . . . . . . . . . . . . . . . . . . . . . . . . . . . . . . . . . . . . 80
Chương 5 Hàm số 83
5.1. Tính đơn điệu . . . . . . . . . . . . . . . . . . . . . . . . . . . . . . . . . . . . . . . . . . . . . . . . . . 83
Vấn đề 1 : Xét chiều biến thiên của hàm số . . . . . . . . . . . . . . . . . . . . . . . . . . . . . . . . . . . 83
Vấn đề 2 : Tìm điều kiện tham số để hàm số đơn điệu trên một miền . . . . . . . . . . . . . . . . . . . . . . 84
Vấn đề 3 : Giá trị lớn nhất, giá trị nhỏ nhất của hàm một biến số . . . . . . . . . . . . . . . . . . . . . . . 87
Vấn đề 4 : Sử dụng tính đơn điệu chứng minh bất đẳng thức . . . . . . . . . . . . . . . . . . . . . . . . . . 89
Vấn đề 5 : Ứng dụng sự biến thiên vào việc giải phương trình, bất phương trình, hệ . . . . . . . . . . . . . 91
Vấn đề 6 : Ứng dụng sự biến thiên vào bài toán số nghiệm phương trình có tham số . . . . . . . . . . . . . 92
5.2. Cực trị của hàm số . . . . . . . . . . . . . . . . . . . . . . . . . . . . . . . . . . . . . . . . . . . . . . . 93
Vấn đề 1 : Sử dụng dấu hiệu 1 và dấu hiệu 2 để xác định các điểm cực trị của hàm số . . . . . . . . . . . . 94
Vấn đề 2 : Điều kiện của tham số để hàm số đạt cực trị (cực đại hoặc cực tiểu) tại x = x0 hoặc đồ thị hàm
số đạt cực trị tại điểm (x0; y0) . . . . . . . . . . . . . . . . . . . . . . . . . . . . . . . . . . . . . 94
Vấn đề 3 : Tìm điều kiện để hàm số có cực trị và thỏa mãn một vài điều kiện . . . . . . . . . . . . . . . . . 95
5.3. Tiệm cận . . . . . . . . . . . . . . . . . . . . . . . . . . . . . . . . . . . . . . . . . . . . . . . . . . . . 100
Vấn đề 1 : Tìm tiệm cận của đồ thị hàm số . . . . . . . . . . . . . . . . . . . . . . . . . . . . . . . . . . . 100
Vấn đề 2 : Các bài toán về tiệm cận có tham số . . . . . . . . . . . . . . . . . . . . . . . . . . . . . . . . 101
5.4. Tâm đối xứng và trục đối xứng. Điểm thuộc đồ thị . . . . . . . . . . . . . . . . . . . . . . . . . . . . . . . 102
Vấn đề 1 : Tâm đối xứng, trục đối xứng . . . . . . . . . . . . . . . . . . . . . . . . . . . . . . . . . . . . . 102
Vấn đề 2 : Khoảng cách . . . . . . . . . . . . . . . . . . . . . . . . . . . . . . . . . . . . . . . . . . . . . 102
5.5. Biện luận số nghiệm của phương trình, bất phương trình bằng phương pháp đồ thị . . . . . . . . . . . . . . 103
5.6. Bài toán về sự tương giao . . . . . . . . . . . . . . . . . . . . . . . . . . . . . . . . . . . . . . . . . . . . 108
5.7. Sự tiếp xúc của hai đường cong và tiếp tuyến . . . . . . . . . . . . . . . . . . . . . . . . . . . . . . . . . . 109
Vấn đề 1 : Viết phương trình tiếp tuyến biết tiếp điểm . . . . . . . . . . . . . . . . . . . . . . . . . . . . . 109
Vấn đề 2 : Hai đường cong tiếp xúc . . . . . . . . . . . . . . . . . . . . . . . . . . . . . . . . . . . . . . . 111
Vấn đề 3 : Tiếp tuyến đi qua một điểm . . . . . . . . . . . . . . . . . . . . . . . . . . . . . . . . . . . . . 112
Vấn đề 4 : Tiếp tuyến có hệ số góc cho trước . . . . . . . . . . . . . . . . . . . . . . . . . . . . . . . . . . 113
5.8. Hàm số trong các kì thi tuyển sinh ĐH . . . . . . . . . . . . . . . . . . . . . . . . . . . . . . . . . . . . . 114
5.9. Bài tập tổng hợp . . . . . . . . . . . . . . . . . . . . . . . . . . . . . . . . . . . . . . . . . . . . . . . . . 121
Chương 6 Mũ và lôgarít 127
6.1. Hàm số mũ, hàm số lũy thừa . . . . . . . . . . . . . . . . . . . . . . . . . . . . . . . . . . . . . . . . . . 127
6.2. Hàm số logarit . . . . . . . . . . . . . . . . . . . . . . . . . . . . . . . . . . . . . . . . . . . . . . . . . . 127
6.3. Phương trình mũ và logarit . . . . . . . . . . . . . . . . . . . . . . . . . . . . . . . . . . . . . . . . . . . 129
Vấn đề 1 : Phương trình cơ bản . . . . . . . . . . . . . . . . . . . . . . . . . . . . . . . . . . . . . . . . . 129
Vấn đề 2 : Phương pháp logarit hai vế . . . . . . . . . . . . . . . . . . . . . . . . . . . . . . . . . . . . . 130
Vấn đề 3 : Phương pháp đặt ẩn phụ . . . . . . . . . . . . . . . . . . . . . . . . . . . . . . . . . . . . . . 130
Vấn đề 4 : Phương pháp phân tích thành nhân tử . . . . . . . . . . . . . . . . . . . . . . . . . . . . . . . 131
Vấn đề 5 : Phương pháp đánh giá . . . . . . . . . . . . . . . . . . . . . . . . . . . . . . . . . . . . . . . 131
6.4. Bất phương trình mũ và logarit . . . . . . . . . . . . . . . . . . . . . . . . . . . . . . . . . . . . . . . . . 132
Vấn đề 1 : Bất phương trình cơ bản . . . . . . . . . . . . . . . . . . . . . . . . . . . . . . . . . . . . . . . 132
Vấn đề 2 : Phương pháp đặt ẩn phụ . . . . . . . . . . . . . . . . . . . . . . . . . . . . . . . . . . . . . . 133
Vấn đề 3 : Phương pháp phân tích thành nhân tử . . . . . . . . . . . . . . . . . . . . . . . . . . . . . . . 134
6.5. Hệ phương trình . . . . . . . . . . . . . . . . . . . . . . . . . . . . . . . . . . . . . . . . . . . . . . . . . 134
6.6. Phương trình mũ và lôgarit trong các kì thi tuyển sinh ĐH . . . . . . . . . . . . . . . . . . . . . . . . . . . 135
6.7. Bài tập tổng hợp . . . . . . . . . . . . . . . . . . . . . . . . . . . . . . . . . . . . . . . . . . . . . . . . . 136
Chương 7 Tích phân 149
7.1. Các dạng toán cơ bản về nguyên hàm . . . . . . . . . . . . . . . . . . . . . . . . . . . . . . . . . . . . . . 149
Vấn đề 1 : Chứng minh một hàm số F(x) là một nguyên hàm của hàm số f(x) . . . . . . . . . . . . . . . . 149
Vấn đề 2 : Sử dụng bảng nguyên hàm cơ bản . . . . . . . . . . . . . . . . . . . . . . . . . . . . . . . . . . 149
Vấn đề 3 : Tìm hằng số C . . . . . . . . . . . . . . . . . . . . . . . . . . . . . . . . . . . . . . . . . . . . 150
Vấn đề 4 : Phương pháp nguyên hàm từng phần . . . . . . . . . . . . . . . . . . . . . . . . . . . . . . . . 150
Vấn đề 5 : Phương pháp đổi biến số . . . . . . . . . . . . . . . . . . . . . . . . . . . . . . . . . . . . . . 151
7.2. Các dạng toán tích phân . . . . . . . . . . . . . . . . . . . . . . . . . . . . . . . . . . . . . . . . . . . . . 152
Vấn đề 1 : Sử dụng tích phân cơ bản . . . . . . . . . . . . . . . . . . . . . . . . . . . . . . . . . . . . . . 152
Vấn đề 2 : Tích phân hàm chứa dấu trị tuyệt đối . . . . . . . . . . . . . . . . . . . . . . . . . . . . . . . . 152
Vấn đề 3 : Phương pháp tích phân từng phần . . . . . . . . . . . . . . . . . . . . . . . . . . . . . . . . . 153
Vấn đề 4 : Phương pháp đổi biến số . . . . . . . . . . . . . . . . . . . . . . . . . . . . . . . . . . . . . . 154
Vấn đề 5 : Tích phân các hàm hữu tỉ . . . . . . . . . . . . . . . . . . . . . . . . . . . . . . . . . . . . . . 157
Vấn đề 6 : Tích phân một số hàm đặc biệt . . . . . . . . . . . . . . . . . . . . . . . . . . . . . . . . . . . 159
7.3. Ứng dụng tích phân để tính diện tích hình phẳng . . . . . . . . . . . . . . . . . . . . . . . . . . . . . . . . 161
7.4. Ứng dụng tích phân tính thể tích vật thể tròn xoay . . . . . . . . . . . . . . . . . . . . . . . . . . . . . . . 162
7.5. Tích phân trong các kì thi ĐH . . . . . . . . . . . . . . . . . . . . . . . . . . . . . . . . . . . . . . . . . 163
7.6. Bài tập tổng hợp . . . . . . . . . . . . . . . . . . . . . . . . . . . . . . . . . . . . . . . . . . . . . . . . . 164
Chương 8 Số phức 167
II Hình học 173
Chương 9 Phương pháp tọa độ trong trong mặt phẳng 175
9.1. Phương pháp tọa độ trong mặt phẳng . . . . . . . . . . . . . . . . . . . . . . . . . . . . . . . . . . . . . . 175
9.2. Phương trình của đường thẳng . . . . . . . . . . . . . . . . . . . . . . . . . . . . . . . . . . . . . . . . . 176
9.2.1. Các bài toán thiết lập phương trình đường thẳng . . . . . . . . . . . . . . . . . . . . . . . . . . . . 176
9.2.2. Các bài toán liên quan đến việc sử dụng phương trình đường thẳng . . . . . . . . . . . . . . . . . . 176
9.2.3. Bài tập tổng hợp . . . . . . . . . . . . . . . . . . . . . . . . . . . . . . . . . . . . . . . . . . . . 177
9.3. Đường tròn . . . . . . . . . . . . . . . . . . . . . . . . . . . . . . . . . . . . . . . . . . . . . . . . . . . 180
9.4. Đường elip . . . . . . . . . . . . . . . . . . . . . . . . . . . . . . . . . . . . . . . . . . . . . . . . . . . 183
9.5. Đường hypebol . . . . . . . . . . . . . . . . . . . . . . . . . . . . . . . . . . . . . . . . . . . . . . . . . 184
9.6. Đường parabol . . . . . . . . . . . . . . . . . . . . . . . . . . . . . . . . . . . . . . . . . . . . . . . . . 186
9.7. Phương pháp tọa độ trong mặt phẳng qua các kì thi tuyển sinh ĐH . . . . . . . . . . . . . . . . . . . . . . 187
9.8. Bài tập tổng hợp . . . . . . . . . . . . . . . . . . . . . . . . . . . . . . . . . . . . . . . . . . . . . . . . . 188
Chương 10 Mở đầu về hình học không gian. Quan hệ song song 191
10.1. Đại cương về đường thẳng và mặt phẳng . . . . . . . . . . . . . . . . . . . . . . . . . . . . . . . . . . . . 192
Vấn đề 1 : Xác định giao tuyến của hai mặt phẳng . . . . . . . . . . . . . . . . . . . . . . . . . . . . . . . 192
Vấn đề 2 : Xác định giao điểm của đường thẳng a và mặt phẳng (P) . . . . . . . . . . . . . . . . . . . . . 192
Vấn đề 3 : Phương pháp chứng minh ba điểm thẳng hàng và ba đường thẳng đồng quy . . . . . . . . . . . 193
Vấn đề 4 : Tìm thiết diện của hình chóp cắt bởi mặt phẳng . . . . . . . . . . . . . . . . . . . . . . . . . . . 193
10.2. Hai đường thẳng song song . . . . . . . . . . . . . . . . . . . . . . . . . . . . . . . . . . . . . . . . . . . 195
Vấn đề 1 : Tìm giao tuyến của hai mặt phẳng (dùng quan hệ song song) . . . . . . . . . . . . . . . . . . . 195
Vấn đề 2 : Chứng minh hai đường thẳng song song . . . . . . . . . . . . . . . . . . . . . . . . . . . . . . 196
Vấn đề 3 : Chứng minh hai đường thẳng chéo nhau . . . . . . . . . . . . . . . . . . . . . . . . . . . . . . 196
10.3. Đường thẳng và mặt phẳng song song . . . . . . . . . . . . . . . . . . . . . . . . . . . . . . . . . . . . . 197
Vấn đề 1 : Chứng minh đường thẳng song song với mặt phẳng . . . . . . . . . . . . . . . . . . . . . . . . 197
Vấn đề 2 : Tìm giao tuyến của hai mặt phẳng. Dựng thiết diện song song với một đường thẳng . . . . . . . 197
Vấn đề 3 : Dựng một mặt phẳng chứa một đường thẳng và song song với đường thẳng khác
Xác định giao điểm của đường thẳng với mặt phẳng . . . . . . . . . . . . . . . . . . . . . . . . . . 198
10.4. Hai mặt phẳng song song . . . . . . . . . . . . . . . . . . . . . . . . . . . . . . . . . . . . . . . . . . . . 199
Vấn đề 1 : Chứng minh hai mặt phẳng song song . . . . . . . . . . . . . . . . . . . . . . . . . . . . . . . 199
Vấn đề 2 : Tìm giao tuyến của hai mặt phẳng
Thiết diện cắt bởi một mặt phẳng song song với một mặt phẳng cho trước . . . . . . . . . . . . . . 199
Chương 11 Vectơ trong không gian. Quan hệ vuông góc 201
11.1. Vectơ trong không gian. Sự đồng phẳng của các vectơ . . . . . . . . . . . . . . . . . . . . . . . . . . . . . 202
Vấn đề 1 : Biểu thị một vectơ qua ba vectơ không đồng phẳng . . . . . . . . . . . . . . . . . . . . . . . . . 202
Vấn đề 2 : Chứng minh các đẳng thức vectơ . . . . . . . . . . . . . . . . . . . . . . . . . . . . . . . . . . 203
Vấn đề 3 : Chứng minh các điểm thẳng hàng và quan hệ song song . . . . . . . . . . . . . . . . . . . . . . 203
Vấn đề 4 : Chứng minh các vectơ đồng phẳng . . . . . . . . . . . . . . . . . . . . . . . . . . . . . . . . . 204
11.2. Hai đường thẳng vuông góc . . . . . . . . . . . . . . . . . . . . . . . . . . . . . . . . . . . . . . . . . . . 205
Vấn đề 1 : Tính góc giữa hai vectơ . . . . . . . . . . . . . . . . . . . . . . . . . . . . . . . . . . . . . . . 205
Vấn đề 2 : Tính góc giữa hai đường thẳng a và b . . . . . . . . . . . . . . . . . . . . . . . . . . . . . . . . 206
Vấn đề 3 : Chứng minh hai đường thẳng vuông góc . . . . . . . . . . . . . . . . . . . . . . . . . . . . . . 207
11.3. Đường thẳng vuông góc với mặt phẳng . . . . . . . . . . . . . . . . . . . . . . . . . . . . . . . . . . . . . 207
Vấn đề 1 : Chứng minh đường thẳng a vuông góc với mặt phẳng (P) . . . . . . . . . . . . . . . . . . . . . 207
Vấn đề 2 : Chứng minh hai đường thẳng vuông góc với nhau . . . . . . . . . . . . . . . . . . . . . . . . . 208
Vấn đề 3 : Xác định góc giữa đường thẳng a và mặt phẳng (P) . . . . . . . . . . . . . . . . . . . . . . . . 210
Vấn đề 4 : Dựng mặt phẳng qua điểm M cho trước và vuông góc với một đường thẳng d cho trước . . . . . 211
11.4. Hai mặt phẳng vuông góc . . . . . . . . . . . . . . . . . . . . . . . . . . . . . . . . . . . . . . . . . . . . 213
Vấn đề 1 : Xác định góc giữa hai mặt phẳng . . . . . . . . . . . . . . . . . . . . . . . . . . . . . . . . . . 213
Vấn đề 2 : Chứng minh hai mặt phẳng (P) và (Q) vuông góc . . . . . . . . . . . . . . . . . . . . . . . . . 214
Vấn đề 3 : Chứng minh đường thẳng a vuông góc với mặt phẳng (P) . . . . . . . . . . . . . . . . . . . . . 215
Vấn đề 4 : Dựng mặt phẳng (Q) chứa a và vuông góc với (P) (giả thiết a không vuông góc với (P)) . . . . . 216
11.5. Khoảng cách . . . . . . . . . . . . . . . . . . . . . . . . . . . . . . . . . . . . . . . . . . . . . . . . . . 217
Vấn đề 1 : Tính khoảng cách từ điểm M đến đường thẳng ∆ cho trước . . . . . . . . . . . . . . . . . . . . 217
Vấn đề 2 : Dựng đường thẳng đi qua một điểm A cho trước và vuông góc với một mặt phẳng (P) cho trước.
Khoảng cách từ điểm A đến mặt phẳng (P) . . . . . . . . . . . . . . . . . . . . . . . . . . . . . . 217
Vấn đề 3 : Đoạn vuông góc chung và khoảng cách giữa hai đường thẳng chéo nhau . . . . . . . . . . . . . 219
11.6. Khối đa diện và thể tích khối đa diện . . . . . . . . . . . . . . . . . . . . . . . . . . . . . . . . . . . . . . 222
Vấn đề 1 : Phương pháp trực tiếp tìm thể tích khối chóp . . . . . . . . . . . . . . . . . . . . . . . . . . . . 222
Vấn đề 2 : Tính thể tích hình chóp một cách gián tiếp . . . . . . . . . . . . . . . . . . . . . . . . . . . . . 227
Vấn đề 3 : Dùng công thức thể tích để giải một số bài toán hình học . . . . . . . . . . . . . . . . . . . . . 228
11.7. Phân loại một số hình khối đa diện . . . . . . . . . . . . . . . . . . . . . . . . . . . . . . . . . . . . . . . 230
11.7.1. Hình chóp có cạnh bên vuông góc với đáy . . . . . . . . . . . . . . . . . . . . . . . . . . . . . . . 230
11.7.2. Hình chóp đều . . . . . . . . . . . . . . . . . . . . . . . . . . . . . . . . . . . . . . . . . . . . . 231
11.7.3. Hình chóp có mặt bên vuông góc với đáy . . . . . . . . . . . . . . . . . . . . . . . . . . . . . . . 232
11.7.4. Hình chóp có hai mặt vuông góc với đáy . . . . . . . . . . . . . . . . . . . . . . . . . . . . . . . . 233
11.7.5. Hình chóp có các cạnh bên bằng nhau hoặc các cạnh bên cùng tạo với đáy những góc bằng nhau . . 233
11.7.6. Hình hộp - Hình lăng trụ . . . . . . . . . . . . . . . . . . . . . . . . . . . . . . . . . . . . . . . . 234
11.8. Bài tập tổng hợp . . . . . . . . . . . . . . . . . . . . . . . . . . . . . . . . . . . . . . . . . . . . . . . . . 235
Chương 12 Mặt cầu và khối tròn xoay 239
12.1. Mặt cầu, khối cầu . . . . . . . . . . . . . . . . . . . . . . . . . . . . . . . . . . . . . . . . . . . . . . . . 239
12.2. Mặt tròn xoay. Mặt trụ, hình trụ và khối trụ . . . . . . . . . . . . . . . . . . . . . . . . . . . . . . . . . . 243
Chương 13 Phương pháp không gian toạ độ trong không gian 249
13.1. Hệ toạ độ trong không gian . . . . . . . . . . . . . . . . . . . . . . . . . . . . . . . . . . . . . . . . . . . 249
Vấn đề 1 : Tìm tọa độ của một vectơ và các yếu tố liên quan đến vectơ thỏa mãn một số điều kiện cho trước . 249
Vấn đề 2 : Ứng dụng của tích vô hướng và tích có hướng . . . . . . . . . . . . . . . . . . . . . . . . . . . 249
Vấn đề 3 : Lập phương trình của mặt cầu . . . . . . . . . . . . . . . . . . . . . . . . . . . . . . . . . . . 252
Vấn đề 4 : Phương pháp tọa độ giải hình học không gian . . . . . . . . . . . . . . . . . . . . . . . . . . . 253
13.2. Phương trình mặt phẳng . . . . . . . . . . . . . . . . . . . . . . . . . . . . . . . . . . . . . . . . . . . . . 254
Vấn đề 1 : Viết phương trình mặt phẳng đi qua một điểm và có một vectơ pháp tuyến cho trước . . . . . . . 254
Vấn đề 2 : Vị trí tương đối của hai mặt phẳng . . . . . . . . . . . . . . . . . . . . . . . . . . . . . . . . . 255
Vấn đề 3 : Khoảng cách từ một điểm đến mặt phẳng . . . . . . . . . . . . . . . . . . . . . . . . . . . . . . 256
Vấn đề 4 : Góc giữa hai mặt phẳng . . . . . . . . . . . . . . . . . . . . . . . . . . . . . . . . . . . . . . . 258
Vấn đề 5 : Vị trí tương đối giữa mặt phẳng và mặt cầu . . . . . . . . . . . . . . . . . . . . . . . . . . . . . 258
13.3. Phương trình đường thẳng . . . . . . . . . . . . . . . . . . . . . . . . . . . . . . . . . . . . . . . . . . . 260
Vấn đề 1 : Phương trình tham số và phương trình chính tắc của đường thẳng . . . . . . . . . . . . . . . . . 260
Vấn đề 2 : Tìm điểm trên đường thẳng thỏa mãn điều kiện cho trước . . . . . . . . . . . . . . . . . . . . . 260
Vấn đề 3 : Vị trí tương đối của hai đường thẳng ∆ và ∆′ trong không gian . . . . . . . . . . . . . . . . . . 261
Vấn đề 4 : Vị trí tương đối giữa đường thẳng ∆ và mặt phẳng (P) . . . . . . . . . . . . . . . . . . . . . . . 262
Vấn đề 5 : Khoảng cách từ một điểm đến đường thẳng . . . . . . . . . . . . . . . . . . . . . . . . . . . . . 263
Vấn đề 6 : Vị trí tương đối giữa đường thẳng và mặt cầu . . . . . . . . . . . . . . . . . . . . . . . . . . . . 264
Vấn đề 7 : Góc giữa hai đường thẳng ; góc giữa đường thẳng và mặt phẳng . . . . . . . . . . . . . . . . . 266
Vấn đề 8 : Phương trình đường thẳng biết đường thẳng đó song song, hoặc vuông góc với đường thẳng
hoặc mặt phẳng khác, hoặc nằm trên mặt phẳng khác . . . . . . . . . . . . . . . . . . . . . . . . . 267
Vấn đề 9 : Phương trình đường thẳng ∆ biết ∆ cắt ∆′ . . . . . . . . . . . . . . . . . . . . . . . . . . . . . 268
Vấn đề 10 : Hình chiếu và tính đối xứng . . . . . . . . . . . . . . . . . . . . . . . . . . . . . . . . . . . . 270
Vấn đề 11 : Bài toán cực trị . . . . . . . . . . . . . . . . . . . . . . . . . . . . . . . . . . . . . . . . . . . 271
13.4. Hình học không gian trong các kì thi tuyển sinh ĐH . . . . . . . . . . . . . . . . . . . . . . . . . . . . . . 273
13.5. Bài tập tổng hợp . . . . . . . . . . . . . . . . . . . . . . . . . . . . . . . . . . . . . . . . . . . . . . . . . 278
III Hướng dẫn và đáp số 287
8
Phần I
Đại số - Lượng giác - Giải tích
9
www.VNMATH.com www.VNMATH.com
WWW.VNMATH.COM
WWW.VNMATH.COM
Chương 1
Phương trình, bất phương trình, hệ đại số
1.1 Phương trình, bất phương trình đa thức
1.1.1 Phương trình, bất phương trình bậc hai
Bài 1.1 : Giải và biện luận các phương trình sau :
1. (m − 2)x2 − 2mx + m + 1 = 0 ; 2.
a
x − 1
+
1
x − a
= 2.
Bài 1.2 : Cho phương trình :
(m2
− 4)x2
+ 2(m + 2)x + 1 = 0.
1. Tìm m để phương trình có hai nghiệm phân biệt.
2. Tìm m để phương trình có nghiệm duy nhất.
Bài 1.3 : Gọi a, b, c là độ dài ba cạnh của một tam giác. Chứng minh phương trình sau vô nghiệm :
c2
x2
+ (a2
− b2
− c2
)x + b2
= 0.
Bài 1.4 : Cho phương trình :
x2
− (2m + 3)x + m2
+ 2m + 2 = 0.
1. Tìm m để phương trình có hai nghiệm x1, x2.
2. Viết phương trình bậc hai có hai nghiệm
1
x1
,
1
x2
.
3. Tìm hệ thức giữa x1, x2 độc lập với tham số m.
4. Tìm m để phương trình có hai nghiệm x1, x2 thỏa mãn x1 = 2x2.
Bài 1.5 : Cho phương trình : x2 − cos a.x + sin a − 1 = 0.
1. Chứng minh rằng phương trình luôn có hai nghiệm x1, x2 với mọi a.
2. Tìm hệ thức giữa x1, x2 độc lập với a.
3. Tìm giá trị lớn nhất, giá trị nhỏ nhất của E = (x1 + x2)2 + x2
1x2
2.
Bài 1.6 : Cho phương trình :
mx2
− 2(m − 2)x + m − 3 = 0.
Tìm m để phương trình có :
11
WWW.VNMATH.COM
CHUYÊN ĐỀ LUYỆN THI ĐẠI HỌC
1. hai nghiệm trái dấu ; 2. hai nghiệm dương phân biệt ; 3. đúng một nghiệm âm.
Bài 1.7 : Giải các bất phương trình sau :
1.
x2 − 4x + 3
3 − 2x
< 1 − x ;
2. (−x2 + 3x − 2)(x2 − 5x + 6) ≥ 0 ;
3.
2
x + 2
+
1
2
≤
−4
x2 + 2x
;
4. x2 + (x + 1)2 ≤
15
x2 + x + 1
;
Bài 1.8 : Giải và biện luận các bất phương trình sau :
1. x2 − mx + m + 3 > 0 ; 2. (m + 1)x2 − 2(m − 1)x + 3m − 3 ≥ 0 ;
Bài 1.9 : Giải hệ bất phương trình sau :
x2 − 7x + 6 ≤ 0
x2 − 8x + 15 ≥ 0
Bài 1.10 : Tìm m để :
1. x2 − mx + m + 3 ≥ 0, ∀x ∈ R ; 2. mx2 + 4x + m > 0, ∀x ∈ R ; 3. mx2 − mx − 5 < 0, ∀x ∈ R.
Bài 1.11 : Tìm m để các hàm số sau xác định với mọi x ∈ R :
1. y = m(m + 2)x2 + 2mx + 2 ; 2. y =
1
(1 − m)x2 − 2mx + 5 − 9m
;
Bài 1.12 : Cho f(x) = (m + 1)x2 − 2(m − 1)x + 3m − 3. Tìm m để bất phương trình :
1. f(x) < 0 vô nghiệm. 2. f(x) ≥ 0 có nghiệm.
Bài 1.13 : Tìm m để các bất phương trình sau có tập nghiệm là R :
1. 1 ≤
3x2 − mx + 5
2x2 − x + 1
< 6 ; 2.
¬
¬
¬
¬
¬
x2 + mx + 1
x2 + 1
¬
¬
¬
¬
¬
< 2 ;
Bài 1.14 : Cho bất phương trình : x2 + 6x + 7 + m ≤ 0. Tìm m để bất phương trình :
1. vô nghiệm.
2. có đúng một nghiệm.
3. có miền nghiệm là một đoạn trên trục số có độ dài bằng 1.
Bài 1.15 : Tìm m để f(x) = mx2 − 4x + 3m + 1 > 0 với mọi x > 0.
Bài 1.16 : Tìm m để f(x) = 2x2 + mx + 3 ≥ 0 với mọi x ∈ [−1; 1].
Bài 1.17 : Tìm m để f(x) = x2 − 2mx − m ≥ 0 với mọi x > 0.
Bài 1.18 : Tìm m để f(x) = mx2 − 2(m + 1)x − m + 5 > 0 với mọi x < 1.
Bài 1.19 : Tìm m để f(x) = 2x2 − (3m + 1)x − (3m + 9) ≤ 0 với mọi x ∈ [−2; 1].
TRẦN ANH TUẤN - 0974 396 391 - (04) 66 515 343 Trang 12
www.VNMATH.com www.VNMATH.com
WWW.VNMATH.COM
CHUYÊN ĐỀ LUYỆN THI ĐẠI HỌC
1.1.2 Phương trình trình bậc ba
Bài 1.20 : Cho phương trình :
x3
− (m2
− m + 7)x − (3m2
+ m − 6) = 0.
1. Tìm m để phương trình có một nghiệm là −1.
2. Với m > 0 tìm được ở câu trên, hãy giải phương trình .
Bài 1.21 : Giải các phương trình sau :
1. x3 − 6x2 + 11x − 6 = 0 ;
2. 2x3 + x + 3 = 0 ;
3. x3 − 5x2 + 7x − 2 = 0 ;
4. x3 − 3
√
3x2 + 7x −
√
3 = 0 ;
Bài 1.22 : Tìm m để các phương trình sau có ba nghiệm phân biệt :
1. x3 − (2m + 1)x2 + 3(m + 4)x − m − 12 = 0 ; 2. mx3 − 2mx2 − (2m − 1)x + m + 1 = 0 ;
Bài 1.23 : Tìm m để phương trình :
mx3
− (3m − 4)x2
+ (3m − 7)x − m + 3 = 0
có ba nghiệm dương phân biệt.
1.1.3 Phương trình, bất phương trình bậc bốn
Bài 1.24 : Giải các phương trình sau :
1. x4 − 3x2 + 4 = 0 ;
2. (x − 1)(x + 5)(x − 3)(x + 7) = 297 ;
3. (x + 2)(x − 3)(x + 1)(x + 6) = −36 ;
4. x4 + (x − 1)4 = 97 ;
5. (x + 3)4 + (x + 5)4 = 16 ;
6. 6x4 − 35x3 + 62x2 − 35 + 6 = 0 ;
7. x4 + x3 − 4x2 + x + 1 = 0 ;
8. x4 − 5x3 + 10x2 − 10x + 4 = 0 ;
9. x4 − x2 + 6x − 9 = 0 ;
10. 2x4 − x3 − 15x2 − x + 3 = 0.
Bài 1.25 : Tìm các giá trị của m sao cho phương trình
x4
+ (1 − 2m)x2
+ m2
− 1 = 0.
1. Vô nghiệm ; 2. Có hai nghiệm phân biệt ; 3. Có bốn nghiệm phân biệt.
Bài 1.26 : Tìm các giá trị của a sao cho phương trình
(a − 1)x4
− ax2
+ a2
− 1 = 0
có ba nghiệm phân biệt.
Bài 1.27 : Cho phương trình :
(m − 1)x4
+ 2(m − 3)x2
+ m + 3 = 0.
Tìm m để phương trình trên vô nghiệm.
TRẦN ANH TUẤN - 0974 396 391 - (04) 66 515 343 Trang 13
www.VNMATH.com www.VNMATH.com
WWW.VNMATH.COM
CHUYÊN ĐỀ LUYỆN THI ĐẠI HỌC
Bài 1.28 : Cho phương trình :
x4
− (2m + 1)x2
+ m + 3 = 0.
Tìm m để phương trình trên có bốn nghiệm phân biệt, trong đó một nghiệm bé hơn −2 và ba nghiệm còn lại lớn hơn −1.
Bài 1.29 : Tìm h để phương trình sau đây có không ít hơn hai nghiệm âm khác nhau :
x4
+ hx3
+ x2
+ hx + 1 = 0.
Bài 1.30 : Cho phương trình :
(x + 1)(x + 2)(x + 3)(x + 4) = m.
Tìm m để phương trình có bốn nghiệm phân biệt.
1.2 Phương trình, bất phương trình chứa giá trị tuyệt đối
1. Phương trình (bất phương trình) | f(x)| + g(x) < 0 (hoặc = , hoặc > , hoặc ≥ , hoặc ≤ ) tương đương với
f(x) ≥ 0
f(x) + g(x) < 0
hoặc
f(x) < 0
− f(x) + g(x) < 0.
Một số phương trình hoặc bất phương trình chứa nhiều hơn một dấu giá trị tuyệt đối thì việc phá dấu giá trị tuyệt đối
sẽ phức tạp hơn nhiều, phải chia thành nhiều trường hợp bằng cách lập bảng xét dấu các biểu thức trong dấu giá trị
tuyệt đối.
2. Phương trình (bất phương trình) | f(x)| < |g(x)| (hoặc = , hoặc > , hoặc ≥ , hoặc ≤ ) phương pháp đơn giản là bình
phương hai vế, chuyển vế, phân tích thành nhân tử.
3. Một số phương trình và bất phương trình thông dụng (giả sử a > 0).
• |x| = a ⇔ x = a hoặc x = −a.
• |x| < a ⇔ −a < x < a.
• |x| ≤ a ⇔ −a ≤ x ≤ a.
• |x| > a ⇔ x < −a hoặc x > a.
• |x| ≥ a ⇔ x ≤ −a hoặc x ≥ a.
Bài 1.31 : Giải phương trình |x2 − 8x + 15| = x − 3.
Bài 1.32 : Giải các phương trình và bất phương trình sau :
1. |x2 − 5x + 4| = x2 + 6x + 5;
2. |x − 1| = 2x − 1;
3. | − x2 + x − 1| ≤ 2x + 5;
4. |x2 − x| ≤ |x2 − 1|.
Bài 1.33 : Giải các phương trình và bất phương trình sau :
1.
¬
¬
¬
¬
¬
x2 − 2
x + 1
¬
¬
¬
¬
¬
= 2; 2.
¬
¬
¬
¬
3x + 4
x − 2
¬
¬
¬
¬ ≤ 3; 3.
¬
¬
¬
¬
2x − 3
x − 3
¬
¬
¬
¬ ≥ 1; 4. |2x + 3| = |4 − 3x|.
Bài 1.34 : Giải các bất phương trình sau :
TRẦN ANH TUẤN - 0974 396 391 - (04) 66 515 343 Trang 14
www.VNMATH.com www.VNMATH.com
WWW.VNMATH.COM
CHUYÊN ĐỀ LUYỆN THI ĐẠI HỌC
1. |x2 − 5x + 4| ≤ x2 + 6x + 5; 2. 4x2 + 4x − |2x + 1| ≥ 5.
Bài 1.35 : Giải các bất phương trình sau :
1.
¬
¬
¬
¬1 −
|x|
1 + |x|
¬
¬
¬
¬ ≥
1
2
;
2. log5 log¹⁄₂
x2 − 4|x|
|x| − 7
≤ 0 ;
3. |x2 − 2x − 8| > 2x ;
4. |x3 − 7x − 3| < x3 + x2 + 3 ;
5. |x3 − x2 + 4| + x3 − x2 − 2x − 2 ≤ 0 ;
6. ||x| − 1| < 1 − x ;
7.
¬
¬|x2 − 3x − 7| + 2x − 1
¬
¬ < x2 − 8x − 5 ;
8.
¬
¬x2 − |x2 − 3x − 5| − 5
¬
¬ < x + 1 ;
9. |x − 1| + |x − 2| > 3 + x ;
10. log3
|x2 − 4x| + 3
x2 + |x − 5|
≥ 0 ;
11. ||3x + 4x − 9| − 8| ≤ 3x − 4x − 1 ;
Bài 1.36 : Giải các bất phương trình sau :
1. |3x + 2| + |2x − 3| < 11 ;
2. |x2 − 3x − 7| + |2x2 − x − 9| + |3x2 − 7x − 5| < x + 15 ;
3. |x − 1| + |2 − x| > 3 + x ;
4. |x2 − 3x − 17| − |x2 − 5x − 7| > 3.
Bài 1.37 : Tìm m để bất phương trình : x2 + |x + m| < 2 có ít nhất một nghiệm âm.
Bài 1.38 : Giải và biện luận bất phương trình sau theo tham số p :
2|x − p| + 5|x − 3p| + 4x + 6p + 12 ≤ 0.
Bài 1.39 : Giải và biện luận bất phương trình sau theo tham số p :
|2x + 21p| − 2|2x − 21p| < x − 21p.
Bài 1.40 : Tìm tất cả các giá trị thực của tham số a sao cho bất phương trình
x2
− |x − a| − |x − 1| + 3 ≥ 0
đúng với mọi x ∈ R.
Bài 1.41 : Tìm tất cả các giá trị của a sao cho giá trị nhỏ nhất của hàm số
y = x2
+ 2x − 1 + |x − a|
lớn hơn 2.
Bài 1.42 : Tìm tất cả các giá trị của a sao cho giá trị nhỏ nhất của hàm số
y = x2
+ |x − a| + |x − 1|
lớn hơn 2.
Bài 1.43 : Tìm tất cả các giá trị của a sao cho giá trị nhỏ nhất của hàm số
y = ax + |x2
− 4x + 3|
lớn hơn 1.
Bài 1.44 : Tìm tất cả các giá trị của a sao cho giá trị lớn nhất của hàm số
y = 4x − x2
+ |x − m|
nhỏ hơn 4.
TRẦN ANH TUẤN - 0974 396 391 - (04) 66 515 343 Trang 15
www.VNMATH.com www.VNMATH.com
WWW.VNMATH.COM
CHUYÊN ĐỀ LUYỆN THI ĐẠI HỌC
1.3 Phương trình, bất phương trình chứa căn
Vấn đề 1 : Phương trình, bất phương trình cơ bản
Phương pháp chung là tìm cách bình phương hai vế (để giảm số căn, hoặc mất căn) với điều kiện là hai vế của phương trình
phải không âm.
1. Phương trình
√
f(x) =
√
g(x) ⇔
f(x) ≥ 0 (hoặc cũng có thể xét g(x) ≥ 0)
f(x) = g(x).
2. Phương trình
√
f(x) = g(x) ⇔
g(x) ≥ 0
f(x) = (g(x))2
.
3. Bất phương trình
√
f(x) >
√
g(x) (hoặc ≥ ) tương đương với
g(x) ≥ 0
f(x) > g(x).
4. Bất phương trình
√
f(x) < g(x) (hoặc ≤ ) tương đương với
f(x) ≥ 0
g(x) ≥ 0
f(x) < (g(x))2
.
5. Bất phương trình
√
f(x) > g(x) (hoặc ≥ ) tương đương với
(I)
f(x) ≥ 0
g(x) < 0
hoặc (II)
g(x) ≥ 0
f(x) > (g(x))2
.
Bài 1.45 : Giải phương trình
√
x2 + 56x + 80 = x + 20.
Bài 1.46 : Giải bất phương trình
√
x2 − 2x − 15 < x − 3.
Bài 1.47 : Giải bất phương trình
√
x2 − 1 > x + 2.
Bài 1.48 : Giải các phương trình sau :
1.
√
2x2 + 4x − 1 = x + 1;
2.
√
4x2 + 101x + 64 = 2(x + 10);
3.
√
x2 + 2x = −2x2 − 4x + 3;
4.
√
(x + 1)(x + 2) = x2 + 3x − 4.
Bài 1.49 : Giải các bất phương trình:
1.
√
x2 + x − 6 < x − 1;
2.
√
2x − 1 ≤ 2x − 3;
3.
√
2x2 − 1 > 1 − x;
4.
√
x2 − 5x − 14 ≥ 2x − 1.
Bài 1.50 : Tìm tập xác định của mỗi hàm số sau :
TRẦN ANH TUẤN - 0974 396 391 - (04) 66 515 343 Trang 16
www.VNMATH.com www.VNMATH.com
WWW.VNMATH.COM
CHUYÊN ĐỀ LUYỆN THI ĐẠI HỌC
1. y =
¬
¬x2 + 3x − 4
¬
¬ − x + 8;
2. y =
x2 + x + 1
|2x − 1| − x − 2
;
3. y =
Ö
1
x2 − 7x + 5
−
1
x2 + 2x + 5
;
4. y =
√
x2 − 5x − 14 − x + 3.
Bài 1.51 : Giải các phương trình sau :
1.
√
5x2 − 6x − 4 = 2(x − 1); 2.
√
x2 + 3x + 12 = x2 + 3x.
Bài 1.52 : Giải các bất phương trình sau :
1.
√
x2 + 6x + 8 ≤ 2x + 3;
2.
2x − 4
√
x2 − 3x − 10
> 1;
3. 6
√
(x − 2)(x − 3) ≤ x2 − 34x + 48 ;
4.
√
x2 − x − 12 ≥ x − 1;
5.
√
x2 − 4x − 12 > 2x + 3;
6.
√
x + 5
1 − x
< 1.
Vấn đề 2 : Phương pháp đặt ẩn phụ
Chúng ta thường sử dụng một số quy tắc đặt ẩn phụ như sau :
1. Nếu phương trình chứa hai loại căn, có thể
(a) Đặt u =
n√
ax + b, rút x, thế vào phương trình được phương trình ẩn u.
(b) Hoặc cũng có thể đặt u = n
√
u(x), v = m
√
v(x), lũy thừa để rút ra ràng buộc giữa u và v để được 1 phương trình
theo u, v. Kết hợp với phương trình ban đầu, ta được hệ hai ẩn u, v.
2. Đặt u = n
√
u(x), lũy thừa hai vế được phương trình chứa u, x. Kết hợp với phương trình ban đầu, ta được hệ hai ẩn
u, x.Giải phương trình bậc hai (có ∆ là bình phương một số).
3. Đặt ẩn phụ không hoàn toàn, đặt u =
√
u(x), đưa về phương trình bậc hai theo u với x coi như là tham số.
4. Nếu phương trình chứa
√
a ±
√
b và
√
ab ta thường đặt u =
√
a ±
√
b.
5. phương trình đẳng cấp, chẳng hạn đẳng cấp bậc 2 : A.x2 + B.xy + C.y2 = 0. Có cách giải như sau :
(a) Xét y = 0, rút được x;
(b) Xét y 0, chia cả hai vế cho y2, đặt u =
x
y
, đưa được về phương trình bậc hai theo u.
Bài 1.53 : Giải các phương trình sau :
1. 3x2 + 21x + 18 + 2
√
x2 + 7x + 7 = 2 ;
2. x2 +
√
x + 1 = 1 ;
3. 2(x2 + 2) = 5(x3 + 1) ;
4. 2x2 − 3x + 2 = x
√
3x − 2 ;
5. 6x2 − 10x + 5 − (4x − 1)
√
6x2 − 6x + 5 = 0 ;
6.
4
√
97 − x + 4
√
x = 5 ;
TRẦN ANH TUẤN - 0974 396 391 - (04) 66 515 343 Trang 17
www.VNMATH.com www.VNMATH.com
WWW.VNMATH.COM
CHUYÊN ĐỀ LUYỆN THI ĐẠI HỌC
Bài 1.54 : Giải các phương trình sau :
1.
√
x + 3 +
√
3x + 1 = 2
√
x +
√
2x + 2 ;
2.
√
2x2 + x + 6 +
√
x2 + x + 2 = x +
4
x
;
3. x2 + 2x
Ö
x −
1
x
= 3x + 1 ;
4. 4
√
x +
4√
x + 1 = 2
4√
2x + 1 ;
5.
√
x2 + 4x + 3 +
√
x2 + x =
√
3x2 + 4x + 1 ;
6. 3
√
x +
√
5 − x ≤ 3 ;
7.
3√
x − 1 +
3√
x + 1 = x
3√
2 ;
8. 3
√
x +
3√
x − 16 =
3
√
x − 8 ;
9.
3√
2x3 − 1 +
3√
1 − x3 = x ;
10.
√
x2 − x + 1 +
√
x2 + x + 1 = 2 ;
11.
√
2x2 + x + 9 +
√
2x2 − x + 1 = x + 4.
Bài 1.55 : Giải các phương trình sau :
1.
√
1 − x +
√
1 + x + 2
√
1 − x2 = 4 ;
2. 2x +
√
x + 1 +
√
x + 2
√
x2 + x = 1 ;
3. x2 + 2x +
√
x + 3 + 2x
√
x + 3 = 9 ;
4. 2x2 + x +
√
x2 + 3 + 2x
√
x2 + 3 = 9 ;
Bài 1.56 : Giải các phương trình sau :
1. 2x2 + x + 3 = 3x
√
x + 3 ;
2.
√
x + 8 =
3x2 + 7x + 8
4x + 2
;
3.
√
x2 + x + 2 =
3x2 + 3x + 2
3x + 1
;
4.
x + 2 + x
√
2x + 1
x +
√
2x + 1
=
√
x + 2 ;
5. (
√
x + 3 −
√
x + 1)(x2 +
√
x2 + 4x + 3) = 2x.
Bài 1.57 : Giải các phương trình sau :
1.
3
√
x + 1 +
3
√
x + 2 = 1 +
3√
x2 + 3x + 2 ;
2.
3√
x + 1 +
3√
x2 = 3
√
x +
3√
x2 + x ;
3.
4√
x + 1 +
√
x = 1 +
4√
x3 + x2 ;
4.
√
x + 3 + 2x
√
x + 1 = 2x +
√
x2 + 4x + 3 ;
5.
√
x3 + x2 + 3x + 3 +
√
2x =
√
x2 + 3 +
√
2x2 + 2x ;
6.
√
x + 3 +
4x
√
x + 3
= 4
√
x ;
7. 4
√
x + 3 = 1 + 4x +
3
x
;
8. 2
√
x + 3 = 9x2 − x − 4 ;
9. 12
√
x + 2
√
x − 1 = 3x + 9 ;
Bài 1.58 : Giải các phương trình sau :
1.
√
x + 3 + 3
√
x = 3 ;
2. 4
√
x +
4
√
x − 1 =
4
√
x + 1 ;
3.
√
2 − x2 = (2 −
√
x)2 ;
4. 2x + 1 + x
√
x2 + 2 + (x + 1)
√
x2 + 2x + 3 = 0 ;
5. x2 √
x + (x − 5)2
√
5 − x = 11(
√
x +
√
5 − x) ;
6. 2x3 = 1 +
3
Ö
x + 1
2
;
Bài 1.59 : Giải các phương trình sau :
1.
8
√
1 − x + 8
√
x = 1 ;
2. 2
√
x +
4
√
1 − 2x = 1 ;
3.
√
x + 4 +
√
x +
√
1 − x = 3 ;
4.
2 +
√
x
3 +
√
1 − x
=
√
x +
√
1 − x ;
TRẦN ANH TUẤN - 0974 396 391 - (04) 66 515 343 Trang 18
www.VNMATH.com www.VNMATH.com
WWW.VNMATH.COM
CHUYÊN ĐỀ LUYỆN THI ĐẠI HỌC
Vấn đề 3 : Phương pháp nhân liên hợp
Dạng 1 : Phương trình dạng
√
u(x) ±
√
v(x) = f(x), trong đó f(x) và u(x) − v(x) có cùng nghiệm x = x0.
(a) Phương trình trở thành
u(x) − v(x)
√
u(x) ∓
√
v(x)
= f(x).
(b) Chuyển vế, đặt (x − x0) làm nhân tử chung.
Dạng 2 : Phương trình dạng ( n
√
u1(x) ± n
√
v1(x)) + ( m
√
u2(x) ± m
√
v2(x)) = f(x), trong đó f(x); u1(x) − v1(x); u2(x) − v2(x) có
cùng nghiệm x = x0 (ở đây f(x) có thể đồng nhất bằng 0).
Phương pháp giải loại này là chúng ta nhân liên hợp theo từng cụm, đặt (x − x0) làm nhân tử chung.
Bài 1.60 : Giải các phương trình, các bất phương trình sau :
1. 3(2 +
√
x − 2) = 2x +
√
x + 6;
2.
x2
1 +
√
1 + x
2 > x − 4;
3.
√
x − 2 +
√
4 − x = x2 − 6x + 11;
4.
√
x − 2 +
√
4 − x = 2x2 − 5x − 1;
5.
Ö
1 − x
x
=
2x + x2
1 + x2
;
6. x2 + x − 1 = (x + 2)
√
x2 − 2x + 2;
7.
3
√
x + 24 +
√
12 − x = 6;
8. 2
√
x2 − 7x + 10 = x +
√
x2 − 12x + 20;
9. 2x2 − 11x + 21 = 3
3
√
4x − 4;
10.
√
5x − 1 +
3
√
9 − x = 2x2 + 3x − 1.
Bài 1.61 : Giải các phương trình sau :
1.
√
x + 4 −
√
2x + 3 = x − 1 ;
2. x +
√
2x =
1
x
+
Ö
x +
1
x
;
3. (x − 1)
√
x + 1 +
√
2x + 1 =
√
x + 2 ;
4.
1
x2
+
√
x + 5 =
1
x
+
√
2x + 4 ;
5. 2 +
√
x + 6 =
√
2x + 5 +
√
x + 3 ;
6. 1 +
4
√
x + 3 = x +
√
2x ;
7.
√
x + 2 +
√
x + 6 =
√
2x + 5 +
√
2x + 1 ;
8.
4√
x + 8 +
√
x + 4 =
√
2x + 3 +
√
3x
Vấn đề 4 : Phương pháp đánh giá
Cơ sở của phương pháp này là chúng ta sử dụng bất đẳng thức hoặc phương pháp hàm số đế đánh giá.
Cách 1 : Cơ sở nhận dạng :
(a) Nếu hàm số y = f(x) đồng biến trên (a; b) và hàm số y = g(x) nghịch biến trên (a; b) thì phương trình
f(x) = g(x) nếu có nghiệm thì nghiệm đó là duy nhất.
(b) Nếu hàm số y = f(x) đồng biến (hoặc nghịch biến) trên (a; b) thì phương trình f(x) = c (với c là hằng số)
nếu có nghiệm thì nghiệm đó là duy nhất.
TRẦN ANH TUẤN - 0974 396 391 - (04) 66 515 343 Trang 19
www.VNMATH.com www.VNMATH.com
WWW.VNMATH.COM
CHUYÊN ĐỀ LUYỆN THI ĐẠI HỌC
Phương pháp giải là :
(a) Nhận thấy x = x0 là một nghiệm của phương trình đã cho.
(b) Nếu x > x0, ta suy ra vế trái lớn hơn vế phải hoặc ngược lại.
(c) Nếu x < x0, ta suy ra vế trái lớn hơn vế phải hoặc ngược lại.
(d) Kết luận phương trình đã cho có nghiệm duy nhất x = x0.
Cách 2 : Nếu hàm số y = f(x) đồng biến (hoặc nghịch biến) trên (a; b) thì phương trình f(u) = f(v) tương đương với
u = v.
Cách 3 : Nếu hàm số y = f(x) thỏa mãn f′(x) = 0 có nhiều hơn 1 nghiệm thì chúng ta lập bảng biến thiên để suy ra
phương trình có tối ta bao nhiêu nghiệm, rồi nhẩm đủ số nghiệm đó, dẫn đến đó là tất cả các nghiệm của phương
trình.
Cách 4 : Nếu f(x) ≥ c và g(x) ≤ c thì phương trình f(x) = g(x) tương đương với
f(x) = c
g(x) = c.
Bài 1.62 : Giải các phương trình sau :
1.
√
x + 3 + 3
√
x = 3 ;
2.
√
x + 3 + x +
√
x + 8 = 4 ;
3.
√
x2 − x + 1 +
√
x2 + 7x + 1 = 4
√
x ;
4.
√
x + 3
1 +
√
2 − x
+
√
2x − 1 = 2 ;
5.
√
x2 − x + 4 +
√
2x − 1 = 5 ;
Vấn đề 5 : Phương trình, bất phương trình có tham số
1. Sử dụng phương trình, bất phương trình cơ bản;
2. Sử dụng đặt ẩn phụ, và đặt điều kiện "chặt" cho ẩn;
3. Sử dụng điều kiện có nghiệm của phương trình bậc hai;
4. Sử dụng phương pháp hàm số để chỉ ra điều kiện có nghiệm.
Bài 1.63 : Tìm điều kiện của m để phương trình
√
x2 + 2x − m = 2x − 1 :
1. có nghiệm thực ; 2. có đúng một nghiệm thực ; 3. có hai nghiệm thực phân biệt.
Bài 1.64 : Tìm điều kiện của m để phương trình x + x +
1
2
+
Ö
x +
1
4
= m có nghiệm thực.
Bài 1.65 : Tìm điều kiện của m để phương trình
√
16 − x2 −
m
√
16 − x2
− 4 = 0 có nghiệm thực.
TRẦN ANH TUẤN - 0974 396 391 - (04) 66 515 343 Trang 20
www.VNMATH.com www.VNMATH.com
WWW.VNMATH.COM
CHUYÊN ĐỀ LUYỆN THI ĐẠI HỌC
Bài 1.66 : Tìm điều kiện của m để phương trình
Ö
x − 1
x + 2
− m
Ö
x + 2
x − 1
+ 2 = 0 có nghiệm thực.
Bài 1.67 : Tìm điều kiện của m để phương trình
√
x + 1 − m
√
x − 1 + 2
4√
x2 − 1 = 0 có nghiệm thực.
Bài 1.68 : Tìm điều kiện của m để phương trình
√
x2 − 2x − 3 = x + m
TRẦN ANH TUẤN - 0974 396 391 - (04) 66 515 343 Trang 21
www.VNMATH.com www.VNMATH.com
WWW.VNMATH.COM
CHUYÊN ĐỀ LUYỆN THI ĐẠI HỌC
1. có nghiệm thực ; 2. có hai nghiệm thực phân biệt.
Bài 1.69 : Biện luận theo m số nghiệm thực của phương trình
√
x + 1 +
√
1 − x = m.
Bài 1.70 : Tìm điều kiện m để phương trình
√
x +
√
9 − x =
√
−x2 + 9x + m có nghiệm thực.
Bài 1.71 : Tìm điều kiện m để phương trình x + 4
√
x − 4 + x +
√
x − 4 = m có nghiệm thực.
Bài 1.72 : Tìm điều kiện m để phương trình x + 6
√
x − 9 + x − 6
√
x − 9 =
x + m
6
có nghiệm thực.
Bài 1.73 : Tìm m để phương trình
√
x4 + 4x + m +
4√
x4 + 4x + m = 6 có nghiệm thực.
Bài 1.74 : Tìm điều kiện của m để phương trình
√
1 − x2 + 2
3√
1 − x2 = m :
1. có nghiệm thực duy nhất ; 2. có nghiệm thực.
Bài 1.75 : Chứng tỏ rằng phương trình
3x2 − 1
√
2x − 1
=
√
2x − 1 + mx luôn có nghiệm thực với mọi giá trị của m.
Bài 1.76 : Tìm m để phương trình (x − 3)(x + 1) + 4(x − 3)
Ö
x + 1
x − 3
= m có nghiệm thực.
Bài 1.77 : Tìm m để phương trình
3√
1 − x +
3√
1 + x = m có nghiệm thực.
Bài 1.78 : Biện luận theo m số nghiệm thực của phương trình m
√
x2 + 2 = x + m.
Bài 1.79 : Tìm m để phương trình
√
x2 − 2x − 3 = mx + m có nghiệm thực x −1.
Bài 1.80 : Tìm m để phương trình sau có nghiệm thực :
√
x +
√
1 − x + 2m x(1 − x) − 2 4
x(1 − x) = m.
Bài 1.81 : Tìm m để phương trình x +
√
x2 − x + 1 = m có nghiệm thực.
Bài 1.82 : Tìm m để các phương trình sau có nghiệm thực :
1.
√
x2 + x + 1 −
√
x2 − x + 1 = m ; 2.
4√
x2 + 1 −
√
x = m.
Bài 1.83 : Tìm m để phương trình sau có nghiệm thực :
x
√
x +
√
x + 12 = m
√
5 − x +
√
4 − x .
Bài 1.84 : Tìm m để phương trình sau có nghiệm thực :
m
√
x − 2 + 2
4√
x2 − 4 −
√
x + 2 = 2
4√
x2 − 4.
Bài 1.85 : Tìm m để phương trình sau có nghiệm thực :
(4m − 3)
√
x + 3 + (3m − 4)
√
1 − x + m − 1 = 0.
Bài 1.86 : Tìm m để phương trình sau có 4 nghiệm thực phân biệt :
m
√
1 + x2 −
√
1 − x2 + 2 = 2
√
1 − x4 +
√
1 + x2 −
√
1 − x2.
Bài 1.87 : Tìm m để phương trình sau có nghiệm duy nhất :
√
x2 − 2x =
√
mx + 1.
TRẦN ANH TUẤN - 0974 396 391 - (04) 66 515 343 Trang 22
www.VNMATH.com www.VNMATH.com
WWW.VNMATH.COM
CHUYÊN ĐỀ LUYỆN THI ĐẠI HỌC
Bài 1.88 : Tìm m để phương trình sau có nghiệm :
x +
√
1 − x2 = m.
Bài 1.89 : Cho phương trình :
−x2
+ 2x + 4 (3 − x)(x + 1) = m − 3.
1. Tìm m để phương trình có nghiệm.
2. Tìm m để phương trình có đúng hai nghiệm phân biệt.
Bài 1.90 : Tìm m để phương trình sau có nghiệm :
√
x + 1 +
√
3 − x − (x + 1)(3 − x) = m.
Bài 1.91 : Cho phương trình :
|x + 1| + m|x − 1| = (m + 1)
√
x2 − 1.
1. Giải phương trình khi m = 2 ;
2. Tìm m để phương trình trên có nghiệm.
Bài 1.92 : Tìm m để các bất phương trình sau có nghiệm :
1.
√
4 − x +
√
x + 5 ≥ m ; 2. mx −
√
x − 3 ≤ m + 1.
Bài 1.93 : Tìm m để bất phương trình
m
√
x2 − 2x + 2 + 1 + x(2 − x) ≤ 0
có nghiệm trong đoạn
ä
0; 1 +
√
3
ç
.
Bài 1.94 : Tìm m để bất phương trình
√
(4 + x)(6 − x) ≤ x2 − 2x + m nghiệm đúng với mọi x ∈ [−4; 6].
1.4 Hệ phương trình
1.4.1 Phương pháp thế
Bài 1.95 : Giải các hệ phương trình sau :
1.
x2(y + 1)(x + y + 1) = 3x2 − 4x + 1
xy + x + 1 = x2
2.
x3y = 16
3x + y = 8
3.
y(1 + x2) = x(1 + y2)
x2 + 3y2 = 1
4.
x −
1
x
= y −
1
y
2y = x3 + 1
5.
√
x + y = 3
√
x + y
√
x − y = 3
√
x − y − 12
6.
√
x + y −
√
x − y = 2
x2 + y2 + x2 − y2 = 4
TRẦN ANH TUẤN - 0974 396 391 - (04) 66 515 343 Trang 23
www.VNMATH.com www.VNMATH.com
WWW.VNMATH.COM
CHUYÊN ĐỀ LUYỆN THI ĐẠI HỌC
7.
x3 − 8x = y3 + 2y
x2 − 3 = 3(y2 + 1)
8.
|x2 − 2x| + y = 1
x2 + |y| = 1
9.
x2 + y2 +
2xy
x + y
= 1
√
x + y = x2 − y
10.
√
7x + y +
√
2x + y = 5
√
2x + y + x − y = 2
11.
√
3x 1 +
1
x + y
= 2
√
7y 1 −
1
x + y
= 4
√
2
12.
x3 + 3xy2 = −49
x2 − 8xy + y2 = 8y − 17x
13.
√
y(
√
x +
√
x + 3) = 3
√
x +
√
y = x + 1
14.
√
x + 1 +
1
y
=
Ö
x
y
√
xy +
√
y + 1 +
√
1 − x = 1
1.4.2 Phương pháp phân tích thành nhân tử hoặc coi một phương trình là phương trình bậc hai (ba)
theo một ẩn
Bài 1.96 : Giải các hệ phương trình sau :
1.
x3 + 3y2x = 4
y3 + 3x2y = 4;
2.
x2 + y + 1 = 0
x + y2 + 1 = 0;
3.
3x3 = x2 + 2y2
3y3 = y2 + 2x2;
4.
x3 = 3x + 8y
y3 = 3y + 8x;
5.
x − 3y =
4y
x
y − 3x =
4x
y
;
6.
x3 = 5x + y
y3 = 5y + x.
Bài 1.97 : Giải các hệ phương trình sau :
1.
x2 = 3x + 2y
y2 = 3y + 2x
2.
x2 − 2y2 = 2x + y
y2 − 2x2 = 2y + x
3.
x3 = 2x + y
y3 = 2y + x
4.
xy + x + y = x2 − 2y2
x
√
2y − y
√
x − 1 = 2x − 2y
5.
y2 = (5x + 4)(4 − x)
y2 − 5x2 − 4xy + 16x − 8y + 16 = 0
6.
x3 + 1 = 2y
y3 + 1 = 2x
7.
√
x + y +
√
x − y = 1 + x2 − y2
√
x +
√
y = 1
8.
x2y + 2x + 3y = 6
3xy + x + y = 5
1.4.3 Phương pháp đặt ẩn phụ
Bài 1.98 : Giải các hệ phương trình sau :
1.
x + xy + y = 11
x − xy + y = 1;
TRẦN ANH TUẤN - 0974 396 391 - (04) 66 515 343 Trang 24
www.VNMATH.com www.VNMATH.com
WWW.VNMATH.COM
CHUYÊN ĐỀ LUYỆN THI ĐẠI HỌC
2.
x2y + xy2 = 20
1
x
+
1
y
=
5
4
;
3.
x2 + y2 = 2x2y2
x + y + 1 = 3xy;
4.
x − y + xy = 1
x2 + y2 = 2;
5.
x2 + y2 + x2y2 = 1 + 2xy
(x − y)(1 + xy) = 1 − xy;
6.
x
y
+
y
x
=
26
5
x2 − y2 = 24;
7.
x2 + y2 + xy = 3
xy3 + yx3 = 2;
8.
x
y
+
y
x
= 2
1
x
+
1
y
+ x + y = 4;
9.
x + y +
x
y
+
y
x
= 4
x + y +
x2
y
+
y2
x
= 4;
10.
x + y + x2y2 = 3xy
1
x
+
1
y
− xy = 1;
11.
x2 + y2 + xy = 3x2y2
x2 + y2 − xy = x2y2;
12.
x + xy + y = 7
x2 + xy + y2 = 13;
Bài 1.99 : Giải các hệ phương trình sau :
1.
x + y + xy = 5
x2 + y2 + xy = 7
2.
x
y
+
y
x
=
13
6
x + y = 5
3.
x2 + xy + y2 = 1
x − y − xy = 3
4.
x2 + 1 + y(x + y) = 4y
(x2 + 1)(y + x − 2) = y
5.
4xy + 4(x2 + y2) +
3
(x + y)2
= 7
2x +
1
x + y
= 3
6.
x2 − 3xy + y2 = −1
3x2 − xy + 3y2 = 13
7.
2x2 − 4xy + y2 = −1
3x2 + 2xy + 2y2 = 7
8.
y2 − 3xy = 4
x2 − 4xy + y2 = 1
9.
x2 + y2 = 1
√
x + y +
√
x − y = 2
10.
x2 + xy + y2 = 19(x − y)2
x2 − xy + y2 = 7(x − y)
11.
x
√
y + y
√
x = 30
x
√
x + y
√
y = 35
12.
x2 + y2 +
√
2xy = 8
√
2
√
x +
√
y = 4
Bài 1.100 : Giải các hệ phương trình sau :
1.
x2 + x + y + 1 + x + y2 + x + y + 1 + y = 18
x2 + x + y + 1 − x + y2 + x + y + 1 − y = 2;
2.
Ö
x
y
+
y
x
=
7
√
xy
+ 1
x
√
xy + y
√
xy = 78;
3.
x2 + y2 +
√
2xy = 8
√
2
√
x +
√
y = 4;
4.
√
x + y +
√
x − y = 4
x2 + y2 = 128;
5.
√
x +
√
y = 1
|x| + |y| = 1;
6.
√
x +
√
y = 4
√
x + 5 +
√
y + 5 = 6;
7.
x + y −
√
xy = 7
x2 + y2 + xy = 133;
8.
(x − y)(x2 − y2) = 7
(x + y)(x2 + y2) = 175;
9.
x
√
x + y
√
y = 2
√
xy
√
x +
√
y = 2;
10.
√
x + y +
√
x − y = 1 + x2 − y2
√
x +
√
y = 1;
TRẦN ANH TUẤN - 0974 396 391 - (04) 66 515 343 Trang 25
www.VNMATH.com www.VNMATH.com
WWW.VNMATH.COM
CHUYÊN ĐỀ LUYỆN THI ĐẠI HỌC
11.
Ö
x
y
+
y
x
=
5
2
x + y = 10
12.
x
√
y + y
√
x = 30
x
√
x + y
√
y = 35;
13.
2(x + y) = 3 3
x2y + 3
xy2
3
√
x + 3
√
y = 6
14.
6x
x + y
+
x + y
6x
=
5
2
x + y −
√
xy = 9;
15.
Ö
x
y
+
y
x
=
7
2 +
√
xy
x
√
xy + y
√
xy = 7;
16.
3 −
5
y + 42x
√
2y = 4
3 +
5
y + 42x
√
x = 2
17.
3 + 2x2y − x4y2 + x4(2 − 2x2) = y4
1 + 1 + (x − y)2 = x3(x3 − x + 2y2);
18.
√
x +
√
y = 10
√
x + 6 +
√
y + 6 = 14
19.
x + x2 − y2
x − x2 − y2
=
9x
5
x
y
=
5 + 3x
30 − 6y
20.
x + y + x2 − y2 = 12
y x2 − y2 = 12.
21.
Ö
20y
x
=
√
x + y +
√
x − y
16x
5y
=
√
x + y −
√
x − y
Bài 1.101 : Giải các hệ phương trình sau :
1.
2x + x2 − y2 = 3
x2 + y2 = 1
2.
x2 + y2 =
1
2
2x3 + 6y2 x = 1
3.
x3 + 3y2x = y
x2 + 3y2 = 1
4.
x − y
1 − xy
=
1 − 3x
3 − x
x + y
1 + xy
=
1 − 2y
2 − y
5.
(x + y)(1 + xy) = 18xy
(x2 + y2)(1 + x2y2) = 208x2y2
6.
(x + y) 1 +
1
xy
= 4
xy +
1
xy
+
x2 + y2
xy
= 4
7.
y(x2 + 1) = 2x(y2 + 1)
(x2 + y2) 1 +
1
x2y2
= 24
8.
(x + y) 1 +
1
xy
= 5
xy +
1
xy
= 4
9.
(x + y) 1 +
1
xy
= 6
(x2 + y2) 1 +
1
xy
2
= 18
10.
(x2 + y2) 1 +
1
xy
2
= 9
(x3 + y3) 1 +
1
xy
3
= 27
11.
x
y
+
y
x
(x + y) = 15
x2
y2
+
y2
x2
(x2 + y2) = 85
12.
2x + y +
1
x
= 4
x2 + xy +
1
x
= 3
13.
x2y + 2y + x = 4xy
1
x2
+
1
xy
+
x
y
= 3
14.
x2y + y = 2
x2 +
1
x2
+ x2y2 = 3;
15.
x2 + y2 + x + y = 4xy
1
x
+
1
y
+
y
x2
+
x
y2
= 4;
16.
2y(x2 − y2) = 3x
x(x2 + y2) = 10y.
Bài 1.102 : Giải các hệ phương trình :
1.
x2 + y2 − 3x + 4y = 1
3x2 − 2y2 − 9x − 8y = 3
TRẦN ANH TUẤN - 0974 396 391 - (04) 66 515 343 Trang 26
www.VNMATH.com www.VNMATH.com
WWW.VNMATH.COM
CHUYÊN ĐỀ LUYỆN THI ĐẠI HỌC
2.
(x + y) 2 −
1
xy
=
9
2
(x − y) 2 +
1
xy
=
5
2
3.
x − xy − y = 1
x2y − xy2 = 6
4.
x(x + 2)(2x + y) = 9
x2 + 4x + y = 6
5.
y + xy2 = 6x2
1 + x2y2 = 5x2;
6.
1 + x3y3 = 19x3
y + xy2 = −6x2.
Bài 1.103 : Cho hệ phương trình :
x + xy + y = a + 1
x2y + xy2 = a.
Tìm a để hệ có ít nhất một nghiệm (x; y) thỏa mãn : x > 0 và y > 0.
Bài 1.104 : Cho hệ phương trình :
√
x + 1 +
√
y + 1 = 3
x
√
y + 1 + y
√
x + 1 +
√
y + 1 +
√
x + 1 = m.
1. Giải hệ phương trình với m = 6.
2. Tìm m để hệ phương trình trên có nghiệm.
1.4.4 Phương pháp hàm số
Bài 1.105 : Giải các hệ phương trình sau :
1.
x3 − 5x = y3 − 5y
x8 + y4 = 1
2.
x +
√
x2 − 2x + 2 = 3y−1 + 1
y + y2 − 2y + 2 = 3x−1 + 1
3.
x2 =
√
y − 1 + 2x − 1
y2 =
√
x − 1 + 2y − 1
4.
√
x + 1 +
√
7 − y = 4
√
y + 1 +
√
7 − x = 4
5.
√
x +
√
x + 3 = 3
√
y
√
y +
√
y + 3 = 3
√
x
6.
x3 − 3x = y3 − 3y
x6 + y6 = 1
7.
ex − ey = x − y
log2
x
2
+ log√
2 4y3 = 10
8.
ln(1 + x) − ln(1 + y) = x − y
2x2 − 5xy + y2 = 0
9.
√
x +
√
2 − y =
√
2
√
y +
√
2 − x =
√
2.
Bài 1.106 : Tìm m để hệ phương trình sau có nghiệm :
√
x + 1 +
√
3 − y = m
√
y + 1 +
√
3 − x = m
Bài 1.107 : Chứng minh rằng với mọi m > 0, hệ phương trình sau có nghiệm duy nhất :
3x2y − 2y2 − m = 0
3y2 x − 2x2 − m = 0
1.4.5 Phương pháp đánh giá
Bài 1.108 : Giải các hệ phương trình sau :
TRẦN ANH TUẤN - 0974 396 391 - (04) 66 515 343 Trang 27
www.VNMATH.com www.VNMATH.com
WWW.VNMATH.COM
CHUYÊN ĐỀ LUYỆN THI ĐẠI HỌC
1.
x +
√
x +
√
y + 1 = 1
y +
√
y +
√
x + 1 = 1
2.
x +
2xy
3√
x2 − 2x + 9
= x2 + y
y +
2xy
3
y2 − 2y + 9
= y2 + x
3.
y = −x3 + 3x + 4
x = 2y3 − 6y − 2
4.
x + y +
1
x
+
1
y
= 4
x2 + y2 +
1
x2
+
1
y2
= 4
5.
x2 + 2y2 = 3
x2(y2 + 1) = 4
6.
x3 − y3 = 7
xy(x − y) = 2
7.
3
√
x + 3
√
y = 1
4
√
x + 4
√
y = 1
8.
x + 2 − y2 = 2
y +
√
2 − x2 = 2;
9.
√
x +
4√
32 − x − y2 = −3
4
√
x +
√
32 − x + 6y = 24.
1.5 Số nghiệm của phương trình, hệ phương trình
Bài toán : Chứng minh rằng phương trình f(x) = 0 có đúng k nghiệm thực phân biệt trong miền D.1
Vấn đề 1 : Chứng minh phương trình có nghiệm duy nhất
Cách 1 : Lập bảng biến thiên của hàm số y = f(x) với x ∈ D (tính đầy đủ các giá trị tại đầu và cuối mũi tên), từ đó suy ra
được số nghiệm của phương trình.
Cách 2 : Dựa vào hai định lí :
Định lí 1 : Nếu hàm số y = f(x) luôn đồng biến hoặc nghịch biến trên (a; b) thì phương trình f(x) = 0 có tối đa
một nghiệm trong khoảng (a; b).
Định lí 2 : Nếu hàm số y = f(x) liên tục trên [a; b] và f(a). f(b) < 0 thì phương trình f(x) = 0 có ít nhất một
nghiệm trong khoảng (a; b).
Bài 1.109 : Chứng minh rằng các phương trình sau có nghiệm duy nhất :
1. x5 + x4 + 2x3 + 2x2 + x + 1 = 0; 2. ex(x2 + 1) − 4 = 0.
Bài 1.110 : Chứng minh rằng phương trình : x3 +
√
x − 1 = 0 có nghiệm duy nhất.
Bài 1.111 : Chứng minh rằng phương trình xx+1 = (x + 1)x có một nghiệm dương duy nhất.
Bài 1.112 : Chứng minh rằng với mọi a > 0, hệ phương trình sau có nghiệm duy nhất :
ex − ey = ln(1 + x) − ln(1 + y)
y − x = a
Vấn đề 2 : Chứng minh phương trình có đúng hai nghiệm phân biệt
1
Nếu k = 0 tức là phương trình vô nghiệm
TRẦN ANH TUẤN - 0974 396 391 - (04) 66 515 343 Trang 28
www.VNMATH.com www.VNMATH.com
WWW.VNMATH.COM
CHUYÊN ĐỀ LUYỆN THI ĐẠI HỌC
Cách 1 : Lập bảng biến thiên của hàm số y = f(x) với x ∈ D (tính đầy đủ các giá trị tại đầu và cuối mũi tên), từ đó suy ra
được số nghiệm của phương trình.
Cách 2 : Chỉ lập bảng biến thiên nhưng không tính được hết tất cả các đầu mút (lúc này y’=0 có nghiệm duy nhất). Từ đó
suy ra được phương trình có tối đa 2 nghiệm. Kết hợp với định lí 1 ta cũng chỉ ra được phương trình có ít nhất 2
nghiệm.
Bài 1.113 : Chứng minh rằng các phương trình sau có đúng hai nghiệm thực phân biệt :
1. x4 − x2 − 2x − 1 = 0;
2. x4 − 3x3 − 1 = 0;
3. 3x4 − 4x3 − 6x2 + 12x − 20 = 0;
4. x3 − 2x −
√
x + 1 = 0.
Vấn đề 3 : Chứng minh phương trình có đúng ba nghiệm phân biệt
Cách 1 : Lập bảng biến thiên của hàm số y = f(x) với x ∈ D (tính đầy đủ các giá trị tại đầu và cuối mũi tên), từ đó suy ra
được số nghiệm của phương trình.
Cách 2 : Chỉ lập bảng biến thiên nhưng không tính được hết tất cả các đầu mút (lúc này y’=0 có đúng 2 nghiệm). Từ đó
suy ra được phương trình có tối đa 3 nghiệm. Kết hợp với định lí 1 ta cũng chỉ ra được phương trình có ít nhất 3
nghiệm.
Bài 1.114 : Chứng minh rằng các phương trình sau có đúng ba nghiệm thực phân biệt :
1. sin x −
x
2
= 0; 2. 4x(4x2 + 1) = 1.
1.6 Phương trình, bất phương trình, hệ đại số trong các kì thi tuyển sinh ĐH
Bài 1.115 (CĐ08) : Tìm giá trị của tham số m để hệ phương trình
x − my = 1
mx + y = 3
có nghiệm (x; y) thỏa mãn xy < 0.
Bài 1.116 (CĐ09) : Giải bất phương trình
√
x + 1 + 2
√
x − 2 ≤
√
5x + 1.
Bài 1.117 (CĐ10) : Giải hệ phương trình
2
√
2x + y = 3 − 2x − y
x2 − 2xy − y2 = 2
(x, y ∈ R).
Bài 1.118 (A03) : Giải hệ phương trình :
x −
1
x
= y −
1
y
2y = x3 + 1.
Bài 1.119 (A04) : Giải bất phương trình :
2(x2 − 16)
√
x − 3
+
√
x − 3 >
7 − x
√
x − 3
.
TRẦN ANH TUẤN - 0974 396 391 - (04) 66 515 343 Trang 29
www.VNMATH.com www.VNMATH.com
WWW.VNMATH.COM
CHUYÊN ĐỀ LUYỆN THI ĐẠI HỌC
Bài 1.120 (A05) : Giải bất phương trình :
√
5x − 1 −
√
x − 1 >
√
2x − 4.
Bài 1.121 (A06) : Giải hệ phương trình :
x + y −
√
xy = 3
√
x + 1 +
√
y + 1 = 4
(x, y ∈ R).
Bài 1.122 (A07) : Tìm m để phương trình sau có nghiệm thực : 3
√
x − 1 + m
√
x + 1 = 2
4√
x2 − 1.
Bài 1.123 (A08) : Giải hệ phương trình :
x2 + y + x3y + xy2 + xy = −
5
4
x4 + y2 + xy(1 + 2x) = −
5
4
(x, y ∈ R).
Bài 1.124 (A08) : Tìm các giá trị của tham số m để phương trình sau có đúng hai nghiệm thực phân biệt :
4√
2x +
√
2x + 2
4√
6 − x + 2
√
6 − x = m (m ∈ R).
Bài 1.125 (A09) : Giải phương trình 2
3
√
3x − 2 + 3
√
6 − 5x − 8 = 0.
Bài 1.126 (A10) : Giải bất phương trình
x −
√
x
1 − 2(x2 − x + 1)
≥ 1.
Bài 1.127 (A10) : Giải hệ phương trình
(4x2 + 1)x + (y − 3)
√
5 − 2y = 0
4x2 + y2 + 2
√
3 − 4x = 7
(x, y ∈ R).
Bài 1.128 (B02) : Giải hệ phương trình :
3
√
x − y =
√
x − y
x + y =
√
x + y + 2.
Bài 1.129 (B03) : Giải hệ phương trình :
3y =
y2 + 2
x2
3x =
x2 + 2
y2
.
Bài 1.130 (B04) : Xác định m để phương trình sau có nghiệm :
m
√
1 + x2 −
√
1 − x2 + 2 = 2
√
1 − x4 +
√
1 + x2 −
√
1 − x2.
Bài 1.131 (B06) : Tìm m để phương trình sau có hai nghiệm thực phân biệt :
√
x2 + mx + 2 = 2x + 1.
Bài 1.132 (B07) : Chứng minh rằng với mọi giá trị dương của tham số m, phương trình sau có hai nghiệm thực phân biệt :
x2
+ 2x − 8 = m(x − 2).
Bài 1.133 (B08) : Giải hệ phương trình :
x4 + 2x3y + x2y2 = 2x + 9
x2 + 2xy = 6x + 6
(x, y ∈ R).
Bài 1.134 (B09) : Giải hệ phương trình
xy + x + 1 = 7y
x2y2 + xy + 1 = 13y2.
Bài 1.135 (B10) : Giải phương trình
√
3x + 1 −
√
6 − x + 3x2 − 14x − 8 = 0 (x ∈ R).
Bài 1.136 (D02) : Giải bất phương trình : (x2 − 3x)
√
2x2 − 3x − 2 ≥ 0.
Bài 1.137 (D02) : Giải hệ phương trình :
23x = 5y2 − 4y
4x + 2x+1
2x + 2
= y.
Bài 1.138 (D04) : Tìm m để hệ phương trình sau có nghiệm :
√
x +
√
y = 1
x
√
x + y
√
y = 1 − 3m.
TRẦN ANH TUẤN - 0974 396 391 - (04) 66 515 343 Trang 30
www.VNMATH.com www.VNMATH.com
WWW.VNMATH.COM
CHUYÊN ĐỀ LUYỆN THI ĐẠI HỌC
Bài 1.139 (D04) : Chứng minh rằng phương trình sau có đúng một nghiệm :
x5
− x2
− 2x − 1 = 0.
Bài 1.140 (D05) : Giải phương trình : 2 x + 2 + 2
√
x + 1 −
√
x + 1 = 4.
Bài 1.141 (D06) : Giải phương trình :
√
2x − 1 + x2 − 3x + 1 = 0 (x ∈ R).
Bài 1.142 (D07) : Tìm các giá trị của tham số m để hệ phương trình sau có nghiệm thực :
x +
1
x
+ y +
1
y
= 5
x3 +
1
x3
+ y3 +
1
y3
= 15m − 10.
Bài 1.143 (D08) : Giải hệ phương trình :
xy + x + y = x2 − 2y2
x
√
2y − y
√
x − 1 = 2x − 2y
(x, y ∈ R).
Bài 1.144 (D09) : Giải hệ phương trình
x(x + y + 1) − 3 = 0
(x + y)2 −
5
x2
+ 1 = 0.
1.7 Bài tập tổng hợp
Bài 1.145 : Giải phương trình :
√
x + 4 +
√
x − 4 = 2x − 12 + 2
√
x2 − 16.
Bài 1.146 : Giải bất phương trình :
√
x + 12 ≥
√
x − 3 +
√
2x + 1.
Bài 1.147 : Giải hệ phương trình :
x2 + y2 + x + y = 4
x(x + y + 1) + y(y + 1) = 2.
Bài 1.148 : Giải hệ phương trình :
√
2x + y + 1 −
√
x + y = 1
3x + 2y = 4.
Bài 1.149 : Giải bất phương trình :
√
8x2 − 6x + 1 − 4x + 1 ≤ 0.
Bài 1.150 : Giải bất phuơng trình :
√
2x + 7 −
√
5 − x ≥
√
3x − 2.
Bài 1.151 : Tìm m để hệ phương trình sau có nghiệm :
72x+
√
x+1 − 72+
√
x+1 + 2005x ≤ 2005
x2 − (m + 2)x + 2m + 3 ≥ 0.
Bài 1.152 : Giải hệ phương trình :
(x2 + 1) + y(y + x) = 4y
(x2 + 1)(y + x − 2) = y
(x, y ∈ R).
Bài 1.153 : Giải hệ phương trình :
x3 − 8x = y3 + 2y
x2 − 3 = 3(y2 + 1)
(x, y ∈ R).
Bài 1.154 : Giải hệ phương trình :
(x − y)(x2 + y2) = 13
(x + y)(x2 − y2) = 25
(x, y ∈ R).
Bài 1.155 : Giải phương trình :
√
3x − 2 +
√
x − 1 = 4x − 9 + 2
√
3x2 − 5x + 2, x ∈ R.
Bài 1.156 : Giải hệ phương trình :
x2 − xy + y2 = 3(x − y)
x2 + xy + y2 = 7(x − y)3
(x, y ∈ R).
TRẦN ANH TUẤN - 0974 396 391 - (04) 66 515 343 Trang 31
www.VNMATH.com www.VNMATH.com
WWW.VNMATH.COM
CHUYÊN ĐỀ LUYỆN THI ĐẠI HỌC
Bài 1.157 : Giải phương trình : x + 2
√
7 − x = 2
√
x − 1 +
√
−x2 + 8x − 7 + 1, x ∈ R.
Bài 1.158 : Tìm m để phương trình :
m
√
x2 − 2x + 2 + 1 + x(2 − x) ≤ 0
có nghiệm thuộc đoạn
ä
0; 1 +
√
3
ç
.
Bài 1.159 : Giải hệ phương trình :
x4 − x3y + x2y2 = 1
x3y − x2 + xy = 1.
Bài 1.160 : Tìm m để phương trình :
4√
x2 + 1 −
√
x = m có nghiệm.
Bài 1.161 : Tìm m để phương trình :
4√
x4 − 13x + m + x − 1 = 0 có đúng một nghiệm.
Bài 1.162 : Tìm m để phương trình : x − 3 − 2
√
x − 4 + x − 6
√
x − 4 + 5 = m có đúng hai nghiệm thực.
Bài 1.163 : Tìm m để hệ phương trình :
2x − y − m = 0
x +
√
xy = 1
có nghiệm duy nhất.
Bài 1.164 : Với giá trị nào của a thì hệ có ít nhất một nghiệm thỏa mãn x, y > 0. Với các giá trị a tìm được hãy tìm tất cả
các nghiệm của hệ đã cho :
x + y +
1
x
+
1
y
= 4
x2 + y2 +
1
x2
+
1
y2
=
√
2 − a2 +
Ö
2 −
1
a2
+
a2 + 1
a
.
Bài 1.165 : Giải hệ phương trình :
y3 + y2x + 3x − 6y = 0
x2 + xy = 3.
Bài 1.166 : Cho hệ phương trình :
x2 + y2 = m
x + y = 6.
1. Giải hệ phương trình với m = 26 ;
2. Tìm m để hệ vô nghiệm ;
3. Tìm m để hệ có nghiệm duy nhất ;
4. Tìm m để hệ hai nghiệm phân biệt.
Bài 1.167 : Cho hệ phương trình :
x + xy + y = m + 2
x2y + xy2 = m + 1.
1. Giải hệ phương trình với m = −3 ; 2. Xác định m để hệ có nghiệm duy nhất.
Bài 1.168 : Cho hệ phương trình :
(x − 2)2 + y2 = m
x2 + (y − 2)2 = m.
Tìm m để hệ có nghiệm duy nhất.
Bài 1.169 : Cho hệ phương trình :
x = y2 − y + m
y = x2 − x + m.
1. Giải hệ phương trình với m = 0 ;
2. Tìm m để hệ phương trình có nghiệm ;
3. Tìm m để hệ phương trình có nghiệm duy nhất.
TRẦN ANH TUẤN - 0974 396 391 - (04) 66 515 343 Trang 32
www.VNMATH.com www.VNMATH.com
WWW.VNMATH.COM
CHUYÊN ĐỀ LUYỆN THI ĐẠI HỌC
Bài 1.170 : Tìm a để hệ sau có nghiệm duy nhất :
2|x| + |x| = y + x2 + a
x2 + y2 = 1.
Bài 1.171 : Tìm a để hệ sau có nghiệm :
x2 + 2xy − 7y2 ≥
1 − a
1 + a
3x2 + 10xy − 5y2 ≤ −2.
Bài 1.172 : Tìm m để phương trình sau có nghiệm duy nhất :
√
4 − x +
√
x + 5 = m.
Bài 1.173 : Tìm m để phương trình sau có nghiệm duy nhất :
4
√
x +
4√
1 − x +
√
x +
√
1 − x = m.
Bài 1.174 : Tìm a để hệ sau có nghiệm :
x2 − 2xy − 3y2 = 8
2x2 + 4xy + 5y2 = a4 − 4a3 + 4a2 − 12 +
√
105.
Bài 1.175 : Cho phương trình x +
√
17 − x2 + x
√
17 − x2 = m.
1. Giải phương trình khi m = 9;
2. Tìm m để phương trình có nghiệm thực;
3. Tìm m để phương trình có nghiệm thực duy nhất.
Bài 1.176 : Giải bất phương trình 2x2 − 5x − 3x
x2 − 3
x
− 6 ≥ 0.
Bài 1.177 : Chứng tỏ rằng với mọi số m không âm thì phương trình sau luôn có nghiệm thực
3x2
+ (3m2
− 7)
√
x2 + 4 − m3
+ 6 = 0.
Bài 1.178 : Giải hệ phương trình
√
x2 + 2 + y2 + 3 + x + y = 5
√
x2 + 2 +
√
2 + 3 − x − y = 2.
Bài 1.179 : Giải hệ phương trình
x2 + y3 = 2y2
x + y3 = 2y.
Bài 1.180 : Giải bất phương trình 2
√
x − 1 −
√
x + 2 > x − 2.
Bài 1.181 : Giải bất phương trình
√
3x + 7 −
√
2x + 3 >
√
x + 2.
Bài 1.182 : Giải hệ phương trình
2x2 + x + y2 = 7
xy − x + y = 3.
Bài 1.183 : Giải hệ phương trình
(x + 3)
√
2x − 1 + (y + 3)
√
2y − 1 = 2
√
(x + 3)(y + 3)
x + y = 2xy.
Bài 1.184 : Giải hệ phương trình
x +
3x − y
x2 + y2
= 3
y −
x + 3y
x2 + y2
= 0.
TRẦN ANH TUẤN - 0974 396 391 - (04) 66 515 343 Trang 33
www.VNMATH.com www.VNMATH.com
WWW.VNMATH.COM
CHUYÊN ĐỀ LUYỆN THI ĐẠI HỌC
Bài 1.185 : Giải phương trình
√
(x + 2)(2x − 1) − 3
√
x + 6 = 4 −
√
(x + 6)(2x − 1) + 3
√
x + 2.
Bài 1.186 : Giải hệ phương trình
√
x − y −
√
x + y = 2
x2 + y2 + x2 − y2 = 4.
Bài 1.187 : Giải hệ phương trình
x2 + xy + y2 = 7(x − y)2
x2 − xy + y2 = 3(x − y).
Bài 1.188 : Giải hệ phương trình
x + y + x2 − y2 = 12
y x2 − y2 = 12.
Bài 1.189 : Giải hệ phương trình
(2x + 1)2 + y2 + y = 2x + 3
xy + x = −1.
Bài 1.190 : Giải phương trình (x2 + 1)2 = 5 − x
√
2x2 + 4.
Bài 1.191 : Giải hệ phương trình
x3 − y3 + 2 = 0
x2 + y2 + x − y = 0.
Bài 1.192 : Giải phương trình |x +
√
1 − x2| =
√
2(1 − 2x2).
Bài 1.193 : Giải hệ phương trình
x2 + 6y = y + 3
√
x + y +
√
x − y = 4.
Bài 1.194 : Tìm m để phương trình sau có nghiệm thực
x3
+ x2
+ x − m(x2
+ 1)2
= 0.
Bài 1.195 : Giải bất phương trình
1
√
2x2 + 3x − 5
>
1
2x − 1
.
Bài 1.196 : Tìm m để phương trình
√
2x2 − mx + 13 = x − 2 có nghiệm thực.
Bài 1.197 : Giải hệ phương trình
2x
y
+
Ö
2y
x
= 3
x − y + xy = 3.
Bài 1.198 : Giải hệ phương trình
√
x + 1 +
√
y − 1 = 4
√
x + 6 +
√
y + 4 = 6.
Bài 1.199 : Tìm các giá trị của tham số m để hệ phương trình sau có nghiệm thực
x2 + y2 + 2(x + y) = 2
xy(x + 2)(y + 2) = 2m(2m+1 − 1).
Bài 1.200 : Chứng minh rằng với mọi m ≥ 2010 hệ phương trình sau có không quá một nghiệm thực
√
x + 27 −
√
y + 1 = (m − 2010)y + 1
√
y + 27 −
√
x + 1 = (m − 2010)x + 1.
Bài 1.201 : Giải phương trình
√
x + 1 + 1 = 4x2 +
√
3x.
Bài 1.202 : Giải hệ phương trình
x3y(1 + y) + x2y2(2 + y) + xy3 − 30 = 0
x2y + x(1 + y + y2) + y − 11 = 0.
TRẦN ANH TUẤN - 0974 396 391 - (04) 66 515 343 Trang 34
www.VNMATH.com www.VNMATH.com
WWW.VNMATH.COM
CHUYÊN ĐỀ LUYỆN THI ĐẠI HỌC
Bài 1.203 : Giải hệ phương trình
x3 + 4y = y3 + 16x
1 + y2 = 5(1 + x2).
Bài 1.204 : Giải hệ phương trình
2 + 6y =
x
y
−
√
x − 2y
x +
√
x − 2y = x + 3y − 2.
Bài 1.205 : Giải bất phương trình x
√
2 − x ≤ x2 − x − 2 −
√
2 − x.
Bài 1.206 : Giải hệ phương trình
x3 + y3 = 1
x2y + 2xy2 + y3 = 2.
Bài 1.207 : Tìm các giá trị của tham số m để bất phương trình
x(4 − x) + m
√
x2 − 4x + 5 + 2 ≤ 0
nghiệm đúng với mọi giá trị của x ∈ [2; 2 +
√
3].
Bài 1.208 : Giải hệ phương trình
2x2y + y3 = 2x4 + x6
(x + 2)
√
y + 1 = (x + 1)2.
Bài 1.209 : Giải hệ phương trình
x − 2y −
√
xy = 0
√
x − 1 +
√
4y − 1 = 2.
Bài 1.210 : Giải hệ phương trình
x
√
x − 8
√
y =
√
x + y
√
y
x − y = 5.
Bài 1.211 : Tìm m để hệ phương trình
x3 − y3 + 3y2 − 3x − 2 = 0
x2 +
√
1 − x2 − 3 2y − y2 + m = 0
có nghiệm thực.
Bài 1.212 : Giải phương trình
√
x + 3 + 2x
√
x + 1 = 2x +
√
x2 + 4x + 3.
Bài 1.213 : Xác định các giá trị của tham số m để phương trình
√
2x2 + 2mx + m + 1 = 1 − x có đúng một nghiệm thực
dương.
Bài 1.214 : Giải hệ phương trình
x2 + y2 + x2y2 = 1 + 2xy
x + x2y + xy = xy2 + y + 1.
Bài 1.215 : Giải phương trình
√
2x2 + 3x + 1 −
√
2x2 − 2 = x + 1.
Bài 1.216 : Xác định các giá trị của tham số m để phương trình
√
x − 1 − m
√
x +
6√
x3 − x2 = 0 có nghiệm thực.
Bài 1.217 : Tìm m để hệ phương trình sau có nghiệm thực
x
3 + x4
+
y
3 + y4
=
1
8
xy
9 + 3x4 + 3y4 + x4y4
= m.
Bài 1.218 : Giải phương trình
1
x
+
1
√
2 − x2
= 2.
Bài 1.219 : Giải hệ phương trình
x2 + y2 = 5
√
y − 1(x + y − 1) = (y − 2)
√
x + y.
Bài 1.220 : Giải hệ phương trình
x2 + 1 + y2 + xy = 4y
x + y − 2 =
y
x2 + 1
.
TRẦN ANH TUẤN - 0974 396 391 - (04) 66 515 343 Trang 35
www.VNMATH.com www.VNMATH.com
WWW.VNMATH.COM
CHUYÊN ĐỀ LUYỆN THI ĐẠI HỌC
Bài 1.221 : Tìm m để phương trình
√
x +
√
x + 4 − m
√
4 − x = 3m có nghiệm thực.
Bài 1.222 : Giải hệ phương trình
x3y = 24
2
√
x3 + y = 6
3√
3.
Bài 1.223 : Giải hệ phương trình
√
x − 1 +
√
y − 1 = 3
x + y −
√
(x − 1)(y − 1) = 5.
Bài 1.224 : Tìm m để phương trình m
√
x − 2 + 2
4√
x2 − 4 −
√
x + 2 = 2
4√
x2 − 4 có nghiệm.
Bài 1.225 : Giải hệ phương trình
x2 + y2 + x + y = 18
x(x + 1)y(y + 1) = 72.
Bài 1.226 : Giải hệ phương trình
√
7x + y +
√
2x + y = 5
√
2x + y + 20x + 5y = 38.
Bài 1.227 : Giải hệ phương trình
xy + x2 = 1 + y
xy + y2 = 1 + x.
Bài 1.228 : Tìm các giá trị của tham số m để phương trình m +
2
3
√
x − x2 =
√
x +
√
1 − x có nghiệm.
Bài 1.229 : Giải bất phương trình 5
√
x +
5
2
√
x
≤ 2x +
1
2x
+ 5.
TRẦN ANH TUẤN - 0974 396 391 - (04) 66 515 343 Trang 36
www.VNMATH.com www.VNMATH.com
WWW.VNMATH.COM
WWW.VNMATH.COM
Chương 2
Bất đẳng thức
2.1 Phương pháp sử dụng bất đẳng thức Cauchy
2.1.1 Bất đẳng thức Cauchy - So sánh giữa tổng và tích
Cho ba số không âm a, b, c, ta có :
1.
a + b
2
≥
√
ab, dấu bằng xảy ra khi a = b ;
2.
a + b + c
3
≥
3
√
abc, dấu bằng xảy ra khi a = b = c.
2.1.2 Một số hệ quả trực tiếp
Hệ quả 1 : So sánh giữa tổng nghịch đảo và tổng.
Cho ba số dương a, b, c có :
1.
1
a
+
1
b
≥
4
a + b
; 2.
1
a
+
1
b
+
1
c
≥
9
a + b + c
.
Hệ quả 2 : So sánh giữa tổng bình phương và tồng.
Cho ba số thực a, b, c có :
1. 2(a2 + b2) ≥ (a + b)2 ; 2. 3(a2 + b2 + c2) ≥ (a + b + c).
Hệ quả 3 : So sánh giữa tổng, tổng bình phương và tích.
Cho ba số thực a, b, c có :
1. (a + b + c)2 ≥ 3(ab + bc + ca) ; 2. a2 + b2 + c2 ≥ ab + bc + ca.
2.1.3 Bài tập đề nghị
Bài 2.1 : Cho a, b, > 0. Chứng minh rằng :
ab(a + b)
2
≤
a + b
2
3
≤
(a + b)(a2 + ab + b2)
6
≤
a3 + b3
2
≤
(a2 + b2)3
(a + b)3
.
Bài 2.2 : Cho a, b > 0 và a + b ≤ 1. Chứng minh rằng :
37
WWW.VNMATH.COM
CHUYÊN ĐỀ LUYỆN THI ĐẠI HỌC
1.
1
a
+
1
b
≥ 4 ; 2.
1
a
+
1
b
+ a + b ≥ 5.
Bài 2.3 : Cho các số không âm a, b, c có a + b + c ≤ 3. Chứng minh rằng :
1. a + b + c ≥ ab + bc + ca ; 2.
√
a +
√
b +
√
c ≥ ab + bc + ca.
Bài 2.4 : Cho x, y > 0. Chứng minh rằng : (1 + x)(1 + y) ≥ (1 +
√
xy)2.
Bài 2.5 : Cho x, y > 0. Chứng minh rằng : x2 + y2 +
1
x
+
1
y
≥ 2(
√
x +
√
y).
Bài 2.6 : Cho x, y > 0 và x + y = 1. Tìm giá trị nhỏ nhất của P =
1
x2 + y2
+
1
xy
.
Bài 2.7 : Cho x, y, z > 0 và x + y + z = 1. Tìm giá trị lớn nhất của P =
x
x + 1
+
y
y + 1
+
z
z + 1
.
Bài 2.8 : Cho a, b > 0 và a + b = 1. Chứng minh rằng :
a2
a + 1
+
b2
b + 1
≥
1
3
.
Bài 2.9 : Cho các số thực dương a, b, c. Chứng minh rằng :
1
a + 3b
+
1
b + 3c
+
1
c + 3a
≥
1
2a + b + c
+
1
2b + c + a
+
1
2c + a + b
.
Bài 2.10 : Chứng minh rằng với mọi a, b, c > 0 đều có :
1.
1
a(b + c)
+
1
b(c + a)
+
1
c(a + b)
≥
27
2(a + b + c)2
; 2.
1
a(a + b)
+
1
b(b + c)
+
1
c(c + a)
≥
27
2(a + b + c)2
.
Bài 2.11 : Cho a, b > 0 và a + b ≤ 1. Tìm giá trị nhỏ nhất của S = ab +
1
ab
.
Bài 2.12 : Cho a, b > 0. Tìm giá trị nhỏ nhất của biểu thức S =
a + b
√
ab
+
√
ab
a + b
.
Bài 2.13 : Cho a, b, c > 0 và a + b + c ≤
3
2
. Tìm giá trị nhỏ nhất của biểu thức S = a + b + c +
1
a
+
1
b
+
1
c
.
Bài 2.14 : Chứng minh rằng với mọi số dương x, y, z đều có : x2 + y2 + z2 ≥
√
2(xy + yz).
Bài 2.15 : Cho a, b, c > 0 và a + b + c = 4. Chứng minh rằng :
ab
a + b + 2c
+
bc
b + c + 2a
+
ca
c + a + 2b
≤ 1.
Bài 2.16 : Cho a, b, c > 0. Chứng minh rằng :
ab
a + 3b + 2c
+
bc
b + 3c + 2a
+
ca
c + 3a + 2b
≤
a + b + c
6
.
Bài 2.17 : Cho a, b, c > 0. Chứng minh rằng :
1.
a + b
c
+
b + c
a
+
c + a
b
≥ 6 ;
2.
a
b + c
+
b
c + a
+
c
a + b
≥
3
2
;
3.
a2
b + c
+
b2
c + a
+
c2
a + b
≥
a + b + c
2
;
4.
a3
b + c
+
b3
c + a
+
c3
a + b
≥
a2 + b2 + c2
2
.
Bài 2.18 : Cho a, b, c > 0 và abc = 1. Tìm giá trị nhỏ nhất của các biểu thức sau :
1. P =
a2
b + c
+
b2
c + a
+
c2
a + b
;
2. Q =
a3
b + c
+
b3
c + a
+
c3
a + b
;
3. R =
a2 √
a
b + c
+
b2
√
b
c + a
+
c2 √
c
a + b
;
4. S =
bc
a2b + a2c
+
ca
b2c + b2a
+
ab
c2a + c2b
;
TRẦN ANH TUẤN - 0974 396 391 - (04) 66 515 343 Trang 38
www.VNMATH.com www.VNMATH.com
WWW.VNMATH.COM
CHUYÊN ĐỀ LUYỆN THI ĐẠI HỌC
Bài 2.19 : Cho x, y, z, t > 0 và xyzt = 1. Tìm giá trị nhỏ nhất của biểu thức :
P =
1
x3(yz + zt + ty)
+
1
y3(zt + tx + xz)
+
1
z3(tx + xy + yt)
+
1
t3(xy + yz + zx)
.
Bài 2.20 : Cho a, b, c > 0. Tìm giá trị nhỏ nhất của biểu thức sau :
1. P =
a
b + 2c
+
b
c + 2a
+
c
a + 2b
. 2. Q =
a
b + mc
+
b
c + ma
+
c
a + mb
, m ∈ N, m > 2.1
Bài 2.21 : Cho a, b, c > 0. Chứng minh rằng :
1. (a + b)(b + c)(c + a) ≥ 8abc ; 2.
bc
a
+
ca
b
+
ba
c
≥ a + b + c.
Bài 2.22 : Cho a, b, c là độ dài ba cạnh của một tam giác. Chứng minh rằng :
1.
a
b + c − a
+
b
c + a − b
+
c
a + b − c
≥ 3 ; 2.
a2
b + c − a
+
b2
c + a − b
+
c2
a + b − c
≥ a + b + c.
Bài 2.23 : 1. Cho a, b, c là độ dài ba cạnh của một tam giác, p là nửa chu vi của tam giác. Chứng minh rằng :
(p − a)(p − b)(p − c) ≤
abc
8
.
2. Cho tam giác ABC có chu vi bằng 3 và độ dài ba cạnh của tam giác là a, b, c. Chứng minh rằng :
4(a3
+ b3
+ c3
) + 15abc ≥ 27.
Bài 2.24 : Cho a, b, c, d > 0 và a + b + c + d = 1. Chứng minh rằng :
1
a
− 1
1
b
− 1
1
c
− 1
1
d
− 1 ≥ 81.
Bài 2.25 : Cho a, b ≥ 1. Chứng minh rằng : a
√
b − 1 + b
√
a − 1 ≤ ab.
Bài 2.26 : Cho a, b, c ≥ 0 và a + b + c = 1. Chứng minh rằng : ab + bc + ca + abc ≤
10
27
.
Bài 2.27 : Cho a, b, c > 0. Chứng minh rằng :
2
a2 + bc
≤
1
2
1
ab
+
1
ac
.
Bài 2.28 : Cho a, b > 0 và a + b = 1. Chứng minh rằng :
3
ab
+
2
a2 + b2
≥ 16.
Bài 2.29 : Cho a, b, c > 0 và
1
1 + a
+
1
1 + b
+
1
1 + c
≥ 2. Chứng minh rằng : abc ≤
1
8
.
Bài 2.30 : Cho a > b > 0 và ab = 1. Chứng minh rằng :
a2 + b2
a − b
≥ 2
√
2.
Bài 2.31 : Tìm giá trị nhỏ nhất của A = (1 + x) 1 +
1
y
+ (1 + y) 1 +
1
x
với x, y > 0 thỏa mãn x2 + y2 = 1.
Bài 2.32 : Cho x, y, z > 1 thỏa mãn x + y + z = xyz. Tìm giá trị nhỏ nhất của :
P =
y − 2
x2
+
z − 2
y2
+
x − 2
z2
.
Bài 2.33 : Cho a, b, c > 1. Chứng minh rằng :
alogb c
+ blogc a
+ cloga b
≥ 3
3√
abc.
1
Một cách tổng quát, tìm giá trị nhỏ nhất của R =
a
xb + yc
+
b
xc + ya
+
c
xa + yb
với a, b, c, x, y là những số dương
TRẦN ANH TUẤN - 0974 396 391 - (04) 66 515 343 Trang 39
www.VNMATH.com www.VNMATH.com
WWW.VNMATH.COM
CHUYÊN ĐỀ LUYỆN THI ĐẠI HỌC
Bài 2.34 : Cho a, b, c > 0 và a + b + c = 1. Chứng minh rằng :
1 +
1
a
1 +
1
b
1 +
1
c
≥ 64.
Bài 2.35 : Cho a, b > 0. Chứng minh rằng : (a + b)2 +
1
a
+
1
b
2
≥ 8.
Bài 2.36 : Cho a, b, c > 0. Chứng minh rằng :
bc
a2b + a2c
+
ca
b2c + b2a
+
ab
c2a + c2b
≥
1
2
1
a
+
1
b
+
1
c
.
Bài 2.37 : Cho a, b, c > 0. Chứng minh rằng :
ab
a + b
+
bc
b + c
+
ca
c + a
≤
a + b + c
2
.
Bài 2.38 : Cho a ≥ 3. Tìm giá trị nhỏ nhất của biểu thức S = a +
1
a
.
Bài 2.39 : Cho a ≥ 2. Tìm giá trị nhỏ nhất của biểu thức S = a +
1
a2
.
Bài 2.40 : Cho a, b, c ≥ 0 thỏa mãn a2 + b2 + c2 = 1. Tìm giá trị nhỏ nhất của biểu thức
S = a + b + c +
1
abc
.
Bài 2.41 : Cho x, y > 0 và x + y = 1. Tìm giá trị nhỏ nhất của biểu thức S =
x
√
1 − x
+
y
√
1 − y
.
Bài 2.42 : Cho a, b, c ≥ 0 và a + b + c = 1. Tìm giá trị lớn nhất của biểu thức
S =
3√
a + b +
3√
b + c +
3
√
c + a.
Bài 2.43 : Cho a, b, c > 0 và a + b + c = 3. Tìm giá trị lớn nhất của biểu thức
S = 3
a(b + 2c) + 3
b(c + 2a) + 3
c(a + 2b).
Bài 2.44 : Cho a ≥ 2; b ≥ 6; c ≥ 12. Tìm giá trị lớn nhất của biểu thức
S =
bc
√
a − 2 + ca
3√
b − 6 + ab
4
√
c − 12
abc
.
Bài 2.45 : Chứng minh rằng :
a
b
+
b
c
+
c
a
2
≥
3
2
a + b
c
+
b + c
a
+
c + a
b
với mọi a, b, c > 0.
Bài 2.46 : Cho a, b, c > 0 và a + b + c = 3. Chứng minh rằng :
a3
(a + b)(a + c)
+
b3
(b + c)(b + a)
+
c3
(c + a)(c + b)
≥
3
4
.
Bài 2.47 : Cho a, b, c > 0 và a + b + c = 3. Chứng minh rằng :
a3
b(2c + a)
+
b3
c(2a + b)
+
c3
c(2b + c)
≥ 1.
Bài 2.48 : Cho a, b, c > 0 và a2 + b2 + c2 = 1. Chứng minh rằng :
a3
b + 2c
+
b3
c + 2a
+
c3
a + 2b
≥
1
3
.
Bài 2.49 : Cho a, b, c > 0 và a2 + b2 + c2 = 1. Chứng minh rằng :
a3
a + b
+
b3
b + c
+
c3
c + a
≥
1
2
.
TRẦN ANH TUẤN - 0974 396 391 - (04) 66 515 343 Trang 40
www.VNMATH.com www.VNMATH.com
WWW.VNMATH.COM
CHUYÊN ĐỀ LUYỆN THI ĐẠI HỌC
Bài 2.50 : Cho a, b, c > 0 và ab + bc + ca = 1. Chứng minh rằng :
a
√
1 + a2
+
b
√
1 + b2
+
c
√
1 + c2
≤
3
2
.
Bài 2.51 : Cho a, b, c > 0 và ab + bc + ca = 1. Chứng minh rằng :
1
a(a + b)
+
1
b(b + c)
+
1
c(c + a)
≥
9
2
.
Bài 2.52 : Cho a, b, c > 0 và a + b + c = 1. Chứng minh rằng :
a
(b + c)2
+
b
(c + a)2
+
c
(a + b)2
≥
9
4
.
Bài 2.53 : Cho a, b, c > 0 và a2 + b2 + c2 = 3. Chứng minh rằng :
ab
c
+
bc
a
+
ca
b
≥ 3.
Bài 2.54 : Cho a, b, c > 0 và a + b + c = 1. Chứng minh rằng :
bc
√
a + bc
+
ca
√
b + ca
+
ab
√
c + ab
≤
1
2
.
Bài 2.55 : Cho a, b, c > 0 và a + b + c = 2. Chứng minh rằng :
bc
√
2a + bc
+
ca
√
2b + ca
+
ab
√
2c + ab
≤ 1.
Bài 2.56 : Cho a, b, c > 0 và abc = 1. Chứng minh rằng :
a3
(1 + b)(1 + c)
+
b3
(1 + c)(1 + a)
+
c3
(1 + a)(1 + b)
≥
3
4
.
Bài 2.57 : Cho a, b, c > 0 và abc = 1. Chứng minh rằng :
1
a3(b + c)
+
1
b3(c + a)
+
1
c3(a + b)
≥
3
2
.
Bài 2.58 : Cho a, b, c > 0. Chứng minh rằng :
1
a
+
1
b
+
1
c
≥ 2
1
a + b
+
1
b + c
+
1
c + a
.
Bài 2.59 : Cho a, b, c > 0 và a + b + c ≤ 1. Chứng minh rằng :
1
a2 + 2bc
+
1
b2 + 2ca
+
1
c2 + 2ab
≥ 9.
Bài 2.60 : Cho a, b > 0 và a + b ≤ 1. Chứng minh rằng :
1
a2 + b2
+
1
ab
≥ 6.
Bài 2.61 : Cho a, b > 0 và a + b ≤ 1. Chứng minh rằng :
1
a2 + b2
+
1
ab
+ 4ab ≥ 7.
Bài 2.62 : Cho a, b, c > 0 và ab + bc + ca = abc. Chứng minh rằng :
1
a + 2b + 3c
+
1
b + 2c + 3a
+
1
c + 2a + 3b
<
3
16
.
Bài 2.63 : Tìm giá trị nhỏ nhất của : A =
a
1 + b − a
+
b
1 + c − b
+
c
1 + a − c
với a, b, c > 0 và a + b + c = 1.
Bài 2.64 : Cho x, y, z > 0 và x2 + y2 + z2 = 1. Tìm giá trị nhỏ nhất của biểu thức :
P =
x
y2 + z2
+
y
z2 + x2
+
z
x2 + y2
.
Bài 2.65 : Cho x, y là hai số thực thay đổi. Tìm giá trị lớn nhất và giá trị nhỏ nhất của biểu thức :
P =
(x + y)(1 − xy)
(1 + x2)2(1 + y2)2
.
TRẦN ANH TUẤN - 0974 396 391 - (04) 66 515 343 Trang 41
www.VNMATH.com www.VNMATH.com
WWW.VNMATH.COM
CHUYÊN ĐỀ LUYỆN THI ĐẠI HỌC
Bài 2.66 : Cho x, y, z là ba số thực thỏa mãn x + y + z = 0. Tìm giá trị nhỏ nhất của
P =
√
2x + 3 +
√
2y + 3 +
√
2z + 3.
Bài 2.67 : Cho các số thực x, y, z thỏa mãn x + y + z = 6. Chứng minh rằng : 8x + 8y + 8z ≥ 4x+1 + 4y+1 + 4z+1.
Bài 2.68 : Cho 0 < a ≤ b ≤ c ≤ d ≤ e và a + b + c + d + e = 1. Chứng minh rằng :
a(bc + be + cd + de) + cd(b + e − a) ≤
1
25
.
Bài 2.69 : Cho a, b, c là ba số dương thỏa mãn điều kiện ab + bc + ca = abc. Chứng minh rằng :
a2
a + bc
+
b2
b + ca
+
c2
c + ab
≥
a + b + c
4
.
Bài 2.70 : Cho a, b, c là các số thực dương, chứng minh rằng :
b + c
a + 3
4(b3 + c3)
+
c + a
b + 3
4(c3 + a3)
+
a + b
c + 3
4(a3 + b3)
≤ 2.
Bài 2.71 : Cho a, b, c là các số thực dương, chứng minh rằng :
1
a3 + b3 + abc
+
1
b3 + c3 + abc
+
1
c3 + a3 + abc
≤
1
abc
.
Bài 2.72 : Cho a, b, c là các số thực dương thỏa mãn abc = 1. Chứng minh rằng :
a3 + b3
a2 + ab + b2
+
b3 + c3
b2 + bc + c2
+
c3 + a3
c2 + ca + a2
≥ 2.
Bài 2.73 : Cho ba số thực dương a, b, c. Chứng minh rằng :
2
√
a
a3 + b2
+
2
√
b
b3 + c2
+
2
√
c
c3 + a2
≤
1
a2
+
1
b2
+
1
c2
.
Bài 2.74 : Cho a, b, c > 0. Chứng minh rằng :
1
a2 + bc
+
1
b2 + ca
+
1
c2 + ab
≤
a + b + c
2abc
.
Bài 2.75 : Cho a, b, c là ba số dương sao cho ab + bc + ca ≥ 1. Chứng minh rằng :
a3
b2 + 1
+
b3
c2 + 1
+
c3
a2 + 1
≥
√
3
4
.
2.2 Bất đẳng thức hình học
Bài 2.76 : Cho a, b, c ∈ R. Chứng minh rằng :
√
a2 + b2 + 4c2 + 4ac +
√
a2 + b2 + 4c2 − 4ac ≥ 2
√
a2 + b2.
Bài 2.77 : Với mọi a, b, c, d ∈ R. Chứng minh rằng :
√
a2 + b2 + c2 + d2 + 2ac + 2bd ≤
√
a2 + b2 +
√
c2 + d2.
Bài 2.78 : Cho x, y, z > 0. Chứng minh rằng :
√
x + 2
√
y + 3
√
z ≤ 14(x + y + z).
TRẦN ANH TUẤN - 0974 396 391 - (04) 66 515 343 Trang 42
www.VNMATH.com www.VNMATH.com
WWW.VNMATH.COM
CHUYÊN ĐỀ LUYỆN THI ĐẠI HỌC
Bài 2.79 : Cho bốn số a, b, c, d ∈ R thỏa mãn a2 + b2 = 1 và c + d = 3. Chứng minh rằng :
ac + bd + cd ≤
9 + 6
√
2
4
.
Bài 2.80 : Với mọi a, b, c ∈ R. Chứng minh rằng :
√
a2 + ab + b2 +
√
a2 + ac + c2 ≥
√
b2 + bc + c2.
Bài 2.81 : Với mọi x, y ∈ R. Chứng minh rằng :
4 cos2 x cos2 y + sin2
(x − y) + 4 sin2
x sin2
y + sin2
(x − y) ≥ 2.
Bài 2.82 : Với mọi x, y ∈ R. Chứng minh rằng :
4x2 + y2 + 12x + 9 + 4x2 + y2 − 4x − 6y + 10 ≥ 5.
Bài 2.83 : Cho a + b + c = 1, ax + by + cz = 4 với a, b, c 0. Chứng minh rằng :
√
9a2 + a2x2 + 9b2 + b2y2 + 9c2 + c2z2 ≥ 5.
Bài 2.84 : Cho a, b, c > 0. Chứng minh rằng :
a2 − ab
√
2 + b2 + b2 − bc
√
3 + c2 ≥
Õ
a2 − ac 2 −
√
3 + c2.
Bài 2.85 : Cho a, b, c > 0 và abc + bc + ca = abc. Chứng minh rằng :
√
b2 + 2a2
ab
+
√
c2 + 2b2
bc
+
√
a2 + 2c2
ac
≥
√
3.
Bài 2.86 : Cho x2 + y2 = 1. Chứng minh rằng : x2
√
5 + 2xy − y2
√
5 ≤
√
6.
Bài 2.87 : Cho
x2 + xy + y2 = 3
y2 + yz + z2 = 16
và x, y, z là các số thực dương. Chứng minh rằng : xy + yz + zx ≤ 8.
Bài 2.88 : Cho x, y, z là những số dương. Chứng minh rằng :
x2 + xy + y2 + y2 + yz + z2 + z2 + zx + x2 ≥
√
3(x + y + z).
Bài 2.89 : Cho a + b + c = 12. Chứng minh rằng :
3a + 2
√
a + 1 + 3b + 2
√
b + 1 + 3c + 2
√
c + 1 ≥ 3
√
17.
Bài 2.90 : Cho các số dương x, y, z và x + y + z ≤ 2. Chứng minh rằng :
4x2 +
1
x2
+ 4y2 +
1
y2
+ 4z2 +
1
z2
≥
√
145
2
.
Bài 2.91 : Giả sử x, y, u, v ∈ R thỏa mãn : x2 + y2 = 1; u2 + v2 + 16 = 8u + 4v. Tìm giá trị lớn nhất của biểu thức
P = 8u + 4v − 2(ux + vy).
Bài 2.92 : Cho x, y, z là các số dương thỏa mãn xy + yz + zx = 5. Tìm giá trị nhỏ nhất của P = 3x2 + 3y2 + z2.
TRẦN ANH TUẤN - 0974 396 391 - (04) 66 515 343 Trang 43
www.VNMATH.com www.VNMATH.com
WWW.VNMATH.COM
CHUYÊN ĐỀ LUYỆN THI ĐẠI HỌC
2.3 Phương pháp sử dụng điều kiện có nghiệm của phương trình hoặc hệ phương trình
- phương pháp miền giá trị
Bài 2.93 : Tìm giá trị lớn nhất và nhỏ nhất của hàm số : f(x) =
2x2 + 7x + 23
x2 + 2x + 10
.
Bài 2.94 : Tìm giá trị lớn nhất và giá trị nhỏ nhất của biểu thức P =
x2 − (x − 4y)2
x2 + 4y2
, với x2 + y2 > 0.
Bài 2.95 : Cho x là số dương, y là số thực tùy ý. Tìm giá trị lớn nhất, giá trị nhỏ nhất (nếu có) của biểu thức :
P =
xy2
(x2 + 3y2) x + x2 + 12y2
.
Bài 2.96 : Tìm giá trị nhỏ nhất của biểu thức P = x2 + y2, với 2x2 + y2 + xy ≥ 1.
Bài 2.97 : Cho các số thực x, y thỏa mãn điều kiện : 3
√
x( 3
√
x − 1) + 3
√
y( 3
√
y − 1) = 3
√
xy. Tìm giá trị lớn nhất, nhỏ nhất của
biểu thức : P = 3
√
x + 3
√
y + 3
√
xy.
Bài 2.98 : Cho x, y thỏa mãn điều kiện : x2 − xy+y2 = 3. Tìm giá trị lớn nhất và nhỏ nhất của biểu thức : P = x2 + xy−2y2.
Bài 2.99 : Cho hai số thực x, y thỏa mãn điều kiện : x − 3
√
x + 1 = 3
√
y + 2 − y. Tìm giá trị lớn nhất, nhỏ nhất của biểu
thức P = x + y.
Bài 2.100 : Cho hai số thực x, y thỏa mãn : x2 + y2 = 2(x + y) + 7. Tìm giá trị lớn nhất, giá trị nhỏ nhất của biểu thức
P = 3
√
x(x − 2) + 3
√
y(y − 2).
Bài 2.101 : Cho các số thực x, y thỏa mãn : 4x2 − 3xy + 3y2 = 6. Tìm giá trị lớn nhất, giá trị nhỏ nhất của biểu thức
P = x2 + xy − 2y2.
Bài 2.102 : Cho các số thực x, y thỏa mãn :
√
x +
√
y = 4. Tìm giá trị lớn nhất, giá trị nhỏ nhất của biểu thức P =
√
x + 1 +
√
y + 9.
Bài 2.103 : Cho các số thực x, y thỏa mãn : xy + x + y = 3. Tìm giá trị lớn nhất, giá trị nhỏ nhất của biểu thức P =
3x
y + 1
+
3y
x + 1
− x2 − y2.
Bài 2.104 : Cho a, b ≥ 0 và a2 + b2 + ab = 3. Tìm giá trị nhỏ nhất và giá trị lớn nhất của biểu thức
P = a4
+ b4
+ 2ab − a5
b5
.
Bài 2.105 : Cho các số thực x, y thỏa mãn x + y = 2. Tìm giá trị lớn nhất của P = (x3 + 2)(y3 + 2).
2.4 Bất đẳng thức trong các kì thi tuyển sinh ĐH
Bài 2.106 (CĐ08) : Cho hai số thực x, y thay đổi và thoả mãn x2 + y2 = 2. Tìm giá trị lớn nhất và giá trị nhỏ nhất của biểu
thức: P = 2(x3 + y3) − 3xy.
Bài 2.107 (CĐ10) : Cho hai số thực dương thay đổi x, y thỏa mãn điều kiện 3x + y ≤ 1. Tìm giá trị nhỏ nhất của biểu thức
A =
1
x
+
1
√
xy
.
Bài 2.108 (A03) : Cho x, y, z là ba số dương và x + y + z ≤ 1. Chứng minh rằng :
x2 +
1
x2
+ y2 +
1
y2
+ z2 +
1
z2
≥
√
82.
TRẦN ANH TUẤN - 0974 396 391 - (04) 66 515 343 Trang 44
www.VNMATH.com www.VNMATH.com
WWW.VNMATH.COM
CHUYÊN ĐỀ LUYỆN THI ĐẠI HỌC
Bài 2.109 (A05) : Cho x, y, z là các số dương thoả mãn :
1
x
+
1
y
+
1
z
= 4. Chứng minh rằng :
1
2x + y + z
+
1
x + 2y + z
+
1
x + y + 2z
≤ 1.
Bài 2.110 (A06) : Cho hai số thực x 0, y 0 thay đổi và thoả mãn điều kiện : (x + y)xy = x2 + y2 − xy. Tim giá trị lớn
nhất của biểu thức A =
1
x3
+
1
y3
.
Bài 2.111 (A07) : Cho x, y, z là các số thực dương thay đổi và thoả mãn điều kiện xyz = 1. Tìm giá trị nhỏ nhất của biểu
thức :
P =
x2(y + z)
y
√
y + 2z
√
z
+
y2(z + x)
z
√
z + 2x
√
x
+
z2(x + y)
x
√
x + 2y
√
y
.
Bài 2.112 (A09) : Chứng minh rằng với mọi số thực dương x, y, z thỏa mãn x(x + y + z) = 3yz ta có :
(x + y)3
+ (x + z)3
+ 3(x + y)(y + z)(z + x) ≤ 5(y + z)3
.
Bài 2.113 (B05) : Chứng minh rằng với mọi x ∈ R, ta có :
12
5
x
+
15
4
x
+
20
3
x
≥ 33 + 4x + 5x.
Khi nào đẳng thức xảy ra.
Bài 2.114 (B06) : Cho x, y là các số thực thay đổi. Tìm giá trị nhỏ nhất của biểu thức :
A = (x − 1)2 + y2 + (x + 1)2 + y2 + |y − 2|.
Bài 2.115 (B07) : Cho x, y, z là ba số thực dương thay đổi. Tìm giá trị nhỏ nhất của biểu thức :
P = x
x
2
+
1
yz
+ y
y
2
+
1
xz
+ z
z
2
+
1
xy
.
Bài 2.116 (B08) : Cho hai số thực x, y thay đổi và thoả mãn hệ thức x2 + y2 = 1. Tìm giá trị lớn nhất và giá trị nhỏ nhất
của biểu thức P =
2(x2 + 6xy)
1 + 2xy + 2y2
.
Bài 2.117 (B09) : Cho các số thực x, y thay đổi thỏa mãn (x + y)3 + 4xy ≥ 2. Tìm giá trị nhỏ nhất của biểu thức :
A = 3(x4
+ y4
+ x2
+ y2
) − 2(x2
+ y2
) + 1.
Bài 2.118 (B10) : Cho các số thực không âm a, b, c thỏa mãn a + b + c = 1. Tìm giá trị nhỏ nhất của biểu thức M =
3(a2b2 + b2c2 + c2a2) + 3(ab + bc + ca) + 2
√
a2 + b2 + c2.
Bài 2.119 (D05) : Cho các số dương x, y, z thoả mãn xyz = 1. Chứng minh rằng :
1 + x3 + y3
xy
+
1 + y3 + z3
yz
+
√
1 + z3 + x3
zx
≥ 3
√
3.
Bài 2.120 (D07) : Cho a ≥ b > 0. Chứng minh rằng : 2a +
1
2a
b
≤ 2b +
1
2b
a
.
Bài 2.121 (D08) : Cho x, y là hai số thực không âm thay đổi. Tìm giá trị lớn nhất và giá trị nhỏ nhất của biểu thức :
P =
(x − y)(1 − xy)
(1 + x)2(1 + y)2
.
Bài 2.122 (D09) : Cho các số thực không âm x, y thay đổi và thỏa mãn x + y = 1. Tìm giá trị lớn nhất và giá trị nhỏ nhất
của biểu thức :
S = (4x2
+ 3y)(4y2
+ 3x) + 25xy.
Bài 2.123 (D10) : Tìm giá trị nhỏ nhất của hàm số y =
√
−x2 + 4x + 21 −
√
−x2 + 3x + 10.
TRẦN ANH TUẤN - 0974 396 391 - (04) 66 515 343 Trang 45
www.VNMATH.com www.VNMATH.com
WWW.VNMATH.COM
CHUYÊN ĐỀ LUYỆN THI ĐẠI HỌC
2.5 Bài tập tổng hợp
Bài 2.124 : Giả sử x, y là hai số dương thay đổi thoả mãn điều kiện x + y =
5
4
. Tìm giá trị nhỏ nhất của biểu thức :
S =
4
x
+
1
4y
.
Bài 2.125 : Giả sử a, b, c, d là bốn số nguyên thay đổi thoả mãn 1 ≤ a < b < c < d ≤ 50. Chứng minh
a
b
+
c
d
≥
b2 + b + 50
50b
và tìm giá trị nhỏ nhất của biểu thức : S =
a
b
+
c
d
.
Bài 2.126 : Cho x, y, z là ba số thoả mãn x + y + z = 0. Chứng minh rằng :
√
3 + 4x +
√
3 + 4y +
√
3 + 4z ≥ 6.
Bài 2.127 : Chứng minh rằng với mọi x, y > 0 ta có :
(1 + x) 1 +
y
x
1 +
9
√
y
2
≥ 256.
Đẳng thức xảy ra khi nào.
Bài 2.128 : Cho a, b, c là ba số dương thoả mãn a + b + c =
3
4
. Chứng minh rằng :
3√
a + 3b +
3√
b + 3c +
3√
c + 3a ≤ 3.
Khi nào đẳng thức xảy ra?
Bài 2.129 : Chứng minh rằng 0 ≤ y ≤ x ≤ 1 thì x
√
y − y
√
x ≤
1
4
. Đẳng thức xảy ra khi nào ?
Bài 2.130 : Cho x, y, z là ba số dương và xyz = 1. Chứng minh rằng :
x2
1 + y
+
y2
1 + z
+
z2
1 + x
≥
3
2
.
Bài 2.131 : Cho x, y là các số thực thoả mãn điều kiện x2 + xy + y2 ≤ 3. Chứng minh rằng :
−4
√
3 − 3 ≤ x2
− xy − 3y2
≤ 4
√
3 − 3.
Bài 2.132 : Cho các số thực x, y, z thoả mãn điều kiện 3−x + 3−y + 3−z = 1. Chứng minh rằng :
9x
3x + 3y+z
+
9y
3y + 3z+x
+
9z
3z + 3x+y
≥
3x + 3y + 3z
4
.
Bài 2.133 : Cho hai số dương x, y thay đổi và thoả mãn điều kiện x + y ≥ 4. Tìm giá trị nhỏ nhất của biểu thức A =
3x2 + 4
4x
+
2 + y3
y2
.
Bài 2.134 : Tìm giá trị nhỏ nhất của hàm số : y = x +
11
2x
+
Ö
4 1 +
7
x2
, x > 0.
Bài 2.135 : Cho x, y, z là các số thực dương. Tìm giá trị nhỏ nhất của biểu thức :
P =
3
4(x3 + y3) +
3
4(y3 + z3) +
3
4(z3 + x3) + 2
x
y2
+
y
z2
+
z
x2
.
Bài 2.136 : Cho a, b là các số dương thoả mãn ab + a + b = 3. Chứng minh rằng :
3a
b + 1
+
3b
a + 1
+
ab
a + b
≤ a2
+ b2
+
3
2
.
Bài 2.137 : Cho x, y > 0 và xy = 100. Hãy xác định giá trị nhỏ nhất của biểu thức P =
x2 + y2
x − y
.
TRẦN ANH TUẤN - 0974 396 391 - (04) 66 515 343 Trang 46
www.VNMATH.com www.VNMATH.com
Chuyen de-luyen-thi-dh-2012
Chuyen de-luyen-thi-dh-2012
Chuyen de-luyen-thi-dh-2012
Chuyen de-luyen-thi-dh-2012
Chuyen de-luyen-thi-dh-2012
Chuyen de-luyen-thi-dh-2012
Chuyen de-luyen-thi-dh-2012
Chuyen de-luyen-thi-dh-2012
Chuyen de-luyen-thi-dh-2012
Chuyen de-luyen-thi-dh-2012
Chuyen de-luyen-thi-dh-2012
Chuyen de-luyen-thi-dh-2012
Chuyen de-luyen-thi-dh-2012
Chuyen de-luyen-thi-dh-2012
Chuyen de-luyen-thi-dh-2012
Chuyen de-luyen-thi-dh-2012
Chuyen de-luyen-thi-dh-2012
Chuyen de-luyen-thi-dh-2012
Chuyen de-luyen-thi-dh-2012
Chuyen de-luyen-thi-dh-2012
Chuyen de-luyen-thi-dh-2012
Chuyen de-luyen-thi-dh-2012
Chuyen de-luyen-thi-dh-2012
Chuyen de-luyen-thi-dh-2012
Chuyen de-luyen-thi-dh-2012
Chuyen de-luyen-thi-dh-2012
Chuyen de-luyen-thi-dh-2012
Chuyen de-luyen-thi-dh-2012
Chuyen de-luyen-thi-dh-2012
Chuyen de-luyen-thi-dh-2012
Chuyen de-luyen-thi-dh-2012
Chuyen de-luyen-thi-dh-2012
Chuyen de-luyen-thi-dh-2012
Chuyen de-luyen-thi-dh-2012
Chuyen de-luyen-thi-dh-2012
Chuyen de-luyen-thi-dh-2012
Chuyen de-luyen-thi-dh-2012
Chuyen de-luyen-thi-dh-2012
Chuyen de-luyen-thi-dh-2012
Chuyen de-luyen-thi-dh-2012
Chuyen de-luyen-thi-dh-2012
Chuyen de-luyen-thi-dh-2012
Chuyen de-luyen-thi-dh-2012
Chuyen de-luyen-thi-dh-2012
Chuyen de-luyen-thi-dh-2012
Chuyen de-luyen-thi-dh-2012
Chuyen de-luyen-thi-dh-2012
Chuyen de-luyen-thi-dh-2012
Chuyen de-luyen-thi-dh-2012
Chuyen de-luyen-thi-dh-2012
Chuyen de-luyen-thi-dh-2012
Chuyen de-luyen-thi-dh-2012
Chuyen de-luyen-thi-dh-2012
Chuyen de-luyen-thi-dh-2012
Chuyen de-luyen-thi-dh-2012
Chuyen de-luyen-thi-dh-2012
Chuyen de-luyen-thi-dh-2012
Chuyen de-luyen-thi-dh-2012
Chuyen de-luyen-thi-dh-2012
Chuyen de-luyen-thi-dh-2012
Chuyen de-luyen-thi-dh-2012
Chuyen de-luyen-thi-dh-2012
Chuyen de-luyen-thi-dh-2012
Chuyen de-luyen-thi-dh-2012
Chuyen de-luyen-thi-dh-2012
Chuyen de-luyen-thi-dh-2012
Chuyen de-luyen-thi-dh-2012
Chuyen de-luyen-thi-dh-2012
Chuyen de-luyen-thi-dh-2012
Chuyen de-luyen-thi-dh-2012
Chuyen de-luyen-thi-dh-2012
Chuyen de-luyen-thi-dh-2012
Chuyen de-luyen-thi-dh-2012
Chuyen de-luyen-thi-dh-2012
Chuyen de-luyen-thi-dh-2012
Chuyen de-luyen-thi-dh-2012
Chuyen de-luyen-thi-dh-2012
Chuyen de-luyen-thi-dh-2012
Chuyen de-luyen-thi-dh-2012
Chuyen de-luyen-thi-dh-2012
Chuyen de-luyen-thi-dh-2012
Chuyen de-luyen-thi-dh-2012
Chuyen de-luyen-thi-dh-2012
Chuyen de-luyen-thi-dh-2012
Chuyen de-luyen-thi-dh-2012
Chuyen de-luyen-thi-dh-2012
Chuyen de-luyen-thi-dh-2012
Chuyen de-luyen-thi-dh-2012
Chuyen de-luyen-thi-dh-2012
Chuyen de-luyen-thi-dh-2012
Chuyen de-luyen-thi-dh-2012
Chuyen de-luyen-thi-dh-2012
Chuyen de-luyen-thi-dh-2012
Chuyen de-luyen-thi-dh-2012
Chuyen de-luyen-thi-dh-2012
Chuyen de-luyen-thi-dh-2012
Chuyen de-luyen-thi-dh-2012
Chuyen de-luyen-thi-dh-2012
Chuyen de-luyen-thi-dh-2012
Chuyen de-luyen-thi-dh-2012
Chuyen de-luyen-thi-dh-2012
Chuyen de-luyen-thi-dh-2012
Chuyen de-luyen-thi-dh-2012
Chuyen de-luyen-thi-dh-2012
Chuyen de-luyen-thi-dh-2012
Chuyen de-luyen-thi-dh-2012
Chuyen de-luyen-thi-dh-2012
Chuyen de-luyen-thi-dh-2012
Chuyen de-luyen-thi-dh-2012
Chuyen de-luyen-thi-dh-2012
Chuyen de-luyen-thi-dh-2012
Chuyen de-luyen-thi-dh-2012
Chuyen de-luyen-thi-dh-2012
Chuyen de-luyen-thi-dh-2012
Chuyen de-luyen-thi-dh-2012
Chuyen de-luyen-thi-dh-2012
Chuyen de-luyen-thi-dh-2012
Chuyen de-luyen-thi-dh-2012
Chuyen de-luyen-thi-dh-2012
Chuyen de-luyen-thi-dh-2012
Chuyen de-luyen-thi-dh-2012
Chuyen de-luyen-thi-dh-2012
Chuyen de-luyen-thi-dh-2012
Chuyen de-luyen-thi-dh-2012
Chuyen de-luyen-thi-dh-2012
Chuyen de-luyen-thi-dh-2012
Chuyen de-luyen-thi-dh-2012
Chuyen de-luyen-thi-dh-2012
Chuyen de-luyen-thi-dh-2012
Chuyen de-luyen-thi-dh-2012
Chuyen de-luyen-thi-dh-2012
Chuyen de-luyen-thi-dh-2012
Chuyen de-luyen-thi-dh-2012
Chuyen de-luyen-thi-dh-2012
Chuyen de-luyen-thi-dh-2012
Chuyen de-luyen-thi-dh-2012
Chuyen de-luyen-thi-dh-2012
Chuyen de-luyen-thi-dh-2012
Chuyen de-luyen-thi-dh-2012
Chuyen de-luyen-thi-dh-2012
Chuyen de-luyen-thi-dh-2012
Chuyen de-luyen-thi-dh-2012
Chuyen de-luyen-thi-dh-2012
Chuyen de-luyen-thi-dh-2012
Chuyen de-luyen-thi-dh-2012
Chuyen de-luyen-thi-dh-2012
Chuyen de-luyen-thi-dh-2012
Chuyen de-luyen-thi-dh-2012
Chuyen de-luyen-thi-dh-2012
Chuyen de-luyen-thi-dh-2012
Chuyen de-luyen-thi-dh-2012
Chuyen de-luyen-thi-dh-2012
Chuyen de-luyen-thi-dh-2012
Chuyen de-luyen-thi-dh-2012
Chuyen de-luyen-thi-dh-2012
Chuyen de-luyen-thi-dh-2012
Chuyen de-luyen-thi-dh-2012
Chuyen de-luyen-thi-dh-2012
Chuyen de-luyen-thi-dh-2012
Chuyen de-luyen-thi-dh-2012
Chuyen de-luyen-thi-dh-2012
Chuyen de-luyen-thi-dh-2012
Chuyen de-luyen-thi-dh-2012
Chuyen de-luyen-thi-dh-2012
Chuyen de-luyen-thi-dh-2012
Chuyen de-luyen-thi-dh-2012
Chuyen de-luyen-thi-dh-2012
Chuyen de-luyen-thi-dh-2012
Chuyen de-luyen-thi-dh-2012
Chuyen de-luyen-thi-dh-2012
Chuyen de-luyen-thi-dh-2012
Chuyen de-luyen-thi-dh-2012
Chuyen de-luyen-thi-dh-2012
Chuyen de-luyen-thi-dh-2012
Chuyen de-luyen-thi-dh-2012
Chuyen de-luyen-thi-dh-2012
Chuyen de-luyen-thi-dh-2012
Chuyen de-luyen-thi-dh-2012
Chuyen de-luyen-thi-dh-2012
Chuyen de-luyen-thi-dh-2012
Chuyen de-luyen-thi-dh-2012
Chuyen de-luyen-thi-dh-2012
Chuyen de-luyen-thi-dh-2012
Chuyen de-luyen-thi-dh-2012
Chuyen de-luyen-thi-dh-2012
Chuyen de-luyen-thi-dh-2012
Chuyen de-luyen-thi-dh-2012
Chuyen de-luyen-thi-dh-2012
Chuyen de-luyen-thi-dh-2012
Chuyen de-luyen-thi-dh-2012
Chuyen de-luyen-thi-dh-2012
Chuyen de-luyen-thi-dh-2012
Chuyen de-luyen-thi-dh-2012
Chuyen de-luyen-thi-dh-2012
Chuyen de-luyen-thi-dh-2012
Chuyen de-luyen-thi-dh-2012
Chuyen de-luyen-thi-dh-2012
Chuyen de-luyen-thi-dh-2012
Chuyen de-luyen-thi-dh-2012
Chuyen de-luyen-thi-dh-2012
Chuyen de-luyen-thi-dh-2012
Chuyen de-luyen-thi-dh-2012
Chuyen de-luyen-thi-dh-2012
Chuyen de-luyen-thi-dh-2012
Chuyen de-luyen-thi-dh-2012
Chuyen de-luyen-thi-dh-2012
Chuyen de-luyen-thi-dh-2012
Chuyen de-luyen-thi-dh-2012
Chuyen de-luyen-thi-dh-2012
Chuyen de-luyen-thi-dh-2012
Chuyen de-luyen-thi-dh-2012
Chuyen de-luyen-thi-dh-2012
Chuyen de-luyen-thi-dh-2012
Chuyen de-luyen-thi-dh-2012
Chuyen de-luyen-thi-dh-2012
Chuyen de-luyen-thi-dh-2012
Chuyen de-luyen-thi-dh-2012
Chuyen de-luyen-thi-dh-2012
Chuyen de-luyen-thi-dh-2012
Chuyen de-luyen-thi-dh-2012
Chuyen de-luyen-thi-dh-2012
Chuyen de-luyen-thi-dh-2012
Chuyen de-luyen-thi-dh-2012
Chuyen de-luyen-thi-dh-2012
Chuyen de-luyen-thi-dh-2012
Chuyen de-luyen-thi-dh-2012
Chuyen de-luyen-thi-dh-2012
Chuyen de-luyen-thi-dh-2012
Chuyen de-luyen-thi-dh-2012
Chuyen de-luyen-thi-dh-2012
Chuyen de-luyen-thi-dh-2012
Chuyen de-luyen-thi-dh-2012
Chuyen de-luyen-thi-dh-2012
Chuyen de-luyen-thi-dh-2012
Chuyen de-luyen-thi-dh-2012
Chuyen de-luyen-thi-dh-2012
Chuyen de-luyen-thi-dh-2012

Weitere ähnliche Inhalte

Was ist angesagt?

Ltnc giao trinh 2017
Ltnc giao trinh 2017Ltnc giao trinh 2017
Ltnc giao trinh 2017TaiTranQuang
 
Bai giang atbmtt
Bai giang atbmttBai giang atbmtt
Bai giang atbmttHuynh MVT
 
{Nguoithay.vn} tong hop 10 phuong phap giai nhanh trac nghiem hoa hoc va 25 ...
{Nguoithay.vn}  tong hop 10 phuong phap giai nhanh trac nghiem hoa hoc va 25 ...{Nguoithay.vn}  tong hop 10 phuong phap giai nhanh trac nghiem hoa hoc va 25 ...
{Nguoithay.vn} tong hop 10 phuong phap giai nhanh trac nghiem hoa hoc va 25 ...Phong Phạm
 
Giao Trinh Lap Trinh Huong Doi Tuong
Giao Trinh Lap Trinh Huong Doi TuongGiao Trinh Lap Trinh Huong Doi Tuong
Giao Trinh Lap Trinh Huong Doi Tuongtrieulongnhi
 
Bat loi chinh_ta_tieng_viet
Bat loi chinh_ta_tieng_vietBat loi chinh_ta_tieng_viet
Bat loi chinh_ta_tieng_vietViet Nam
 
Đề tài: Đường thẳng và đường tròn trong hình học tọa độ lớp 10 - Gửi miễn phí...
Đề tài: Đường thẳng và đường tròn trong hình học tọa độ lớp 10 - Gửi miễn phí...Đề tài: Đường thẳng và đường tròn trong hình học tọa độ lớp 10 - Gửi miễn phí...
Đề tài: Đường thẳng và đường tròn trong hình học tọa độ lớp 10 - Gửi miễn phí...Dịch vụ viết bài trọn gói ZALO: 0909232620
 
Luận án: Nghiên cứu ứng dụng hiện tượng hỗn loạn của hệ thống động cho mật mã...
Luận án: Nghiên cứu ứng dụng hiện tượng hỗn loạn của hệ thống động cho mật mã...Luận án: Nghiên cứu ứng dụng hiện tượng hỗn loạn của hệ thống động cho mật mã...
Luận án: Nghiên cứu ứng dụng hiện tượng hỗn loạn của hệ thống động cho mật mã...Dịch vụ viết thuê Khóa Luận - ZALO 0932091562
 

Was ist angesagt? (16)

Đề tài: Đánh giá sai số hệ thống trong kỹ thuật quét gamma, 9đ
Đề tài: Đánh giá sai số hệ thống trong kỹ thuật quét gamma, 9đĐề tài: Đánh giá sai số hệ thống trong kỹ thuật quét gamma, 9đ
Đề tài: Đánh giá sai số hệ thống trong kỹ thuật quét gamma, 9đ
 
Vô tuyến nhận thức hợp tác cảm nhận phổ trong môi trường pha đinh
Vô tuyến nhận thức hợp tác cảm nhận phổ trong môi trường pha đinhVô tuyến nhận thức hợp tác cảm nhận phổ trong môi trường pha đinh
Vô tuyến nhận thức hợp tác cảm nhận phổ trong môi trường pha đinh
 
Ltnc giao trinh 2017
Ltnc giao trinh 2017Ltnc giao trinh 2017
Ltnc giao trinh 2017
 
Bai giang atbmtt
Bai giang atbmttBai giang atbmtt
Bai giang atbmtt
 
{Nguoithay.vn} tong hop 10 phuong phap giai nhanh trac nghiem hoa hoc va 25 ...
{Nguoithay.vn}  tong hop 10 phuong phap giai nhanh trac nghiem hoa hoc va 25 ...{Nguoithay.vn}  tong hop 10 phuong phap giai nhanh trac nghiem hoa hoc va 25 ...
{Nguoithay.vn} tong hop 10 phuong phap giai nhanh trac nghiem hoa hoc va 25 ...
 
Ôn tập các dạng toán lớp 3
Ôn tập các dạng toán lớp 3Ôn tập các dạng toán lớp 3
Ôn tập các dạng toán lớp 3
 
Giao trinh ngon ngu lap trinh c#
Giao trinh ngon ngu lap trinh c#Giao trinh ngon ngu lap trinh c#
Giao trinh ngon ngu lap trinh c#
 
Giao Trinh Lap Trinh Huong Doi Tuong
Giao Trinh Lap Trinh Huong Doi TuongGiao Trinh Lap Trinh Huong Doi Tuong
Giao Trinh Lap Trinh Huong Doi Tuong
 
Bat loi chinh_ta_tieng_viet
Bat loi chinh_ta_tieng_vietBat loi chinh_ta_tieng_viet
Bat loi chinh_ta_tieng_viet
 
Đề tài: Đường thẳng và đường tròn trong hình học tọa độ lớp 10 - Gửi miễn phí...
Đề tài: Đường thẳng và đường tròn trong hình học tọa độ lớp 10 - Gửi miễn phí...Đề tài: Đường thẳng và đường tròn trong hình học tọa độ lớp 10 - Gửi miễn phí...
Đề tài: Đường thẳng và đường tròn trong hình học tọa độ lớp 10 - Gửi miễn phí...
 
La0008
La0008La0008
La0008
 
Luận án: Nghiên cứu ứng dụng hiện tượng hỗn loạn của hệ thống động cho mật mã...
Luận án: Nghiên cứu ứng dụng hiện tượng hỗn loạn của hệ thống động cho mật mã...Luận án: Nghiên cứu ứng dụng hiện tượng hỗn loạn của hệ thống động cho mật mã...
Luận án: Nghiên cứu ứng dụng hiện tượng hỗn loạn của hệ thống động cho mật mã...
 
Đề tài: Nhận dạng, phân loại, xử lý ảnh biển số xe bằng phần mềm
Đề tài: Nhận dạng, phân loại, xử lý ảnh biển số xe bằng phần mềmĐề tài: Nhận dạng, phân loại, xử lý ảnh biển số xe bằng phần mềm
Đề tài: Nhận dạng, phân loại, xử lý ảnh biển số xe bằng phần mềm
 
BÀI TẬP 35 TUẦN MÔN TOÁN 4
BÀI TẬP 35 TUẦN MÔN TOÁN 4BÀI TẬP 35 TUẦN MÔN TOÁN 4
BÀI TẬP 35 TUẦN MÔN TOÁN 4
 
Chuyên đề tich phan on thi dh
Chuyên đề tich phan on thi dhChuyên đề tich phan on thi dh
Chuyên đề tich phan on thi dh
 
Luận văn: Bài toán nhận dạng biển số xe, HAY
Luận văn: Bài toán nhận dạng biển số xe, HAYLuận văn: Bài toán nhận dạng biển số xe, HAY
Luận văn: Bài toán nhận dạng biển số xe, HAY
 

Ähnlich wie Chuyen de-luyen-thi-dh-2012

Giáo trình Tester Full
Giáo trình Tester FullGiáo trình Tester Full
Giáo trình Tester FullThanh Sơn
 
TongHopLyThuyet.pdf
TongHopLyThuyet.pdfTongHopLyThuyet.pdf
TongHopLyThuyet.pdfmaytinh_5p
 
bai-giang-thong-ke-xa-hoi-hoc-2022.pdf
bai-giang-thong-ke-xa-hoi-hoc-2022.pdfbai-giang-thong-ke-xa-hoi-hoc-2022.pdf
bai-giang-thong-ke-xa-hoi-hoc-2022.pdfPhmVnt11
 
3 câu thi thử;
3 câu thi thử;3 câu thi thử;
3 câu thi thử;nam nam
 
Bài giảng giải tích 3 - thầy nguyễn xuân diệu.pdf
Bài giảng giải tích 3 - thầy nguyễn xuân diệu.pdfBài giảng giải tích 3 - thầy nguyễn xuân diệu.pdf
Bài giảng giải tích 3 - thầy nguyễn xuân diệu.pdfTrường Việt Nam
 
OLYMPIC SINH VIÊN MÔN ĐẠI SỐ ĐỊNH THỨC, HỆ PHƯƠNG TRÌNH TUYẾN TÍNH, MA TRẬN V...
OLYMPIC SINH VIÊN MÔN ĐẠI SỐ ĐỊNH THỨC, HỆ PHƯƠNG TRÌNH TUYẾN TÍNH, MA TRẬN V...OLYMPIC SINH VIÊN MÔN ĐẠI SỐ ĐỊNH THỨC, HỆ PHƯƠNG TRÌNH TUYẾN TÍNH, MA TRẬN V...
OLYMPIC SINH VIÊN MÔN ĐẠI SỐ ĐỊNH THỨC, HỆ PHƯƠNG TRÌNH TUYẾN TÍNH, MA TRẬN V...Man_Ebook
 
Đề thi đại học môn Toán theo chủ đề từ 2002 đến 2012
Đề thi đại học môn Toán theo chủ đề từ 2002 đến 2012 Đề thi đại học môn Toán theo chủ đề từ 2002 đến 2012
Đề thi đại học môn Toán theo chủ đề từ 2002 đến 2012 Summer Song
 
Tri tue-nhan-tao-dinh-manh-tuong
Tri tue-nhan-tao-dinh-manh-tuongTri tue-nhan-tao-dinh-manh-tuong
Tri tue-nhan-tao-dinh-manh-tuongQuyên Đinh
 
giao-trinh-python-co-ban-9642-2-10.pdf
giao-trinh-python-co-ban-9642-2-10.pdfgiao-trinh-python-co-ban-9642-2-10.pdf
giao-trinh-python-co-ban-9642-2-10.pdfLeQucPhi
 
giao-trinh-python-co-ban.pdf
giao-trinh-python-co-ban.pdfgiao-trinh-python-co-ban.pdf
giao-trinh-python-co-ban.pdfCongNgo13
 
Bai giang toan kinh te 2015
Bai giang toan kinh te 2015Bai giang toan kinh te 2015
Bai giang toan kinh te 2015ICTU
 
K2pi.net --ms ebook release
K2pi.net --ms ebook releaseK2pi.net --ms ebook release
K2pi.net --ms ebook releaseMit Rin
 
ĐIỀU KHIỂN HỆ ĐA TÁC TỬ.pdf
ĐIỀU KHIỂN HỆ ĐA TÁC TỬ.pdfĐIỀU KHIỂN HỆ ĐA TÁC TỬ.pdf
ĐIỀU KHIỂN HỆ ĐA TÁC TỬ.pdfMan_Ebook
 
Phuong trinh he pt
Phuong trinh   he ptPhuong trinh   he pt
Phuong trinh he ptcongbang12vn
 
Chuong trinh bat loi chinh ta
Chuong trinh bat loi chinh taChuong trinh bat loi chinh ta
Chuong trinh bat loi chinh taVcoi Vit
 
Bat loi chinh_ta_tieng_viet
Bat loi chinh_ta_tieng_vietBat loi chinh_ta_tieng_viet
Bat loi chinh_ta_tieng_vietDuy Vọng
 
Tổng hợp các phương pháp giải phương trình và hệ phương trình môn toán
Tổng hợp các phương pháp giải phương trình và hệ phương trình môn toánTổng hợp các phương pháp giải phương trình và hệ phương trình môn toán
Tổng hợp các phương pháp giải phương trình và hệ phương trình môn toánhuyenltv274
 

Ähnlich wie Chuyen de-luyen-thi-dh-2012 (20)

Giáo trình Tester Full
Giáo trình Tester FullGiáo trình Tester Full
Giáo trình Tester Full
 
TongHopLyThuyet.pdf
TongHopLyThuyet.pdfTongHopLyThuyet.pdf
TongHopLyThuyet.pdf
 
bai-giang-thong-ke-xa-hoi-hoc-2022.pdf
bai-giang-thong-ke-xa-hoi-hoc-2022.pdfbai-giang-thong-ke-xa-hoi-hoc-2022.pdf
bai-giang-thong-ke-xa-hoi-hoc-2022.pdf
 
3 câu thi thử;
3 câu thi thử;3 câu thi thử;
3 câu thi thử;
 
Bai_Giang_GT3.pdf
Bai_Giang_GT3.pdfBai_Giang_GT3.pdf
Bai_Giang_GT3.pdf
 
Bài giảng giải tích 3 - thầy nguyễn xuân diệu.pdf
Bài giảng giải tích 3 - thầy nguyễn xuân diệu.pdfBài giảng giải tích 3 - thầy nguyễn xuân diệu.pdf
Bài giảng giải tích 3 - thầy nguyễn xuân diệu.pdf
 
OLYMPIC SINH VIÊN MÔN ĐẠI SỐ ĐỊNH THỨC, HỆ PHƯƠNG TRÌNH TUYẾN TÍNH, MA TRẬN V...
OLYMPIC SINH VIÊN MÔN ĐẠI SỐ ĐỊNH THỨC, HỆ PHƯƠNG TRÌNH TUYẾN TÍNH, MA TRẬN V...OLYMPIC SINH VIÊN MÔN ĐẠI SỐ ĐỊNH THỨC, HỆ PHƯƠNG TRÌNH TUYẾN TÍNH, MA TRẬN V...
OLYMPIC SINH VIÊN MÔN ĐẠI SỐ ĐỊNH THỨC, HỆ PHƯƠNG TRÌNH TUYẾN TÍNH, MA TRẬN V...
 
Đề thi đại học môn Toán theo chủ đề từ 2002 đến 2012
Đề thi đại học môn Toán theo chủ đề từ 2002 đến 2012 Đề thi đại học môn Toán theo chủ đề từ 2002 đến 2012
Đề thi đại học môn Toán theo chủ đề từ 2002 đến 2012
 
Luận văn: Các dạng phương trình lượng giác, HOT - Gửi miễn phí qua zalo=> 090...
Luận văn: Các dạng phương trình lượng giác, HOT - Gửi miễn phí qua zalo=> 090...Luận văn: Các dạng phương trình lượng giác, HOT - Gửi miễn phí qua zalo=> 090...
Luận văn: Các dạng phương trình lượng giác, HOT - Gửi miễn phí qua zalo=> 090...
 
Tri tue-nhan-tao-dinh-manh-tuong
Tri tue-nhan-tao-dinh-manh-tuongTri tue-nhan-tao-dinh-manh-tuong
Tri tue-nhan-tao-dinh-manh-tuong
 
Dãy số và giới hạn
Dãy số và giới hạnDãy số và giới hạn
Dãy số và giới hạn
 
giao-trinh-python-co-ban-9642-2-10.pdf
giao-trinh-python-co-ban-9642-2-10.pdfgiao-trinh-python-co-ban-9642-2-10.pdf
giao-trinh-python-co-ban-9642-2-10.pdf
 
giao-trinh-python-co-ban.pdf
giao-trinh-python-co-ban.pdfgiao-trinh-python-co-ban.pdf
giao-trinh-python-co-ban.pdf
 
Bai giang toan kinh te 2015
Bai giang toan kinh te 2015Bai giang toan kinh te 2015
Bai giang toan kinh te 2015
 
K2pi.net --ms ebook release
K2pi.net --ms ebook releaseK2pi.net --ms ebook release
K2pi.net --ms ebook release
 
ĐIỀU KHIỂN HỆ ĐA TÁC TỬ.pdf
ĐIỀU KHIỂN HỆ ĐA TÁC TỬ.pdfĐIỀU KHIỂN HỆ ĐA TÁC TỬ.pdf
ĐIỀU KHIỂN HỆ ĐA TÁC TỬ.pdf
 
Phuong trinh he pt
Phuong trinh   he ptPhuong trinh   he pt
Phuong trinh he pt
 
Chuong trinh bat loi chinh ta
Chuong trinh bat loi chinh taChuong trinh bat loi chinh ta
Chuong trinh bat loi chinh ta
 
Bat loi chinh_ta_tieng_viet
Bat loi chinh_ta_tieng_vietBat loi chinh_ta_tieng_viet
Bat loi chinh_ta_tieng_viet
 
Tổng hợp các phương pháp giải phương trình và hệ phương trình môn toán
Tổng hợp các phương pháp giải phương trình và hệ phương trình môn toánTổng hợp các phương pháp giải phương trình và hệ phương trình môn toán
Tổng hợp các phương pháp giải phương trình và hệ phương trình môn toán
 

Chuyen de-luyen-thi-dh-2012

  • 1. TRẦN ANH TUẤN TRƯỜNG ĐẠI HỌC THƯƠNG MẠI Các chuyên đề LUYỆN THI ĐẠI HỌC HÀ NỘI - 2011 WWW.VNMATH.COM
  • 2.
  • 3. Mục lục I Đại số - Lượng giác - Giải tích 9 Chương 1 Phương trình, bất phương trình, hệ đại số 11 1.1. Phương trình, bất phương trình đa thức . . . . . . . . . . . . . . . . . . . . . . . . . . . . . . . . . . . . . 11 1.1.1. Phương trình, bất phương trình bậc hai . . . . . . . . . . . . . . . . . . . . . . . . . . . . . . . . . 11 1.1.2. Phương trình trình bậc ba . . . . . . . . . . . . . . . . . . . . . . . . . . . . . . . . . . . . . . . . 13 1.1.3. Phương trình, bất phương trình bậc bốn . . . . . . . . . . . . . . . . . . . . . . . . . . . . . . . . 13 1.2. Phương trình, bất phương trình chứa giá trị tuyệt đối . . . . . . . . . . . . . . . . . . . . . . . . . . . . . . 14 1.3. Phương trình, bất phương trình chứa căn . . . . . . . . . . . . . . . . . . . . . . . . . . . . . . . . . . . . 16 Vấn đề 1 : Phương trình, bất phương trình cơ bản . . . . . . . . . . . . . . . . . . . . . . . . . . . . . . . 16 Vấn đề 2 : Phương pháp đặt ẩn phụ . . . . . . . . . . . . . . . . . . . . . . . . . . . . . . . . . . . . . . 17 Vấn đề 3 : Phương pháp nhân liên hợp . . . . . . . . . . . . . . . . . . . . . . . . . . . . . . . . . . . . . 19 Vấn đề 4 : Phương pháp đánh giá . . . . . . . . . . . . . . . . . . . . . . . . . . . . . . . . . . . . . . . 19 Vấn đề 5 : Phương trình, bất phương trình có tham số . . . . . . . . . . . . . . . . . . . . . . . . . . . . . 20 1.4. Hệ phương trình . . . . . . . . . . . . . . . . . . . . . . . . . . . . . . . . . . . . . . . . . . . . . . . . . 23 1.4.1. Phương pháp thế . . . . . . . . . . . . . . . . . . . . . . . . . . . . . . . . . . . . . . . . . . . . 23 1.4.2. Phương pháp phân tích thành nhân tử hoặc coi một phương trình là phương trình bậc hai (ba) theo một ẩn . . . . . . . . . . . . . . . . . . . . . . . . . . . . . . . . . . . . . . . . . . . . . . . . . 24 1.4.3. Phương pháp đặt ẩn phụ . . . . . . . . . . . . . . . . . . . . . . . . . . . . . . . . . . . . . . . . 24 1.4.4. Phương pháp hàm số . . . . . . . . . . . . . . . . . . . . . . . . . . . . . . . . . . . . . . . . . . 27 1.4.5. Phương pháp đánh giá . . . . . . . . . . . . . . . . . . . . . . . . . . . . . . . . . . . . . . . . . 27 1.5. Số nghiệm của phương trình, hệ phương trình . . . . . . . . . . . . . . . . . . . . . . . . . . . . . . . . . 28 Vấn đề 1 : Chứng minh phương trình có nghiệm duy nhất . . . . . . . . . . . . . . . . . . . . . . . . . . . 28 Vấn đề 2 : Chứng minh phương trình có đúng hai nghiệm phân biệt . . . . . . . . . . . . . . . . . . . . . . 28 Vấn đề 3 : Chứng minh phương trình có đúng ba nghiệm phân biệt . . . . . . . . . . . . . . . . . . . . . . 29 1.6. Phương trình, bất phương trình, hệ đại số trong các kì thi tuyển sinh ĐH . . . . . . . . . . . . . . . . . . . 29 1.7. Bài tập tổng hợp . . . . . . . . . . . . . . . . . . . . . . . . . . . . . . . . . . . . . . . . . . . . . . . . . 31 Chương 2 Bất đẳng thức 37 2.1. Phương pháp sử dụng bất đẳng thức Cauchy . . . . . . . . . . . . . . . . . . . . . . . . . . . . . . . . . . 37 2.1.1. Bất đẳng thức Cauchy - So sánh giữa tổng và tích . . . . . . . . . . . . . . . . . . . . . . . . . . . 37 2.1.2. Một số hệ quả trực tiếp . . . . . . . . . . . . . . . . . . . . . . . . . . . . . . . . . . . . . . . . . 37 2.1.3. Bài tập đề nghị . . . . . . . . . . . . . . . . . . . . . . . . . . . . . . . . . . . . . . . . . . . . . 37 2.2. Bất đẳng thức hình học . . . . . . . . . . . . . . . . . . . . . . . . . . . . . . . . . . . . . . . . . . . . . 42 2.3. Phương pháp sử dụng điều kiện có nghiệm của phương trình hoặc hệ phương trình . . . . . . . . . . . . . . 44 3 WWW.VNMATH.COM
  • 4. 2.4. Bất đẳng thức trong các kì thi tuyển sinh ĐH . . . . . . . . . . . . . . . . . . . . . . . . . . . . . . . . . . 44 2.5. Bài tập tổng hợp . . . . . . . . . . . . . . . . . . . . . . . . . . . . . . . . . . . . . . . . . . . . . . . . . 46 Chương 3 Lượng giác 51 3.1. Phương trình cơ bản . . . . . . . . . . . . . . . . . . . . . . . . . . . . . . . . . . . . . . . . . . . . . . . 51 3.2. Phương trình dạng a sin x + b cos x = c . . . . . . . . . . . . . . . . . . . . . . . . . . . . . . . . . . . . . 52 3.3. Phương pháp đặt ẩn phụ . . . . . . . . . . . . . . . . . . . . . . . . . . . . . . . . . . . . . . . . . . . . . 53 3.4. Đưa phương trình về dạng tích . . . . . . . . . . . . . . . . . . . . . . . . . . . . . . . . . . . . . . . . . 60 3.5. Phương pháp đánh giá và phương pháp hàm số . . . . . . . . . . . . . . . . . . . . . . . . . . . . . . . . . 62 3.6. Giá trị lớn nhất và nhỏ nhất của biểu thức lượng giác . . . . . . . . . . . . . . . . . . . . . . . . . . . . . 63 3.7. Lượng giác trong các kì thi tuyển sinh ĐH . . . . . . . . . . . . . . . . . . . . . . . . . . . . . . . . . . . 63 3.8. Bài tập tổng hợp . . . . . . . . . . . . . . . . . . . . . . . . . . . . . . . . . . . . . . . . . . . . . . . . . 64 Chương 4 Tổ hợp 69 4.1. Các quy tắc đếm. Tổ hợp, chỉnh hợp, hoán vị . . . . . . . . . . . . . . . . . . . . . . . . . . . . . . . . . . 69 4.2. Giải phương trình, bất phương trình, hệ . . . . . . . . . . . . . . . . . . . . . . . . . . . . . . . . . . . . 74 4.3. Hệ số của xk trong khai triển . . . . . . . . . . . . . . . . . . . . . . . . . . . . . . . . . . . . . . . . . . 76 4.4. Hệ số của xk trong khai triển nhị thức (a + b)n . . . . . . . . . . . . . . . . . . . . . . . . . . . . . . . . . 76 4.5. Hệ số của xk trong khai triển (a + b)n(c + d)m . . . . . . . . . . . . . . . . . . . . . . . . . . . . . . . . . 77 4.6. Hệ số của xk trong khai triển (a + b + c)n . . . . . . . . . . . . . . . . . . . . . . . . . . . . . . . . . . . . 77 4.7. Tính tổng các hệ số tổ hợp : nÈ k=0 akCk n . . . . . . . . . . . . . . . . . . . . . . . . . . . . . . . . . . . . . . 77 4.8. Phương pháp cơ bản với ak chỉ là hàm số mũ theo biến k . . . . . . . . . . . . . . . . . . . . . . . . . . . 77 4.9. Phương pháp đạo hàm với ak là tích hàm số mũ và đa thức theo k . . . . . . . . . . . . . . . . . . . . . . . 78 4.10. Phương pháp tích phân với ak là tích hàm số mũ và phân thức theo k . . . . . . . . . . . . . . . . . . . . . 79 4.11. Bài tập tổng hợp . . . . . . . . . . . . . . . . . . . . . . . . . . . . . . . . . . . . . . . . . . . . . . . . . 80 Chương 5 Hàm số 83 5.1. Tính đơn điệu . . . . . . . . . . . . . . . . . . . . . . . . . . . . . . . . . . . . . . . . . . . . . . . . . . 83 Vấn đề 1 : Xét chiều biến thiên của hàm số . . . . . . . . . . . . . . . . . . . . . . . . . . . . . . . . . . . 83 Vấn đề 2 : Tìm điều kiện tham số để hàm số đơn điệu trên một miền . . . . . . . . . . . . . . . . . . . . . . 84 Vấn đề 3 : Giá trị lớn nhất, giá trị nhỏ nhất của hàm một biến số . . . . . . . . . . . . . . . . . . . . . . . 87 Vấn đề 4 : Sử dụng tính đơn điệu chứng minh bất đẳng thức . . . . . . . . . . . . . . . . . . . . . . . . . . 89 Vấn đề 5 : Ứng dụng sự biến thiên vào việc giải phương trình, bất phương trình, hệ . . . . . . . . . . . . . 91 Vấn đề 6 : Ứng dụng sự biến thiên vào bài toán số nghiệm phương trình có tham số . . . . . . . . . . . . . 92 5.2. Cực trị của hàm số . . . . . . . . . . . . . . . . . . . . . . . . . . . . . . . . . . . . . . . . . . . . . . . 93 Vấn đề 1 : Sử dụng dấu hiệu 1 và dấu hiệu 2 để xác định các điểm cực trị của hàm số . . . . . . . . . . . . 94 Vấn đề 2 : Điều kiện của tham số để hàm số đạt cực trị (cực đại hoặc cực tiểu) tại x = x0 hoặc đồ thị hàm số đạt cực trị tại điểm (x0; y0) . . . . . . . . . . . . . . . . . . . . . . . . . . . . . . . . . . . . . 94 Vấn đề 3 : Tìm điều kiện để hàm số có cực trị và thỏa mãn một vài điều kiện . . . . . . . . . . . . . . . . . 95 5.3. Tiệm cận . . . . . . . . . . . . . . . . . . . . . . . . . . . . . . . . . . . . . . . . . . . . . . . . . . . . 100 Vấn đề 1 : Tìm tiệm cận của đồ thị hàm số . . . . . . . . . . . . . . . . . . . . . . . . . . . . . . . . . . . 100 Vấn đề 2 : Các bài toán về tiệm cận có tham số . . . . . . . . . . . . . . . . . . . . . . . . . . . . . . . . 101 5.4. Tâm đối xứng và trục đối xứng. Điểm thuộc đồ thị . . . . . . . . . . . . . . . . . . . . . . . . . . . . . . . 102
  • 5. Vấn đề 1 : Tâm đối xứng, trục đối xứng . . . . . . . . . . . . . . . . . . . . . . . . . . . . . . . . . . . . . 102 Vấn đề 2 : Khoảng cách . . . . . . . . . . . . . . . . . . . . . . . . . . . . . . . . . . . . . . . . . . . . . 102 5.5. Biện luận số nghiệm của phương trình, bất phương trình bằng phương pháp đồ thị . . . . . . . . . . . . . . 103 5.6. Bài toán về sự tương giao . . . . . . . . . . . . . . . . . . . . . . . . . . . . . . . . . . . . . . . . . . . . 108 5.7. Sự tiếp xúc của hai đường cong và tiếp tuyến . . . . . . . . . . . . . . . . . . . . . . . . . . . . . . . . . . 109 Vấn đề 1 : Viết phương trình tiếp tuyến biết tiếp điểm . . . . . . . . . . . . . . . . . . . . . . . . . . . . . 109 Vấn đề 2 : Hai đường cong tiếp xúc . . . . . . . . . . . . . . . . . . . . . . . . . . . . . . . . . . . . . . . 111 Vấn đề 3 : Tiếp tuyến đi qua một điểm . . . . . . . . . . . . . . . . . . . . . . . . . . . . . . . . . . . . . 112 Vấn đề 4 : Tiếp tuyến có hệ số góc cho trước . . . . . . . . . . . . . . . . . . . . . . . . . . . . . . . . . . 113 5.8. Hàm số trong các kì thi tuyển sinh ĐH . . . . . . . . . . . . . . . . . . . . . . . . . . . . . . . . . . . . . 114 5.9. Bài tập tổng hợp . . . . . . . . . . . . . . . . . . . . . . . . . . . . . . . . . . . . . . . . . . . . . . . . . 121 Chương 6 Mũ và lôgarít 127 6.1. Hàm số mũ, hàm số lũy thừa . . . . . . . . . . . . . . . . . . . . . . . . . . . . . . . . . . . . . . . . . . 127 6.2. Hàm số logarit . . . . . . . . . . . . . . . . . . . . . . . . . . . . . . . . . . . . . . . . . . . . . . . . . . 127 6.3. Phương trình mũ và logarit . . . . . . . . . . . . . . . . . . . . . . . . . . . . . . . . . . . . . . . . . . . 129 Vấn đề 1 : Phương trình cơ bản . . . . . . . . . . . . . . . . . . . . . . . . . . . . . . . . . . . . . . . . . 129 Vấn đề 2 : Phương pháp logarit hai vế . . . . . . . . . . . . . . . . . . . . . . . . . . . . . . . . . . . . . 130 Vấn đề 3 : Phương pháp đặt ẩn phụ . . . . . . . . . . . . . . . . . . . . . . . . . . . . . . . . . . . . . . 130 Vấn đề 4 : Phương pháp phân tích thành nhân tử . . . . . . . . . . . . . . . . . . . . . . . . . . . . . . . 131 Vấn đề 5 : Phương pháp đánh giá . . . . . . . . . . . . . . . . . . . . . . . . . . . . . . . . . . . . . . . 131 6.4. Bất phương trình mũ và logarit . . . . . . . . . . . . . . . . . . . . . . . . . . . . . . . . . . . . . . . . . 132 Vấn đề 1 : Bất phương trình cơ bản . . . . . . . . . . . . . . . . . . . . . . . . . . . . . . . . . . . . . . . 132 Vấn đề 2 : Phương pháp đặt ẩn phụ . . . . . . . . . . . . . . . . . . . . . . . . . . . . . . . . . . . . . . 133 Vấn đề 3 : Phương pháp phân tích thành nhân tử . . . . . . . . . . . . . . . . . . . . . . . . . . . . . . . 134 6.5. Hệ phương trình . . . . . . . . . . . . . . . . . . . . . . . . . . . . . . . . . . . . . . . . . . . . . . . . . 134 6.6. Phương trình mũ và lôgarit trong các kì thi tuyển sinh ĐH . . . . . . . . . . . . . . . . . . . . . . . . . . . 135 6.7. Bài tập tổng hợp . . . . . . . . . . . . . . . . . . . . . . . . . . . . . . . . . . . . . . . . . . . . . . . . . 136 Chương 7 Tích phân 149 7.1. Các dạng toán cơ bản về nguyên hàm . . . . . . . . . . . . . . . . . . . . . . . . . . . . . . . . . . . . . . 149 Vấn đề 1 : Chứng minh một hàm số F(x) là một nguyên hàm của hàm số f(x) . . . . . . . . . . . . . . . . 149 Vấn đề 2 : Sử dụng bảng nguyên hàm cơ bản . . . . . . . . . . . . . . . . . . . . . . . . . . . . . . . . . . 149 Vấn đề 3 : Tìm hằng số C . . . . . . . . . . . . . . . . . . . . . . . . . . . . . . . . . . . . . . . . . . . . 150 Vấn đề 4 : Phương pháp nguyên hàm từng phần . . . . . . . . . . . . . . . . . . . . . . . . . . . . . . . . 150 Vấn đề 5 : Phương pháp đổi biến số . . . . . . . . . . . . . . . . . . . . . . . . . . . . . . . . . . . . . . 151 7.2. Các dạng toán tích phân . . . . . . . . . . . . . . . . . . . . . . . . . . . . . . . . . . . . . . . . . . . . . 152 Vấn đề 1 : Sử dụng tích phân cơ bản . . . . . . . . . . . . . . . . . . . . . . . . . . . . . . . . . . . . . . 152 Vấn đề 2 : Tích phân hàm chứa dấu trị tuyệt đối . . . . . . . . . . . . . . . . . . . . . . . . . . . . . . . . 152 Vấn đề 3 : Phương pháp tích phân từng phần . . . . . . . . . . . . . . . . . . . . . . . . . . . . . . . . . 153 Vấn đề 4 : Phương pháp đổi biến số . . . . . . . . . . . . . . . . . . . . . . . . . . . . . . . . . . . . . . 154 Vấn đề 5 : Tích phân các hàm hữu tỉ . . . . . . . . . . . . . . . . . . . . . . . . . . . . . . . . . . . . . . 157 Vấn đề 6 : Tích phân một số hàm đặc biệt . . . . . . . . . . . . . . . . . . . . . . . . . . . . . . . . . . . 159
  • 6. 7.3. Ứng dụng tích phân để tính diện tích hình phẳng . . . . . . . . . . . . . . . . . . . . . . . . . . . . . . . . 161 7.4. Ứng dụng tích phân tính thể tích vật thể tròn xoay . . . . . . . . . . . . . . . . . . . . . . . . . . . . . . . 162 7.5. Tích phân trong các kì thi ĐH . . . . . . . . . . . . . . . . . . . . . . . . . . . . . . . . . . . . . . . . . 163 7.6. Bài tập tổng hợp . . . . . . . . . . . . . . . . . . . . . . . . . . . . . . . . . . . . . . . . . . . . . . . . . 164 Chương 8 Số phức 167 II Hình học 173 Chương 9 Phương pháp tọa độ trong trong mặt phẳng 175 9.1. Phương pháp tọa độ trong mặt phẳng . . . . . . . . . . . . . . . . . . . . . . . . . . . . . . . . . . . . . . 175 9.2. Phương trình của đường thẳng . . . . . . . . . . . . . . . . . . . . . . . . . . . . . . . . . . . . . . . . . 176 9.2.1. Các bài toán thiết lập phương trình đường thẳng . . . . . . . . . . . . . . . . . . . . . . . . . . . . 176 9.2.2. Các bài toán liên quan đến việc sử dụng phương trình đường thẳng . . . . . . . . . . . . . . . . . . 176 9.2.3. Bài tập tổng hợp . . . . . . . . . . . . . . . . . . . . . . . . . . . . . . . . . . . . . . . . . . . . 177 9.3. Đường tròn . . . . . . . . . . . . . . . . . . . . . . . . . . . . . . . . . . . . . . . . . . . . . . . . . . . 180 9.4. Đường elip . . . . . . . . . . . . . . . . . . . . . . . . . . . . . . . . . . . . . . . . . . . . . . . . . . . 183 9.5. Đường hypebol . . . . . . . . . . . . . . . . . . . . . . . . . . . . . . . . . . . . . . . . . . . . . . . . . 184 9.6. Đường parabol . . . . . . . . . . . . . . . . . . . . . . . . . . . . . . . . . . . . . . . . . . . . . . . . . 186 9.7. Phương pháp tọa độ trong mặt phẳng qua các kì thi tuyển sinh ĐH . . . . . . . . . . . . . . . . . . . . . . 187 9.8. Bài tập tổng hợp . . . . . . . . . . . . . . . . . . . . . . . . . . . . . . . . . . . . . . . . . . . . . . . . . 188 Chương 10 Mở đầu về hình học không gian. Quan hệ song song 191 10.1. Đại cương về đường thẳng và mặt phẳng . . . . . . . . . . . . . . . . . . . . . . . . . . . . . . . . . . . . 192 Vấn đề 1 : Xác định giao tuyến của hai mặt phẳng . . . . . . . . . . . . . . . . . . . . . . . . . . . . . . . 192 Vấn đề 2 : Xác định giao điểm của đường thẳng a và mặt phẳng (P) . . . . . . . . . . . . . . . . . . . . . 192 Vấn đề 3 : Phương pháp chứng minh ba điểm thẳng hàng và ba đường thẳng đồng quy . . . . . . . . . . . 193 Vấn đề 4 : Tìm thiết diện của hình chóp cắt bởi mặt phẳng . . . . . . . . . . . . . . . . . . . . . . . . . . . 193 10.2. Hai đường thẳng song song . . . . . . . . . . . . . . . . . . . . . . . . . . . . . . . . . . . . . . . . . . . 195 Vấn đề 1 : Tìm giao tuyến của hai mặt phẳng (dùng quan hệ song song) . . . . . . . . . . . . . . . . . . . 195 Vấn đề 2 : Chứng minh hai đường thẳng song song . . . . . . . . . . . . . . . . . . . . . . . . . . . . . . 196 Vấn đề 3 : Chứng minh hai đường thẳng chéo nhau . . . . . . . . . . . . . . . . . . . . . . . . . . . . . . 196 10.3. Đường thẳng và mặt phẳng song song . . . . . . . . . . . . . . . . . . . . . . . . . . . . . . . . . . . . . 197 Vấn đề 1 : Chứng minh đường thẳng song song với mặt phẳng . . . . . . . . . . . . . . . . . . . . . . . . 197 Vấn đề 2 : Tìm giao tuyến của hai mặt phẳng. Dựng thiết diện song song với một đường thẳng . . . . . . . 197 Vấn đề 3 : Dựng một mặt phẳng chứa một đường thẳng và song song với đường thẳng khác Xác định giao điểm của đường thẳng với mặt phẳng . . . . . . . . . . . . . . . . . . . . . . . . . . 198 10.4. Hai mặt phẳng song song . . . . . . . . . . . . . . . . . . . . . . . . . . . . . . . . . . . . . . . . . . . . 199 Vấn đề 1 : Chứng minh hai mặt phẳng song song . . . . . . . . . . . . . . . . . . . . . . . . . . . . . . . 199 Vấn đề 2 : Tìm giao tuyến của hai mặt phẳng Thiết diện cắt bởi một mặt phẳng song song với một mặt phẳng cho trước . . . . . . . . . . . . . . 199
  • 7. Chương 11 Vectơ trong không gian. Quan hệ vuông góc 201 11.1. Vectơ trong không gian. Sự đồng phẳng của các vectơ . . . . . . . . . . . . . . . . . . . . . . . . . . . . . 202 Vấn đề 1 : Biểu thị một vectơ qua ba vectơ không đồng phẳng . . . . . . . . . . . . . . . . . . . . . . . . . 202 Vấn đề 2 : Chứng minh các đẳng thức vectơ . . . . . . . . . . . . . . . . . . . . . . . . . . . . . . . . . . 203 Vấn đề 3 : Chứng minh các điểm thẳng hàng và quan hệ song song . . . . . . . . . . . . . . . . . . . . . . 203 Vấn đề 4 : Chứng minh các vectơ đồng phẳng . . . . . . . . . . . . . . . . . . . . . . . . . . . . . . . . . 204 11.2. Hai đường thẳng vuông góc . . . . . . . . . . . . . . . . . . . . . . . . . . . . . . . . . . . . . . . . . . . 205 Vấn đề 1 : Tính góc giữa hai vectơ . . . . . . . . . . . . . . . . . . . . . . . . . . . . . . . . . . . . . . . 205 Vấn đề 2 : Tính góc giữa hai đường thẳng a và b . . . . . . . . . . . . . . . . . . . . . . . . . . . . . . . . 206 Vấn đề 3 : Chứng minh hai đường thẳng vuông góc . . . . . . . . . . . . . . . . . . . . . . . . . . . . . . 207 11.3. Đường thẳng vuông góc với mặt phẳng . . . . . . . . . . . . . . . . . . . . . . . . . . . . . . . . . . . . . 207 Vấn đề 1 : Chứng minh đường thẳng a vuông góc với mặt phẳng (P) . . . . . . . . . . . . . . . . . . . . . 207 Vấn đề 2 : Chứng minh hai đường thẳng vuông góc với nhau . . . . . . . . . . . . . . . . . . . . . . . . . 208 Vấn đề 3 : Xác định góc giữa đường thẳng a và mặt phẳng (P) . . . . . . . . . . . . . . . . . . . . . . . . 210 Vấn đề 4 : Dựng mặt phẳng qua điểm M cho trước và vuông góc với một đường thẳng d cho trước . . . . . 211 11.4. Hai mặt phẳng vuông góc . . . . . . . . . . . . . . . . . . . . . . . . . . . . . . . . . . . . . . . . . . . . 213 Vấn đề 1 : Xác định góc giữa hai mặt phẳng . . . . . . . . . . . . . . . . . . . . . . . . . . . . . . . . . . 213 Vấn đề 2 : Chứng minh hai mặt phẳng (P) và (Q) vuông góc . . . . . . . . . . . . . . . . . . . . . . . . . 214 Vấn đề 3 : Chứng minh đường thẳng a vuông góc với mặt phẳng (P) . . . . . . . . . . . . . . . . . . . . . 215 Vấn đề 4 : Dựng mặt phẳng (Q) chứa a và vuông góc với (P) (giả thiết a không vuông góc với (P)) . . . . . 216 11.5. Khoảng cách . . . . . . . . . . . . . . . . . . . . . . . . . . . . . . . . . . . . . . . . . . . . . . . . . . 217 Vấn đề 1 : Tính khoảng cách từ điểm M đến đường thẳng ∆ cho trước . . . . . . . . . . . . . . . . . . . . 217 Vấn đề 2 : Dựng đường thẳng đi qua một điểm A cho trước và vuông góc với một mặt phẳng (P) cho trước. Khoảng cách từ điểm A đến mặt phẳng (P) . . . . . . . . . . . . . . . . . . . . . . . . . . . . . . 217 Vấn đề 3 : Đoạn vuông góc chung và khoảng cách giữa hai đường thẳng chéo nhau . . . . . . . . . . . . . 219 11.6. Khối đa diện và thể tích khối đa diện . . . . . . . . . . . . . . . . . . . . . . . . . . . . . . . . . . . . . . 222 Vấn đề 1 : Phương pháp trực tiếp tìm thể tích khối chóp . . . . . . . . . . . . . . . . . . . . . . . . . . . . 222 Vấn đề 2 : Tính thể tích hình chóp một cách gián tiếp . . . . . . . . . . . . . . . . . . . . . . . . . . . . . 227 Vấn đề 3 : Dùng công thức thể tích để giải một số bài toán hình học . . . . . . . . . . . . . . . . . . . . . 228 11.7. Phân loại một số hình khối đa diện . . . . . . . . . . . . . . . . . . . . . . . . . . . . . . . . . . . . . . . 230 11.7.1. Hình chóp có cạnh bên vuông góc với đáy . . . . . . . . . . . . . . . . . . . . . . . . . . . . . . . 230 11.7.2. Hình chóp đều . . . . . . . . . . . . . . . . . . . . . . . . . . . . . . . . . . . . . . . . . . . . . 231 11.7.3. Hình chóp có mặt bên vuông góc với đáy . . . . . . . . . . . . . . . . . . . . . . . . . . . . . . . 232 11.7.4. Hình chóp có hai mặt vuông góc với đáy . . . . . . . . . . . . . . . . . . . . . . . . . . . . . . . . 233 11.7.5. Hình chóp có các cạnh bên bằng nhau hoặc các cạnh bên cùng tạo với đáy những góc bằng nhau . . 233 11.7.6. Hình hộp - Hình lăng trụ . . . . . . . . . . . . . . . . . . . . . . . . . . . . . . . . . . . . . . . . 234 11.8. Bài tập tổng hợp . . . . . . . . . . . . . . . . . . . . . . . . . . . . . . . . . . . . . . . . . . . . . . . . . 235 Chương 12 Mặt cầu và khối tròn xoay 239 12.1. Mặt cầu, khối cầu . . . . . . . . . . . . . . . . . . . . . . . . . . . . . . . . . . . . . . . . . . . . . . . . 239 12.2. Mặt tròn xoay. Mặt trụ, hình trụ và khối trụ . . . . . . . . . . . . . . . . . . . . . . . . . . . . . . . . . . 243
  • 8. Chương 13 Phương pháp không gian toạ độ trong không gian 249 13.1. Hệ toạ độ trong không gian . . . . . . . . . . . . . . . . . . . . . . . . . . . . . . . . . . . . . . . . . . . 249 Vấn đề 1 : Tìm tọa độ của một vectơ và các yếu tố liên quan đến vectơ thỏa mãn một số điều kiện cho trước . 249 Vấn đề 2 : Ứng dụng của tích vô hướng và tích có hướng . . . . . . . . . . . . . . . . . . . . . . . . . . . 249 Vấn đề 3 : Lập phương trình của mặt cầu . . . . . . . . . . . . . . . . . . . . . . . . . . . . . . . . . . . 252 Vấn đề 4 : Phương pháp tọa độ giải hình học không gian . . . . . . . . . . . . . . . . . . . . . . . . . . . 253 13.2. Phương trình mặt phẳng . . . . . . . . . . . . . . . . . . . . . . . . . . . . . . . . . . . . . . . . . . . . . 254 Vấn đề 1 : Viết phương trình mặt phẳng đi qua một điểm và có một vectơ pháp tuyến cho trước . . . . . . . 254 Vấn đề 2 : Vị trí tương đối của hai mặt phẳng . . . . . . . . . . . . . . . . . . . . . . . . . . . . . . . . . 255 Vấn đề 3 : Khoảng cách từ một điểm đến mặt phẳng . . . . . . . . . . . . . . . . . . . . . . . . . . . . . . 256 Vấn đề 4 : Góc giữa hai mặt phẳng . . . . . . . . . . . . . . . . . . . . . . . . . . . . . . . . . . . . . . . 258 Vấn đề 5 : Vị trí tương đối giữa mặt phẳng và mặt cầu . . . . . . . . . . . . . . . . . . . . . . . . . . . . . 258 13.3. Phương trình đường thẳng . . . . . . . . . . . . . . . . . . . . . . . . . . . . . . . . . . . . . . . . . . . 260 Vấn đề 1 : Phương trình tham số và phương trình chính tắc của đường thẳng . . . . . . . . . . . . . . . . . 260 Vấn đề 2 : Tìm điểm trên đường thẳng thỏa mãn điều kiện cho trước . . . . . . . . . . . . . . . . . . . . . 260 Vấn đề 3 : Vị trí tương đối của hai đường thẳng ∆ và ∆′ trong không gian . . . . . . . . . . . . . . . . . . 261 Vấn đề 4 : Vị trí tương đối giữa đường thẳng ∆ và mặt phẳng (P) . . . . . . . . . . . . . . . . . . . . . . . 262 Vấn đề 5 : Khoảng cách từ một điểm đến đường thẳng . . . . . . . . . . . . . . . . . . . . . . . . . . . . . 263 Vấn đề 6 : Vị trí tương đối giữa đường thẳng và mặt cầu . . . . . . . . . . . . . . . . . . . . . . . . . . . . 264 Vấn đề 7 : Góc giữa hai đường thẳng ; góc giữa đường thẳng và mặt phẳng . . . . . . . . . . . . . . . . . 266 Vấn đề 8 : Phương trình đường thẳng biết đường thẳng đó song song, hoặc vuông góc với đường thẳng hoặc mặt phẳng khác, hoặc nằm trên mặt phẳng khác . . . . . . . . . . . . . . . . . . . . . . . . . 267 Vấn đề 9 : Phương trình đường thẳng ∆ biết ∆ cắt ∆′ . . . . . . . . . . . . . . . . . . . . . . . . . . . . . 268 Vấn đề 10 : Hình chiếu và tính đối xứng . . . . . . . . . . . . . . . . . . . . . . . . . . . . . . . . . . . . 270 Vấn đề 11 : Bài toán cực trị . . . . . . . . . . . . . . . . . . . . . . . . . . . . . . . . . . . . . . . . . . . 271 13.4. Hình học không gian trong các kì thi tuyển sinh ĐH . . . . . . . . . . . . . . . . . . . . . . . . . . . . . . 273 13.5. Bài tập tổng hợp . . . . . . . . . . . . . . . . . . . . . . . . . . . . . . . . . . . . . . . . . . . . . . . . . 278 III Hướng dẫn và đáp số 287 8
  • 9. Phần I Đại số - Lượng giác - Giải tích 9 www.VNMATH.com www.VNMATH.com
  • 11. Chương 1 Phương trình, bất phương trình, hệ đại số 1.1 Phương trình, bất phương trình đa thức 1.1.1 Phương trình, bất phương trình bậc hai Bài 1.1 : Giải và biện luận các phương trình sau : 1. (m − 2)x2 − 2mx + m + 1 = 0 ; 2. a x − 1 + 1 x − a = 2. Bài 1.2 : Cho phương trình : (m2 − 4)x2 + 2(m + 2)x + 1 = 0. 1. Tìm m để phương trình có hai nghiệm phân biệt. 2. Tìm m để phương trình có nghiệm duy nhất. Bài 1.3 : Gọi a, b, c là độ dài ba cạnh của một tam giác. Chứng minh phương trình sau vô nghiệm : c2 x2 + (a2 − b2 − c2 )x + b2 = 0. Bài 1.4 : Cho phương trình : x2 − (2m + 3)x + m2 + 2m + 2 = 0. 1. Tìm m để phương trình có hai nghiệm x1, x2. 2. Viết phương trình bậc hai có hai nghiệm 1 x1 , 1 x2 . 3. Tìm hệ thức giữa x1, x2 độc lập với tham số m. 4. Tìm m để phương trình có hai nghiệm x1, x2 thỏa mãn x1 = 2x2. Bài 1.5 : Cho phương trình : x2 − cos a.x + sin a − 1 = 0. 1. Chứng minh rằng phương trình luôn có hai nghiệm x1, x2 với mọi a. 2. Tìm hệ thức giữa x1, x2 độc lập với a. 3. Tìm giá trị lớn nhất, giá trị nhỏ nhất của E = (x1 + x2)2 + x2 1x2 2. Bài 1.6 : Cho phương trình : mx2 − 2(m − 2)x + m − 3 = 0. Tìm m để phương trình có : 11
  • 12. WWW.VNMATH.COM CHUYÊN ĐỀ LUYỆN THI ĐẠI HỌC 1. hai nghiệm trái dấu ; 2. hai nghiệm dương phân biệt ; 3. đúng một nghiệm âm. Bài 1.7 : Giải các bất phương trình sau : 1. x2 − 4x + 3 3 − 2x < 1 − x ; 2. (−x2 + 3x − 2)(x2 − 5x + 6) ≥ 0 ; 3. 2 x + 2 + 1 2 ≤ −4 x2 + 2x ; 4. x2 + (x + 1)2 ≤ 15 x2 + x + 1 ; Bài 1.8 : Giải và biện luận các bất phương trình sau : 1. x2 − mx + m + 3 > 0 ; 2. (m + 1)x2 − 2(m − 1)x + 3m − 3 ≥ 0 ; Bài 1.9 : Giải hệ bất phương trình sau : x2 − 7x + 6 ≤ 0 x2 − 8x + 15 ≥ 0 Bài 1.10 : Tìm m để : 1. x2 − mx + m + 3 ≥ 0, ∀x ∈ R ; 2. mx2 + 4x + m > 0, ∀x ∈ R ; 3. mx2 − mx − 5 < 0, ∀x ∈ R. Bài 1.11 : Tìm m để các hàm số sau xác định với mọi x ∈ R : 1. y = m(m + 2)x2 + 2mx + 2 ; 2. y = 1 (1 − m)x2 − 2mx + 5 − 9m ; Bài 1.12 : Cho f(x) = (m + 1)x2 − 2(m − 1)x + 3m − 3. Tìm m để bất phương trình : 1. f(x) < 0 vô nghiệm. 2. f(x) ≥ 0 có nghiệm. Bài 1.13 : Tìm m để các bất phương trình sau có tập nghiệm là R : 1. 1 ≤ 3x2 − mx + 5 2x2 − x + 1 < 6 ; 2. ¬ ¬ ¬ ¬ ¬ x2 + mx + 1 x2 + 1 ¬ ¬ ¬ ¬ ¬ < 2 ; Bài 1.14 : Cho bất phương trình : x2 + 6x + 7 + m ≤ 0. Tìm m để bất phương trình : 1. vô nghiệm. 2. có đúng một nghiệm. 3. có miền nghiệm là một đoạn trên trục số có độ dài bằng 1. Bài 1.15 : Tìm m để f(x) = mx2 − 4x + 3m + 1 > 0 với mọi x > 0. Bài 1.16 : Tìm m để f(x) = 2x2 + mx + 3 ≥ 0 với mọi x ∈ [−1; 1]. Bài 1.17 : Tìm m để f(x) = x2 − 2mx − m ≥ 0 với mọi x > 0. Bài 1.18 : Tìm m để f(x) = mx2 − 2(m + 1)x − m + 5 > 0 với mọi x < 1. Bài 1.19 : Tìm m để f(x) = 2x2 − (3m + 1)x − (3m + 9) ≤ 0 với mọi x ∈ [−2; 1]. TRẦN ANH TUẤN - 0974 396 391 - (04) 66 515 343 Trang 12 www.VNMATH.com www.VNMATH.com
  • 13. WWW.VNMATH.COM CHUYÊN ĐỀ LUYỆN THI ĐẠI HỌC 1.1.2 Phương trình trình bậc ba Bài 1.20 : Cho phương trình : x3 − (m2 − m + 7)x − (3m2 + m − 6) = 0. 1. Tìm m để phương trình có một nghiệm là −1. 2. Với m > 0 tìm được ở câu trên, hãy giải phương trình . Bài 1.21 : Giải các phương trình sau : 1. x3 − 6x2 + 11x − 6 = 0 ; 2. 2x3 + x + 3 = 0 ; 3. x3 − 5x2 + 7x − 2 = 0 ; 4. x3 − 3 √ 3x2 + 7x − √ 3 = 0 ; Bài 1.22 : Tìm m để các phương trình sau có ba nghiệm phân biệt : 1. x3 − (2m + 1)x2 + 3(m + 4)x − m − 12 = 0 ; 2. mx3 − 2mx2 − (2m − 1)x + m + 1 = 0 ; Bài 1.23 : Tìm m để phương trình : mx3 − (3m − 4)x2 + (3m − 7)x − m + 3 = 0 có ba nghiệm dương phân biệt. 1.1.3 Phương trình, bất phương trình bậc bốn Bài 1.24 : Giải các phương trình sau : 1. x4 − 3x2 + 4 = 0 ; 2. (x − 1)(x + 5)(x − 3)(x + 7) = 297 ; 3. (x + 2)(x − 3)(x + 1)(x + 6) = −36 ; 4. x4 + (x − 1)4 = 97 ; 5. (x + 3)4 + (x + 5)4 = 16 ; 6. 6x4 − 35x3 + 62x2 − 35 + 6 = 0 ; 7. x4 + x3 − 4x2 + x + 1 = 0 ; 8. x4 − 5x3 + 10x2 − 10x + 4 = 0 ; 9. x4 − x2 + 6x − 9 = 0 ; 10. 2x4 − x3 − 15x2 − x + 3 = 0. Bài 1.25 : Tìm các giá trị của m sao cho phương trình x4 + (1 − 2m)x2 + m2 − 1 = 0. 1. Vô nghiệm ; 2. Có hai nghiệm phân biệt ; 3. Có bốn nghiệm phân biệt. Bài 1.26 : Tìm các giá trị của a sao cho phương trình (a − 1)x4 − ax2 + a2 − 1 = 0 có ba nghiệm phân biệt. Bài 1.27 : Cho phương trình : (m − 1)x4 + 2(m − 3)x2 + m + 3 = 0. Tìm m để phương trình trên vô nghiệm. TRẦN ANH TUẤN - 0974 396 391 - (04) 66 515 343 Trang 13 www.VNMATH.com www.VNMATH.com
  • 14. WWW.VNMATH.COM CHUYÊN ĐỀ LUYỆN THI ĐẠI HỌC Bài 1.28 : Cho phương trình : x4 − (2m + 1)x2 + m + 3 = 0. Tìm m để phương trình trên có bốn nghiệm phân biệt, trong đó một nghiệm bé hơn −2 và ba nghiệm còn lại lớn hơn −1. Bài 1.29 : Tìm h để phương trình sau đây có không ít hơn hai nghiệm âm khác nhau : x4 + hx3 + x2 + hx + 1 = 0. Bài 1.30 : Cho phương trình : (x + 1)(x + 2)(x + 3)(x + 4) = m. Tìm m để phương trình có bốn nghiệm phân biệt. 1.2 Phương trình, bất phương trình chứa giá trị tuyệt đối 1. Phương trình (bất phương trình) | f(x)| + g(x) < 0 (hoặc = , hoặc > , hoặc ≥ , hoặc ≤ ) tương đương với f(x) ≥ 0 f(x) + g(x) < 0 hoặc f(x) < 0 − f(x) + g(x) < 0. Một số phương trình hoặc bất phương trình chứa nhiều hơn một dấu giá trị tuyệt đối thì việc phá dấu giá trị tuyệt đối sẽ phức tạp hơn nhiều, phải chia thành nhiều trường hợp bằng cách lập bảng xét dấu các biểu thức trong dấu giá trị tuyệt đối. 2. Phương trình (bất phương trình) | f(x)| < |g(x)| (hoặc = , hoặc > , hoặc ≥ , hoặc ≤ ) phương pháp đơn giản là bình phương hai vế, chuyển vế, phân tích thành nhân tử. 3. Một số phương trình và bất phương trình thông dụng (giả sử a > 0). • |x| = a ⇔ x = a hoặc x = −a. • |x| < a ⇔ −a < x < a. • |x| ≤ a ⇔ −a ≤ x ≤ a. • |x| > a ⇔ x < −a hoặc x > a. • |x| ≥ a ⇔ x ≤ −a hoặc x ≥ a. Bài 1.31 : Giải phương trình |x2 − 8x + 15| = x − 3. Bài 1.32 : Giải các phương trình và bất phương trình sau : 1. |x2 − 5x + 4| = x2 + 6x + 5; 2. |x − 1| = 2x − 1; 3. | − x2 + x − 1| ≤ 2x + 5; 4. |x2 − x| ≤ |x2 − 1|. Bài 1.33 : Giải các phương trình và bất phương trình sau : 1. ¬ ¬ ¬ ¬ ¬ x2 − 2 x + 1 ¬ ¬ ¬ ¬ ¬ = 2; 2. ¬ ¬ ¬ ¬ 3x + 4 x − 2 ¬ ¬ ¬ ¬ ≤ 3; 3. ¬ ¬ ¬ ¬ 2x − 3 x − 3 ¬ ¬ ¬ ¬ ≥ 1; 4. |2x + 3| = |4 − 3x|. Bài 1.34 : Giải các bất phương trình sau : TRẦN ANH TUẤN - 0974 396 391 - (04) 66 515 343 Trang 14 www.VNMATH.com www.VNMATH.com
  • 15. WWW.VNMATH.COM CHUYÊN ĐỀ LUYỆN THI ĐẠI HỌC 1. |x2 − 5x + 4| ≤ x2 + 6x + 5; 2. 4x2 + 4x − |2x + 1| ≥ 5. Bài 1.35 : Giải các bất phương trình sau : 1. ¬ ¬ ¬ ¬1 − |x| 1 + |x| ¬ ¬ ¬ ¬ ≥ 1 2 ; 2. log5 log¹⁄₂ x2 − 4|x| |x| − 7 ≤ 0 ; 3. |x2 − 2x − 8| > 2x ; 4. |x3 − 7x − 3| < x3 + x2 + 3 ; 5. |x3 − x2 + 4| + x3 − x2 − 2x − 2 ≤ 0 ; 6. ||x| − 1| < 1 − x ; 7. ¬ ¬|x2 − 3x − 7| + 2x − 1 ¬ ¬ < x2 − 8x − 5 ; 8. ¬ ¬x2 − |x2 − 3x − 5| − 5 ¬ ¬ < x + 1 ; 9. |x − 1| + |x − 2| > 3 + x ; 10. log3 |x2 − 4x| + 3 x2 + |x − 5| ≥ 0 ; 11. ||3x + 4x − 9| − 8| ≤ 3x − 4x − 1 ; Bài 1.36 : Giải các bất phương trình sau : 1. |3x + 2| + |2x − 3| < 11 ; 2. |x2 − 3x − 7| + |2x2 − x − 9| + |3x2 − 7x − 5| < x + 15 ; 3. |x − 1| + |2 − x| > 3 + x ; 4. |x2 − 3x − 17| − |x2 − 5x − 7| > 3. Bài 1.37 : Tìm m để bất phương trình : x2 + |x + m| < 2 có ít nhất một nghiệm âm. Bài 1.38 : Giải và biện luận bất phương trình sau theo tham số p : 2|x − p| + 5|x − 3p| + 4x + 6p + 12 ≤ 0. Bài 1.39 : Giải và biện luận bất phương trình sau theo tham số p : |2x + 21p| − 2|2x − 21p| < x − 21p. Bài 1.40 : Tìm tất cả các giá trị thực của tham số a sao cho bất phương trình x2 − |x − a| − |x − 1| + 3 ≥ 0 đúng với mọi x ∈ R. Bài 1.41 : Tìm tất cả các giá trị của a sao cho giá trị nhỏ nhất của hàm số y = x2 + 2x − 1 + |x − a| lớn hơn 2. Bài 1.42 : Tìm tất cả các giá trị của a sao cho giá trị nhỏ nhất của hàm số y = x2 + |x − a| + |x − 1| lớn hơn 2. Bài 1.43 : Tìm tất cả các giá trị của a sao cho giá trị nhỏ nhất của hàm số y = ax + |x2 − 4x + 3| lớn hơn 1. Bài 1.44 : Tìm tất cả các giá trị của a sao cho giá trị lớn nhất của hàm số y = 4x − x2 + |x − m| nhỏ hơn 4. TRẦN ANH TUẤN - 0974 396 391 - (04) 66 515 343 Trang 15 www.VNMATH.com www.VNMATH.com
  • 16. WWW.VNMATH.COM CHUYÊN ĐỀ LUYỆN THI ĐẠI HỌC 1.3 Phương trình, bất phương trình chứa căn Vấn đề 1 : Phương trình, bất phương trình cơ bản Phương pháp chung là tìm cách bình phương hai vế (để giảm số căn, hoặc mất căn) với điều kiện là hai vế của phương trình phải không âm. 1. Phương trình √ f(x) = √ g(x) ⇔ f(x) ≥ 0 (hoặc cũng có thể xét g(x) ≥ 0) f(x) = g(x). 2. Phương trình √ f(x) = g(x) ⇔ g(x) ≥ 0 f(x) = (g(x))2 . 3. Bất phương trình √ f(x) > √ g(x) (hoặc ≥ ) tương đương với g(x) ≥ 0 f(x) > g(x). 4. Bất phương trình √ f(x) < g(x) (hoặc ≤ ) tương đương với f(x) ≥ 0 g(x) ≥ 0 f(x) < (g(x))2 . 5. Bất phương trình √ f(x) > g(x) (hoặc ≥ ) tương đương với (I) f(x) ≥ 0 g(x) < 0 hoặc (II) g(x) ≥ 0 f(x) > (g(x))2 . Bài 1.45 : Giải phương trình √ x2 + 56x + 80 = x + 20. Bài 1.46 : Giải bất phương trình √ x2 − 2x − 15 < x − 3. Bài 1.47 : Giải bất phương trình √ x2 − 1 > x + 2. Bài 1.48 : Giải các phương trình sau : 1. √ 2x2 + 4x − 1 = x + 1; 2. √ 4x2 + 101x + 64 = 2(x + 10); 3. √ x2 + 2x = −2x2 − 4x + 3; 4. √ (x + 1)(x + 2) = x2 + 3x − 4. Bài 1.49 : Giải các bất phương trình: 1. √ x2 + x − 6 < x − 1; 2. √ 2x − 1 ≤ 2x − 3; 3. √ 2x2 − 1 > 1 − x; 4. √ x2 − 5x − 14 ≥ 2x − 1. Bài 1.50 : Tìm tập xác định của mỗi hàm số sau : TRẦN ANH TUẤN - 0974 396 391 - (04) 66 515 343 Trang 16 www.VNMATH.com www.VNMATH.com
  • 17. WWW.VNMATH.COM CHUYÊN ĐỀ LUYỆN THI ĐẠI HỌC 1. y = ¬ ¬x2 + 3x − 4 ¬ ¬ − x + 8; 2. y = x2 + x + 1 |2x − 1| − x − 2 ; 3. y = Ö 1 x2 − 7x + 5 − 1 x2 + 2x + 5 ; 4. y = √ x2 − 5x − 14 − x + 3. Bài 1.51 : Giải các phương trình sau : 1. √ 5x2 − 6x − 4 = 2(x − 1); 2. √ x2 + 3x + 12 = x2 + 3x. Bài 1.52 : Giải các bất phương trình sau : 1. √ x2 + 6x + 8 ≤ 2x + 3; 2. 2x − 4 √ x2 − 3x − 10 > 1; 3. 6 √ (x − 2)(x − 3) ≤ x2 − 34x + 48 ; 4. √ x2 − x − 12 ≥ x − 1; 5. √ x2 − 4x − 12 > 2x + 3; 6. √ x + 5 1 − x < 1. Vấn đề 2 : Phương pháp đặt ẩn phụ Chúng ta thường sử dụng một số quy tắc đặt ẩn phụ như sau : 1. Nếu phương trình chứa hai loại căn, có thể (a) Đặt u = n√ ax + b, rút x, thế vào phương trình được phương trình ẩn u. (b) Hoặc cũng có thể đặt u = n √ u(x), v = m √ v(x), lũy thừa để rút ra ràng buộc giữa u và v để được 1 phương trình theo u, v. Kết hợp với phương trình ban đầu, ta được hệ hai ẩn u, v. 2. Đặt u = n √ u(x), lũy thừa hai vế được phương trình chứa u, x. Kết hợp với phương trình ban đầu, ta được hệ hai ẩn u, x.Giải phương trình bậc hai (có ∆ là bình phương một số). 3. Đặt ẩn phụ không hoàn toàn, đặt u = √ u(x), đưa về phương trình bậc hai theo u với x coi như là tham số. 4. Nếu phương trình chứa √ a ± √ b và √ ab ta thường đặt u = √ a ± √ b. 5. phương trình đẳng cấp, chẳng hạn đẳng cấp bậc 2 : A.x2 + B.xy + C.y2 = 0. Có cách giải như sau : (a) Xét y = 0, rút được x; (b) Xét y 0, chia cả hai vế cho y2, đặt u = x y , đưa được về phương trình bậc hai theo u. Bài 1.53 : Giải các phương trình sau : 1. 3x2 + 21x + 18 + 2 √ x2 + 7x + 7 = 2 ; 2. x2 + √ x + 1 = 1 ; 3. 2(x2 + 2) = 5(x3 + 1) ; 4. 2x2 − 3x + 2 = x √ 3x − 2 ; 5. 6x2 − 10x + 5 − (4x − 1) √ 6x2 − 6x + 5 = 0 ; 6. 4 √ 97 − x + 4 √ x = 5 ; TRẦN ANH TUẤN - 0974 396 391 - (04) 66 515 343 Trang 17 www.VNMATH.com www.VNMATH.com
  • 18. WWW.VNMATH.COM CHUYÊN ĐỀ LUYỆN THI ĐẠI HỌC Bài 1.54 : Giải các phương trình sau : 1. √ x + 3 + √ 3x + 1 = 2 √ x + √ 2x + 2 ; 2. √ 2x2 + x + 6 + √ x2 + x + 2 = x + 4 x ; 3. x2 + 2x Ö x − 1 x = 3x + 1 ; 4. 4 √ x + 4√ x + 1 = 2 4√ 2x + 1 ; 5. √ x2 + 4x + 3 + √ x2 + x = √ 3x2 + 4x + 1 ; 6. 3 √ x + √ 5 − x ≤ 3 ; 7. 3√ x − 1 + 3√ x + 1 = x 3√ 2 ; 8. 3 √ x + 3√ x − 16 = 3 √ x − 8 ; 9. 3√ 2x3 − 1 + 3√ 1 − x3 = x ; 10. √ x2 − x + 1 + √ x2 + x + 1 = 2 ; 11. √ 2x2 + x + 9 + √ 2x2 − x + 1 = x + 4. Bài 1.55 : Giải các phương trình sau : 1. √ 1 − x + √ 1 + x + 2 √ 1 − x2 = 4 ; 2. 2x + √ x + 1 + √ x + 2 √ x2 + x = 1 ; 3. x2 + 2x + √ x + 3 + 2x √ x + 3 = 9 ; 4. 2x2 + x + √ x2 + 3 + 2x √ x2 + 3 = 9 ; Bài 1.56 : Giải các phương trình sau : 1. 2x2 + x + 3 = 3x √ x + 3 ; 2. √ x + 8 = 3x2 + 7x + 8 4x + 2 ; 3. √ x2 + x + 2 = 3x2 + 3x + 2 3x + 1 ; 4. x + 2 + x √ 2x + 1 x + √ 2x + 1 = √ x + 2 ; 5. ( √ x + 3 − √ x + 1)(x2 + √ x2 + 4x + 3) = 2x. Bài 1.57 : Giải các phương trình sau : 1. 3 √ x + 1 + 3 √ x + 2 = 1 + 3√ x2 + 3x + 2 ; 2. 3√ x + 1 + 3√ x2 = 3 √ x + 3√ x2 + x ; 3. 4√ x + 1 + √ x = 1 + 4√ x3 + x2 ; 4. √ x + 3 + 2x √ x + 1 = 2x + √ x2 + 4x + 3 ; 5. √ x3 + x2 + 3x + 3 + √ 2x = √ x2 + 3 + √ 2x2 + 2x ; 6. √ x + 3 + 4x √ x + 3 = 4 √ x ; 7. 4 √ x + 3 = 1 + 4x + 3 x ; 8. 2 √ x + 3 = 9x2 − x − 4 ; 9. 12 √ x + 2 √ x − 1 = 3x + 9 ; Bài 1.58 : Giải các phương trình sau : 1. √ x + 3 + 3 √ x = 3 ; 2. 4 √ x + 4 √ x − 1 = 4 √ x + 1 ; 3. √ 2 − x2 = (2 − √ x)2 ; 4. 2x + 1 + x √ x2 + 2 + (x + 1) √ x2 + 2x + 3 = 0 ; 5. x2 √ x + (x − 5)2 √ 5 − x = 11( √ x + √ 5 − x) ; 6. 2x3 = 1 + 3 Ö x + 1 2 ; Bài 1.59 : Giải các phương trình sau : 1. 8 √ 1 − x + 8 √ x = 1 ; 2. 2 √ x + 4 √ 1 − 2x = 1 ; 3. √ x + 4 + √ x + √ 1 − x = 3 ; 4. 2 + √ x 3 + √ 1 − x = √ x + √ 1 − x ; TRẦN ANH TUẤN - 0974 396 391 - (04) 66 515 343 Trang 18 www.VNMATH.com www.VNMATH.com
  • 19. WWW.VNMATH.COM CHUYÊN ĐỀ LUYỆN THI ĐẠI HỌC Vấn đề 3 : Phương pháp nhân liên hợp Dạng 1 : Phương trình dạng √ u(x) ± √ v(x) = f(x), trong đó f(x) và u(x) − v(x) có cùng nghiệm x = x0. (a) Phương trình trở thành u(x) − v(x) √ u(x) ∓ √ v(x) = f(x). (b) Chuyển vế, đặt (x − x0) làm nhân tử chung. Dạng 2 : Phương trình dạng ( n √ u1(x) ± n √ v1(x)) + ( m √ u2(x) ± m √ v2(x)) = f(x), trong đó f(x); u1(x) − v1(x); u2(x) − v2(x) có cùng nghiệm x = x0 (ở đây f(x) có thể đồng nhất bằng 0). Phương pháp giải loại này là chúng ta nhân liên hợp theo từng cụm, đặt (x − x0) làm nhân tử chung. Bài 1.60 : Giải các phương trình, các bất phương trình sau : 1. 3(2 + √ x − 2) = 2x + √ x + 6; 2. x2 1 + √ 1 + x 2 > x − 4; 3. √ x − 2 + √ 4 − x = x2 − 6x + 11; 4. √ x − 2 + √ 4 − x = 2x2 − 5x − 1; 5. Ö 1 − x x = 2x + x2 1 + x2 ; 6. x2 + x − 1 = (x + 2) √ x2 − 2x + 2; 7. 3 √ x + 24 + √ 12 − x = 6; 8. 2 √ x2 − 7x + 10 = x + √ x2 − 12x + 20; 9. 2x2 − 11x + 21 = 3 3 √ 4x − 4; 10. √ 5x − 1 + 3 √ 9 − x = 2x2 + 3x − 1. Bài 1.61 : Giải các phương trình sau : 1. √ x + 4 − √ 2x + 3 = x − 1 ; 2. x + √ 2x = 1 x + Ö x + 1 x ; 3. (x − 1) √ x + 1 + √ 2x + 1 = √ x + 2 ; 4. 1 x2 + √ x + 5 = 1 x + √ 2x + 4 ; 5. 2 + √ x + 6 = √ 2x + 5 + √ x + 3 ; 6. 1 + 4 √ x + 3 = x + √ 2x ; 7. √ x + 2 + √ x + 6 = √ 2x + 5 + √ 2x + 1 ; 8. 4√ x + 8 + √ x + 4 = √ 2x + 3 + √ 3x Vấn đề 4 : Phương pháp đánh giá Cơ sở của phương pháp này là chúng ta sử dụng bất đẳng thức hoặc phương pháp hàm số đế đánh giá. Cách 1 : Cơ sở nhận dạng : (a) Nếu hàm số y = f(x) đồng biến trên (a; b) và hàm số y = g(x) nghịch biến trên (a; b) thì phương trình f(x) = g(x) nếu có nghiệm thì nghiệm đó là duy nhất. (b) Nếu hàm số y = f(x) đồng biến (hoặc nghịch biến) trên (a; b) thì phương trình f(x) = c (với c là hằng số) nếu có nghiệm thì nghiệm đó là duy nhất. TRẦN ANH TUẤN - 0974 396 391 - (04) 66 515 343 Trang 19 www.VNMATH.com www.VNMATH.com
  • 20. WWW.VNMATH.COM CHUYÊN ĐỀ LUYỆN THI ĐẠI HỌC Phương pháp giải là : (a) Nhận thấy x = x0 là một nghiệm của phương trình đã cho. (b) Nếu x > x0, ta suy ra vế trái lớn hơn vế phải hoặc ngược lại. (c) Nếu x < x0, ta suy ra vế trái lớn hơn vế phải hoặc ngược lại. (d) Kết luận phương trình đã cho có nghiệm duy nhất x = x0. Cách 2 : Nếu hàm số y = f(x) đồng biến (hoặc nghịch biến) trên (a; b) thì phương trình f(u) = f(v) tương đương với u = v. Cách 3 : Nếu hàm số y = f(x) thỏa mãn f′(x) = 0 có nhiều hơn 1 nghiệm thì chúng ta lập bảng biến thiên để suy ra phương trình có tối ta bao nhiêu nghiệm, rồi nhẩm đủ số nghiệm đó, dẫn đến đó là tất cả các nghiệm của phương trình. Cách 4 : Nếu f(x) ≥ c và g(x) ≤ c thì phương trình f(x) = g(x) tương đương với f(x) = c g(x) = c. Bài 1.62 : Giải các phương trình sau : 1. √ x + 3 + 3 √ x = 3 ; 2. √ x + 3 + x + √ x + 8 = 4 ; 3. √ x2 − x + 1 + √ x2 + 7x + 1 = 4 √ x ; 4. √ x + 3 1 + √ 2 − x + √ 2x − 1 = 2 ; 5. √ x2 − x + 4 + √ 2x − 1 = 5 ; Vấn đề 5 : Phương trình, bất phương trình có tham số 1. Sử dụng phương trình, bất phương trình cơ bản; 2. Sử dụng đặt ẩn phụ, và đặt điều kiện "chặt" cho ẩn; 3. Sử dụng điều kiện có nghiệm của phương trình bậc hai; 4. Sử dụng phương pháp hàm số để chỉ ra điều kiện có nghiệm. Bài 1.63 : Tìm điều kiện của m để phương trình √ x2 + 2x − m = 2x − 1 : 1. có nghiệm thực ; 2. có đúng một nghiệm thực ; 3. có hai nghiệm thực phân biệt. Bài 1.64 : Tìm điều kiện của m để phương trình x + x + 1 2 + Ö x + 1 4 = m có nghiệm thực. Bài 1.65 : Tìm điều kiện của m để phương trình √ 16 − x2 − m √ 16 − x2 − 4 = 0 có nghiệm thực. TRẦN ANH TUẤN - 0974 396 391 - (04) 66 515 343 Trang 20 www.VNMATH.com www.VNMATH.com
  • 21. WWW.VNMATH.COM CHUYÊN ĐỀ LUYỆN THI ĐẠI HỌC Bài 1.66 : Tìm điều kiện của m để phương trình Ö x − 1 x + 2 − m Ö x + 2 x − 1 + 2 = 0 có nghiệm thực. Bài 1.67 : Tìm điều kiện của m để phương trình √ x + 1 − m √ x − 1 + 2 4√ x2 − 1 = 0 có nghiệm thực. Bài 1.68 : Tìm điều kiện của m để phương trình √ x2 − 2x − 3 = x + m TRẦN ANH TUẤN - 0974 396 391 - (04) 66 515 343 Trang 21 www.VNMATH.com www.VNMATH.com
  • 22. WWW.VNMATH.COM CHUYÊN ĐỀ LUYỆN THI ĐẠI HỌC 1. có nghiệm thực ; 2. có hai nghiệm thực phân biệt. Bài 1.69 : Biện luận theo m số nghiệm thực của phương trình √ x + 1 + √ 1 − x = m. Bài 1.70 : Tìm điều kiện m để phương trình √ x + √ 9 − x = √ −x2 + 9x + m có nghiệm thực. Bài 1.71 : Tìm điều kiện m để phương trình x + 4 √ x − 4 + x + √ x − 4 = m có nghiệm thực. Bài 1.72 : Tìm điều kiện m để phương trình x + 6 √ x − 9 + x − 6 √ x − 9 = x + m 6 có nghiệm thực. Bài 1.73 : Tìm m để phương trình √ x4 + 4x + m + 4√ x4 + 4x + m = 6 có nghiệm thực. Bài 1.74 : Tìm điều kiện của m để phương trình √ 1 − x2 + 2 3√ 1 − x2 = m : 1. có nghiệm thực duy nhất ; 2. có nghiệm thực. Bài 1.75 : Chứng tỏ rằng phương trình 3x2 − 1 √ 2x − 1 = √ 2x − 1 + mx luôn có nghiệm thực với mọi giá trị của m. Bài 1.76 : Tìm m để phương trình (x − 3)(x + 1) + 4(x − 3) Ö x + 1 x − 3 = m có nghiệm thực. Bài 1.77 : Tìm m để phương trình 3√ 1 − x + 3√ 1 + x = m có nghiệm thực. Bài 1.78 : Biện luận theo m số nghiệm thực của phương trình m √ x2 + 2 = x + m. Bài 1.79 : Tìm m để phương trình √ x2 − 2x − 3 = mx + m có nghiệm thực x −1. Bài 1.80 : Tìm m để phương trình sau có nghiệm thực : √ x + √ 1 − x + 2m x(1 − x) − 2 4 x(1 − x) = m. Bài 1.81 : Tìm m để phương trình x + √ x2 − x + 1 = m có nghiệm thực. Bài 1.82 : Tìm m để các phương trình sau có nghiệm thực : 1. √ x2 + x + 1 − √ x2 − x + 1 = m ; 2. 4√ x2 + 1 − √ x = m. Bài 1.83 : Tìm m để phương trình sau có nghiệm thực : x √ x + √ x + 12 = m √ 5 − x + √ 4 − x . Bài 1.84 : Tìm m để phương trình sau có nghiệm thực : m √ x − 2 + 2 4√ x2 − 4 − √ x + 2 = 2 4√ x2 − 4. Bài 1.85 : Tìm m để phương trình sau có nghiệm thực : (4m − 3) √ x + 3 + (3m − 4) √ 1 − x + m − 1 = 0. Bài 1.86 : Tìm m để phương trình sau có 4 nghiệm thực phân biệt : m √ 1 + x2 − √ 1 − x2 + 2 = 2 √ 1 − x4 + √ 1 + x2 − √ 1 − x2. Bài 1.87 : Tìm m để phương trình sau có nghiệm duy nhất : √ x2 − 2x = √ mx + 1. TRẦN ANH TUẤN - 0974 396 391 - (04) 66 515 343 Trang 22 www.VNMATH.com www.VNMATH.com
  • 23. WWW.VNMATH.COM CHUYÊN ĐỀ LUYỆN THI ĐẠI HỌC Bài 1.88 : Tìm m để phương trình sau có nghiệm : x + √ 1 − x2 = m. Bài 1.89 : Cho phương trình : −x2 + 2x + 4 (3 − x)(x + 1) = m − 3. 1. Tìm m để phương trình có nghiệm. 2. Tìm m để phương trình có đúng hai nghiệm phân biệt. Bài 1.90 : Tìm m để phương trình sau có nghiệm : √ x + 1 + √ 3 − x − (x + 1)(3 − x) = m. Bài 1.91 : Cho phương trình : |x + 1| + m|x − 1| = (m + 1) √ x2 − 1. 1. Giải phương trình khi m = 2 ; 2. Tìm m để phương trình trên có nghiệm. Bài 1.92 : Tìm m để các bất phương trình sau có nghiệm : 1. √ 4 − x + √ x + 5 ≥ m ; 2. mx − √ x − 3 ≤ m + 1. Bài 1.93 : Tìm m để bất phương trình m √ x2 − 2x + 2 + 1 + x(2 − x) ≤ 0 có nghiệm trong đoạn ä 0; 1 + √ 3 ç . Bài 1.94 : Tìm m để bất phương trình √ (4 + x)(6 − x) ≤ x2 − 2x + m nghiệm đúng với mọi x ∈ [−4; 6]. 1.4 Hệ phương trình 1.4.1 Phương pháp thế Bài 1.95 : Giải các hệ phương trình sau : 1. x2(y + 1)(x + y + 1) = 3x2 − 4x + 1 xy + x + 1 = x2 2. x3y = 16 3x + y = 8 3. y(1 + x2) = x(1 + y2) x2 + 3y2 = 1 4. x − 1 x = y − 1 y 2y = x3 + 1 5. √ x + y = 3 √ x + y √ x − y = 3 √ x − y − 12 6. √ x + y − √ x − y = 2 x2 + y2 + x2 − y2 = 4 TRẦN ANH TUẤN - 0974 396 391 - (04) 66 515 343 Trang 23 www.VNMATH.com www.VNMATH.com
  • 24. WWW.VNMATH.COM CHUYÊN ĐỀ LUYỆN THI ĐẠI HỌC 7. x3 − 8x = y3 + 2y x2 − 3 = 3(y2 + 1) 8. |x2 − 2x| + y = 1 x2 + |y| = 1 9. x2 + y2 + 2xy x + y = 1 √ x + y = x2 − y 10. √ 7x + y + √ 2x + y = 5 √ 2x + y + x − y = 2 11. √ 3x 1 + 1 x + y = 2 √ 7y 1 − 1 x + y = 4 √ 2 12. x3 + 3xy2 = −49 x2 − 8xy + y2 = 8y − 17x 13. √ y( √ x + √ x + 3) = 3 √ x + √ y = x + 1 14. √ x + 1 + 1 y = Ö x y √ xy + √ y + 1 + √ 1 − x = 1 1.4.2 Phương pháp phân tích thành nhân tử hoặc coi một phương trình là phương trình bậc hai (ba) theo một ẩn Bài 1.96 : Giải các hệ phương trình sau : 1. x3 + 3y2x = 4 y3 + 3x2y = 4; 2. x2 + y + 1 = 0 x + y2 + 1 = 0; 3. 3x3 = x2 + 2y2 3y3 = y2 + 2x2; 4. x3 = 3x + 8y y3 = 3y + 8x; 5. x − 3y = 4y x y − 3x = 4x y ; 6. x3 = 5x + y y3 = 5y + x. Bài 1.97 : Giải các hệ phương trình sau : 1. x2 = 3x + 2y y2 = 3y + 2x 2. x2 − 2y2 = 2x + y y2 − 2x2 = 2y + x 3. x3 = 2x + y y3 = 2y + x 4. xy + x + y = x2 − 2y2 x √ 2y − y √ x − 1 = 2x − 2y 5. y2 = (5x + 4)(4 − x) y2 − 5x2 − 4xy + 16x − 8y + 16 = 0 6. x3 + 1 = 2y y3 + 1 = 2x 7. √ x + y + √ x − y = 1 + x2 − y2 √ x + √ y = 1 8. x2y + 2x + 3y = 6 3xy + x + y = 5 1.4.3 Phương pháp đặt ẩn phụ Bài 1.98 : Giải các hệ phương trình sau : 1. x + xy + y = 11 x − xy + y = 1; TRẦN ANH TUẤN - 0974 396 391 - (04) 66 515 343 Trang 24 www.VNMATH.com www.VNMATH.com
  • 25. WWW.VNMATH.COM CHUYÊN ĐỀ LUYỆN THI ĐẠI HỌC 2. x2y + xy2 = 20 1 x + 1 y = 5 4 ; 3. x2 + y2 = 2x2y2 x + y + 1 = 3xy; 4. x − y + xy = 1 x2 + y2 = 2; 5. x2 + y2 + x2y2 = 1 + 2xy (x − y)(1 + xy) = 1 − xy; 6. x y + y x = 26 5 x2 − y2 = 24; 7. x2 + y2 + xy = 3 xy3 + yx3 = 2; 8. x y + y x = 2 1 x + 1 y + x + y = 4; 9. x + y + x y + y x = 4 x + y + x2 y + y2 x = 4; 10. x + y + x2y2 = 3xy 1 x + 1 y − xy = 1; 11. x2 + y2 + xy = 3x2y2 x2 + y2 − xy = x2y2; 12. x + xy + y = 7 x2 + xy + y2 = 13; Bài 1.99 : Giải các hệ phương trình sau : 1. x + y + xy = 5 x2 + y2 + xy = 7 2. x y + y x = 13 6 x + y = 5 3. x2 + xy + y2 = 1 x − y − xy = 3 4. x2 + 1 + y(x + y) = 4y (x2 + 1)(y + x − 2) = y 5. 4xy + 4(x2 + y2) + 3 (x + y)2 = 7 2x + 1 x + y = 3 6. x2 − 3xy + y2 = −1 3x2 − xy + 3y2 = 13 7. 2x2 − 4xy + y2 = −1 3x2 + 2xy + 2y2 = 7 8. y2 − 3xy = 4 x2 − 4xy + y2 = 1 9. x2 + y2 = 1 √ x + y + √ x − y = 2 10. x2 + xy + y2 = 19(x − y)2 x2 − xy + y2 = 7(x − y) 11. x √ y + y √ x = 30 x √ x + y √ y = 35 12. x2 + y2 + √ 2xy = 8 √ 2 √ x + √ y = 4 Bài 1.100 : Giải các hệ phương trình sau : 1. x2 + x + y + 1 + x + y2 + x + y + 1 + y = 18 x2 + x + y + 1 − x + y2 + x + y + 1 − y = 2; 2. Ö x y + y x = 7 √ xy + 1 x √ xy + y √ xy = 78; 3. x2 + y2 + √ 2xy = 8 √ 2 √ x + √ y = 4; 4. √ x + y + √ x − y = 4 x2 + y2 = 128; 5. √ x + √ y = 1 |x| + |y| = 1; 6. √ x + √ y = 4 √ x + 5 + √ y + 5 = 6; 7. x + y − √ xy = 7 x2 + y2 + xy = 133; 8. (x − y)(x2 − y2) = 7 (x + y)(x2 + y2) = 175; 9. x √ x + y √ y = 2 √ xy √ x + √ y = 2; 10. √ x + y + √ x − y = 1 + x2 − y2 √ x + √ y = 1; TRẦN ANH TUẤN - 0974 396 391 - (04) 66 515 343 Trang 25 www.VNMATH.com www.VNMATH.com
  • 26. WWW.VNMATH.COM CHUYÊN ĐỀ LUYỆN THI ĐẠI HỌC 11. Ö x y + y x = 5 2 x + y = 10 12. x √ y + y √ x = 30 x √ x + y √ y = 35; 13. 2(x + y) = 3 3 x2y + 3 xy2 3 √ x + 3 √ y = 6 14. 6x x + y + x + y 6x = 5 2 x + y − √ xy = 9; 15. Ö x y + y x = 7 2 + √ xy x √ xy + y √ xy = 7; 16. 3 − 5 y + 42x √ 2y = 4 3 + 5 y + 42x √ x = 2 17. 3 + 2x2y − x4y2 + x4(2 − 2x2) = y4 1 + 1 + (x − y)2 = x3(x3 − x + 2y2); 18. √ x + √ y = 10 √ x + 6 + √ y + 6 = 14 19. x + x2 − y2 x − x2 − y2 = 9x 5 x y = 5 + 3x 30 − 6y 20. x + y + x2 − y2 = 12 y x2 − y2 = 12. 21. Ö 20y x = √ x + y + √ x − y 16x 5y = √ x + y − √ x − y Bài 1.101 : Giải các hệ phương trình sau : 1. 2x + x2 − y2 = 3 x2 + y2 = 1 2. x2 + y2 = 1 2 2x3 + 6y2 x = 1 3. x3 + 3y2x = y x2 + 3y2 = 1 4. x − y 1 − xy = 1 − 3x 3 − x x + y 1 + xy = 1 − 2y 2 − y 5. (x + y)(1 + xy) = 18xy (x2 + y2)(1 + x2y2) = 208x2y2 6. (x + y) 1 + 1 xy = 4 xy + 1 xy + x2 + y2 xy = 4 7. y(x2 + 1) = 2x(y2 + 1) (x2 + y2) 1 + 1 x2y2 = 24 8. (x + y) 1 + 1 xy = 5 xy + 1 xy = 4 9. (x + y) 1 + 1 xy = 6 (x2 + y2) 1 + 1 xy 2 = 18 10. (x2 + y2) 1 + 1 xy 2 = 9 (x3 + y3) 1 + 1 xy 3 = 27 11. x y + y x (x + y) = 15 x2 y2 + y2 x2 (x2 + y2) = 85 12. 2x + y + 1 x = 4 x2 + xy + 1 x = 3 13. x2y + 2y + x = 4xy 1 x2 + 1 xy + x y = 3 14. x2y + y = 2 x2 + 1 x2 + x2y2 = 3; 15. x2 + y2 + x + y = 4xy 1 x + 1 y + y x2 + x y2 = 4; 16. 2y(x2 − y2) = 3x x(x2 + y2) = 10y. Bài 1.102 : Giải các hệ phương trình : 1. x2 + y2 − 3x + 4y = 1 3x2 − 2y2 − 9x − 8y = 3 TRẦN ANH TUẤN - 0974 396 391 - (04) 66 515 343 Trang 26 www.VNMATH.com www.VNMATH.com
  • 27. WWW.VNMATH.COM CHUYÊN ĐỀ LUYỆN THI ĐẠI HỌC 2. (x + y) 2 − 1 xy = 9 2 (x − y) 2 + 1 xy = 5 2 3. x − xy − y = 1 x2y − xy2 = 6 4. x(x + 2)(2x + y) = 9 x2 + 4x + y = 6 5. y + xy2 = 6x2 1 + x2y2 = 5x2; 6. 1 + x3y3 = 19x3 y + xy2 = −6x2. Bài 1.103 : Cho hệ phương trình : x + xy + y = a + 1 x2y + xy2 = a. Tìm a để hệ có ít nhất một nghiệm (x; y) thỏa mãn : x > 0 và y > 0. Bài 1.104 : Cho hệ phương trình : √ x + 1 + √ y + 1 = 3 x √ y + 1 + y √ x + 1 + √ y + 1 + √ x + 1 = m. 1. Giải hệ phương trình với m = 6. 2. Tìm m để hệ phương trình trên có nghiệm. 1.4.4 Phương pháp hàm số Bài 1.105 : Giải các hệ phương trình sau : 1. x3 − 5x = y3 − 5y x8 + y4 = 1 2. x + √ x2 − 2x + 2 = 3y−1 + 1 y + y2 − 2y + 2 = 3x−1 + 1 3. x2 = √ y − 1 + 2x − 1 y2 = √ x − 1 + 2y − 1 4. √ x + 1 + √ 7 − y = 4 √ y + 1 + √ 7 − x = 4 5. √ x + √ x + 3 = 3 √ y √ y + √ y + 3 = 3 √ x 6. x3 − 3x = y3 − 3y x6 + y6 = 1 7. ex − ey = x − y log2 x 2 + log√ 2 4y3 = 10 8. ln(1 + x) − ln(1 + y) = x − y 2x2 − 5xy + y2 = 0 9. √ x + √ 2 − y = √ 2 √ y + √ 2 − x = √ 2. Bài 1.106 : Tìm m để hệ phương trình sau có nghiệm : √ x + 1 + √ 3 − y = m √ y + 1 + √ 3 − x = m Bài 1.107 : Chứng minh rằng với mọi m > 0, hệ phương trình sau có nghiệm duy nhất : 3x2y − 2y2 − m = 0 3y2 x − 2x2 − m = 0 1.4.5 Phương pháp đánh giá Bài 1.108 : Giải các hệ phương trình sau : TRẦN ANH TUẤN - 0974 396 391 - (04) 66 515 343 Trang 27 www.VNMATH.com www.VNMATH.com
  • 28. WWW.VNMATH.COM CHUYÊN ĐỀ LUYỆN THI ĐẠI HỌC 1. x + √ x + √ y + 1 = 1 y + √ y + √ x + 1 = 1 2. x + 2xy 3√ x2 − 2x + 9 = x2 + y y + 2xy 3 y2 − 2y + 9 = y2 + x 3. y = −x3 + 3x + 4 x = 2y3 − 6y − 2 4. x + y + 1 x + 1 y = 4 x2 + y2 + 1 x2 + 1 y2 = 4 5. x2 + 2y2 = 3 x2(y2 + 1) = 4 6. x3 − y3 = 7 xy(x − y) = 2 7. 3 √ x + 3 √ y = 1 4 √ x + 4 √ y = 1 8. x + 2 − y2 = 2 y + √ 2 − x2 = 2; 9. √ x + 4√ 32 − x − y2 = −3 4 √ x + √ 32 − x + 6y = 24. 1.5 Số nghiệm của phương trình, hệ phương trình Bài toán : Chứng minh rằng phương trình f(x) = 0 có đúng k nghiệm thực phân biệt trong miền D.1 Vấn đề 1 : Chứng minh phương trình có nghiệm duy nhất Cách 1 : Lập bảng biến thiên của hàm số y = f(x) với x ∈ D (tính đầy đủ các giá trị tại đầu và cuối mũi tên), từ đó suy ra được số nghiệm của phương trình. Cách 2 : Dựa vào hai định lí : Định lí 1 : Nếu hàm số y = f(x) luôn đồng biến hoặc nghịch biến trên (a; b) thì phương trình f(x) = 0 có tối đa một nghiệm trong khoảng (a; b). Định lí 2 : Nếu hàm số y = f(x) liên tục trên [a; b] và f(a). f(b) < 0 thì phương trình f(x) = 0 có ít nhất một nghiệm trong khoảng (a; b). Bài 1.109 : Chứng minh rằng các phương trình sau có nghiệm duy nhất : 1. x5 + x4 + 2x3 + 2x2 + x + 1 = 0; 2. ex(x2 + 1) − 4 = 0. Bài 1.110 : Chứng minh rằng phương trình : x3 + √ x − 1 = 0 có nghiệm duy nhất. Bài 1.111 : Chứng minh rằng phương trình xx+1 = (x + 1)x có một nghiệm dương duy nhất. Bài 1.112 : Chứng minh rằng với mọi a > 0, hệ phương trình sau có nghiệm duy nhất : ex − ey = ln(1 + x) − ln(1 + y) y − x = a Vấn đề 2 : Chứng minh phương trình có đúng hai nghiệm phân biệt 1 Nếu k = 0 tức là phương trình vô nghiệm TRẦN ANH TUẤN - 0974 396 391 - (04) 66 515 343 Trang 28 www.VNMATH.com www.VNMATH.com
  • 29. WWW.VNMATH.COM CHUYÊN ĐỀ LUYỆN THI ĐẠI HỌC Cách 1 : Lập bảng biến thiên của hàm số y = f(x) với x ∈ D (tính đầy đủ các giá trị tại đầu và cuối mũi tên), từ đó suy ra được số nghiệm của phương trình. Cách 2 : Chỉ lập bảng biến thiên nhưng không tính được hết tất cả các đầu mút (lúc này y’=0 có nghiệm duy nhất). Từ đó suy ra được phương trình có tối đa 2 nghiệm. Kết hợp với định lí 1 ta cũng chỉ ra được phương trình có ít nhất 2 nghiệm. Bài 1.113 : Chứng minh rằng các phương trình sau có đúng hai nghiệm thực phân biệt : 1. x4 − x2 − 2x − 1 = 0; 2. x4 − 3x3 − 1 = 0; 3. 3x4 − 4x3 − 6x2 + 12x − 20 = 0; 4. x3 − 2x − √ x + 1 = 0. Vấn đề 3 : Chứng minh phương trình có đúng ba nghiệm phân biệt Cách 1 : Lập bảng biến thiên của hàm số y = f(x) với x ∈ D (tính đầy đủ các giá trị tại đầu và cuối mũi tên), từ đó suy ra được số nghiệm của phương trình. Cách 2 : Chỉ lập bảng biến thiên nhưng không tính được hết tất cả các đầu mút (lúc này y’=0 có đúng 2 nghiệm). Từ đó suy ra được phương trình có tối đa 3 nghiệm. Kết hợp với định lí 1 ta cũng chỉ ra được phương trình có ít nhất 3 nghiệm. Bài 1.114 : Chứng minh rằng các phương trình sau có đúng ba nghiệm thực phân biệt : 1. sin x − x 2 = 0; 2. 4x(4x2 + 1) = 1. 1.6 Phương trình, bất phương trình, hệ đại số trong các kì thi tuyển sinh ĐH Bài 1.115 (CĐ08) : Tìm giá trị của tham số m để hệ phương trình x − my = 1 mx + y = 3 có nghiệm (x; y) thỏa mãn xy < 0. Bài 1.116 (CĐ09) : Giải bất phương trình √ x + 1 + 2 √ x − 2 ≤ √ 5x + 1. Bài 1.117 (CĐ10) : Giải hệ phương trình 2 √ 2x + y = 3 − 2x − y x2 − 2xy − y2 = 2 (x, y ∈ R). Bài 1.118 (A03) : Giải hệ phương trình : x − 1 x = y − 1 y 2y = x3 + 1. Bài 1.119 (A04) : Giải bất phương trình : 2(x2 − 16) √ x − 3 + √ x − 3 > 7 − x √ x − 3 . TRẦN ANH TUẤN - 0974 396 391 - (04) 66 515 343 Trang 29 www.VNMATH.com www.VNMATH.com
  • 30. WWW.VNMATH.COM CHUYÊN ĐỀ LUYỆN THI ĐẠI HỌC Bài 1.120 (A05) : Giải bất phương trình : √ 5x − 1 − √ x − 1 > √ 2x − 4. Bài 1.121 (A06) : Giải hệ phương trình : x + y − √ xy = 3 √ x + 1 + √ y + 1 = 4 (x, y ∈ R). Bài 1.122 (A07) : Tìm m để phương trình sau có nghiệm thực : 3 √ x − 1 + m √ x + 1 = 2 4√ x2 − 1. Bài 1.123 (A08) : Giải hệ phương trình : x2 + y + x3y + xy2 + xy = − 5 4 x4 + y2 + xy(1 + 2x) = − 5 4 (x, y ∈ R). Bài 1.124 (A08) : Tìm các giá trị của tham số m để phương trình sau có đúng hai nghiệm thực phân biệt : 4√ 2x + √ 2x + 2 4√ 6 − x + 2 √ 6 − x = m (m ∈ R). Bài 1.125 (A09) : Giải phương trình 2 3 √ 3x − 2 + 3 √ 6 − 5x − 8 = 0. Bài 1.126 (A10) : Giải bất phương trình x − √ x 1 − 2(x2 − x + 1) ≥ 1. Bài 1.127 (A10) : Giải hệ phương trình (4x2 + 1)x + (y − 3) √ 5 − 2y = 0 4x2 + y2 + 2 √ 3 − 4x = 7 (x, y ∈ R). Bài 1.128 (B02) : Giải hệ phương trình : 3 √ x − y = √ x − y x + y = √ x + y + 2. Bài 1.129 (B03) : Giải hệ phương trình : 3y = y2 + 2 x2 3x = x2 + 2 y2 . Bài 1.130 (B04) : Xác định m để phương trình sau có nghiệm : m √ 1 + x2 − √ 1 − x2 + 2 = 2 √ 1 − x4 + √ 1 + x2 − √ 1 − x2. Bài 1.131 (B06) : Tìm m để phương trình sau có hai nghiệm thực phân biệt : √ x2 + mx + 2 = 2x + 1. Bài 1.132 (B07) : Chứng minh rằng với mọi giá trị dương của tham số m, phương trình sau có hai nghiệm thực phân biệt : x2 + 2x − 8 = m(x − 2). Bài 1.133 (B08) : Giải hệ phương trình : x4 + 2x3y + x2y2 = 2x + 9 x2 + 2xy = 6x + 6 (x, y ∈ R). Bài 1.134 (B09) : Giải hệ phương trình xy + x + 1 = 7y x2y2 + xy + 1 = 13y2. Bài 1.135 (B10) : Giải phương trình √ 3x + 1 − √ 6 − x + 3x2 − 14x − 8 = 0 (x ∈ R). Bài 1.136 (D02) : Giải bất phương trình : (x2 − 3x) √ 2x2 − 3x − 2 ≥ 0. Bài 1.137 (D02) : Giải hệ phương trình : 23x = 5y2 − 4y 4x + 2x+1 2x + 2 = y. Bài 1.138 (D04) : Tìm m để hệ phương trình sau có nghiệm : √ x + √ y = 1 x √ x + y √ y = 1 − 3m. TRẦN ANH TUẤN - 0974 396 391 - (04) 66 515 343 Trang 30 www.VNMATH.com www.VNMATH.com
  • 31. WWW.VNMATH.COM CHUYÊN ĐỀ LUYỆN THI ĐẠI HỌC Bài 1.139 (D04) : Chứng minh rằng phương trình sau có đúng một nghiệm : x5 − x2 − 2x − 1 = 0. Bài 1.140 (D05) : Giải phương trình : 2 x + 2 + 2 √ x + 1 − √ x + 1 = 4. Bài 1.141 (D06) : Giải phương trình : √ 2x − 1 + x2 − 3x + 1 = 0 (x ∈ R). Bài 1.142 (D07) : Tìm các giá trị của tham số m để hệ phương trình sau có nghiệm thực : x + 1 x + y + 1 y = 5 x3 + 1 x3 + y3 + 1 y3 = 15m − 10. Bài 1.143 (D08) : Giải hệ phương trình : xy + x + y = x2 − 2y2 x √ 2y − y √ x − 1 = 2x − 2y (x, y ∈ R). Bài 1.144 (D09) : Giải hệ phương trình x(x + y + 1) − 3 = 0 (x + y)2 − 5 x2 + 1 = 0. 1.7 Bài tập tổng hợp Bài 1.145 : Giải phương trình : √ x + 4 + √ x − 4 = 2x − 12 + 2 √ x2 − 16. Bài 1.146 : Giải bất phương trình : √ x + 12 ≥ √ x − 3 + √ 2x + 1. Bài 1.147 : Giải hệ phương trình : x2 + y2 + x + y = 4 x(x + y + 1) + y(y + 1) = 2. Bài 1.148 : Giải hệ phương trình : √ 2x + y + 1 − √ x + y = 1 3x + 2y = 4. Bài 1.149 : Giải bất phương trình : √ 8x2 − 6x + 1 − 4x + 1 ≤ 0. Bài 1.150 : Giải bất phuơng trình : √ 2x + 7 − √ 5 − x ≥ √ 3x − 2. Bài 1.151 : Tìm m để hệ phương trình sau có nghiệm : 72x+ √ x+1 − 72+ √ x+1 + 2005x ≤ 2005 x2 − (m + 2)x + 2m + 3 ≥ 0. Bài 1.152 : Giải hệ phương trình : (x2 + 1) + y(y + x) = 4y (x2 + 1)(y + x − 2) = y (x, y ∈ R). Bài 1.153 : Giải hệ phương trình : x3 − 8x = y3 + 2y x2 − 3 = 3(y2 + 1) (x, y ∈ R). Bài 1.154 : Giải hệ phương trình : (x − y)(x2 + y2) = 13 (x + y)(x2 − y2) = 25 (x, y ∈ R). Bài 1.155 : Giải phương trình : √ 3x − 2 + √ x − 1 = 4x − 9 + 2 √ 3x2 − 5x + 2, x ∈ R. Bài 1.156 : Giải hệ phương trình : x2 − xy + y2 = 3(x − y) x2 + xy + y2 = 7(x − y)3 (x, y ∈ R). TRẦN ANH TUẤN - 0974 396 391 - (04) 66 515 343 Trang 31 www.VNMATH.com www.VNMATH.com
  • 32. WWW.VNMATH.COM CHUYÊN ĐỀ LUYỆN THI ĐẠI HỌC Bài 1.157 : Giải phương trình : x + 2 √ 7 − x = 2 √ x − 1 + √ −x2 + 8x − 7 + 1, x ∈ R. Bài 1.158 : Tìm m để phương trình : m √ x2 − 2x + 2 + 1 + x(2 − x) ≤ 0 có nghiệm thuộc đoạn ä 0; 1 + √ 3 ç . Bài 1.159 : Giải hệ phương trình : x4 − x3y + x2y2 = 1 x3y − x2 + xy = 1. Bài 1.160 : Tìm m để phương trình : 4√ x2 + 1 − √ x = m có nghiệm. Bài 1.161 : Tìm m để phương trình : 4√ x4 − 13x + m + x − 1 = 0 có đúng một nghiệm. Bài 1.162 : Tìm m để phương trình : x − 3 − 2 √ x − 4 + x − 6 √ x − 4 + 5 = m có đúng hai nghiệm thực. Bài 1.163 : Tìm m để hệ phương trình : 2x − y − m = 0 x + √ xy = 1 có nghiệm duy nhất. Bài 1.164 : Với giá trị nào của a thì hệ có ít nhất một nghiệm thỏa mãn x, y > 0. Với các giá trị a tìm được hãy tìm tất cả các nghiệm của hệ đã cho : x + y + 1 x + 1 y = 4 x2 + y2 + 1 x2 + 1 y2 = √ 2 − a2 + Ö 2 − 1 a2 + a2 + 1 a . Bài 1.165 : Giải hệ phương trình : y3 + y2x + 3x − 6y = 0 x2 + xy = 3. Bài 1.166 : Cho hệ phương trình : x2 + y2 = m x + y = 6. 1. Giải hệ phương trình với m = 26 ; 2. Tìm m để hệ vô nghiệm ; 3. Tìm m để hệ có nghiệm duy nhất ; 4. Tìm m để hệ hai nghiệm phân biệt. Bài 1.167 : Cho hệ phương trình : x + xy + y = m + 2 x2y + xy2 = m + 1. 1. Giải hệ phương trình với m = −3 ; 2. Xác định m để hệ có nghiệm duy nhất. Bài 1.168 : Cho hệ phương trình : (x − 2)2 + y2 = m x2 + (y − 2)2 = m. Tìm m để hệ có nghiệm duy nhất. Bài 1.169 : Cho hệ phương trình : x = y2 − y + m y = x2 − x + m. 1. Giải hệ phương trình với m = 0 ; 2. Tìm m để hệ phương trình có nghiệm ; 3. Tìm m để hệ phương trình có nghiệm duy nhất. TRẦN ANH TUẤN - 0974 396 391 - (04) 66 515 343 Trang 32 www.VNMATH.com www.VNMATH.com
  • 33. WWW.VNMATH.COM CHUYÊN ĐỀ LUYỆN THI ĐẠI HỌC Bài 1.170 : Tìm a để hệ sau có nghiệm duy nhất : 2|x| + |x| = y + x2 + a x2 + y2 = 1. Bài 1.171 : Tìm a để hệ sau có nghiệm : x2 + 2xy − 7y2 ≥ 1 − a 1 + a 3x2 + 10xy − 5y2 ≤ −2. Bài 1.172 : Tìm m để phương trình sau có nghiệm duy nhất : √ 4 − x + √ x + 5 = m. Bài 1.173 : Tìm m để phương trình sau có nghiệm duy nhất : 4 √ x + 4√ 1 − x + √ x + √ 1 − x = m. Bài 1.174 : Tìm a để hệ sau có nghiệm : x2 − 2xy − 3y2 = 8 2x2 + 4xy + 5y2 = a4 − 4a3 + 4a2 − 12 + √ 105. Bài 1.175 : Cho phương trình x + √ 17 − x2 + x √ 17 − x2 = m. 1. Giải phương trình khi m = 9; 2. Tìm m để phương trình có nghiệm thực; 3. Tìm m để phương trình có nghiệm thực duy nhất. Bài 1.176 : Giải bất phương trình 2x2 − 5x − 3x x2 − 3 x − 6 ≥ 0. Bài 1.177 : Chứng tỏ rằng với mọi số m không âm thì phương trình sau luôn có nghiệm thực 3x2 + (3m2 − 7) √ x2 + 4 − m3 + 6 = 0. Bài 1.178 : Giải hệ phương trình √ x2 + 2 + y2 + 3 + x + y = 5 √ x2 + 2 + √ 2 + 3 − x − y = 2. Bài 1.179 : Giải hệ phương trình x2 + y3 = 2y2 x + y3 = 2y. Bài 1.180 : Giải bất phương trình 2 √ x − 1 − √ x + 2 > x − 2. Bài 1.181 : Giải bất phương trình √ 3x + 7 − √ 2x + 3 > √ x + 2. Bài 1.182 : Giải hệ phương trình 2x2 + x + y2 = 7 xy − x + y = 3. Bài 1.183 : Giải hệ phương trình (x + 3) √ 2x − 1 + (y + 3) √ 2y − 1 = 2 √ (x + 3)(y + 3) x + y = 2xy. Bài 1.184 : Giải hệ phương trình x + 3x − y x2 + y2 = 3 y − x + 3y x2 + y2 = 0. TRẦN ANH TUẤN - 0974 396 391 - (04) 66 515 343 Trang 33 www.VNMATH.com www.VNMATH.com
  • 34. WWW.VNMATH.COM CHUYÊN ĐỀ LUYỆN THI ĐẠI HỌC Bài 1.185 : Giải phương trình √ (x + 2)(2x − 1) − 3 √ x + 6 = 4 − √ (x + 6)(2x − 1) + 3 √ x + 2. Bài 1.186 : Giải hệ phương trình √ x − y − √ x + y = 2 x2 + y2 + x2 − y2 = 4. Bài 1.187 : Giải hệ phương trình x2 + xy + y2 = 7(x − y)2 x2 − xy + y2 = 3(x − y). Bài 1.188 : Giải hệ phương trình x + y + x2 − y2 = 12 y x2 − y2 = 12. Bài 1.189 : Giải hệ phương trình (2x + 1)2 + y2 + y = 2x + 3 xy + x = −1. Bài 1.190 : Giải phương trình (x2 + 1)2 = 5 − x √ 2x2 + 4. Bài 1.191 : Giải hệ phương trình x3 − y3 + 2 = 0 x2 + y2 + x − y = 0. Bài 1.192 : Giải phương trình |x + √ 1 − x2| = √ 2(1 − 2x2). Bài 1.193 : Giải hệ phương trình x2 + 6y = y + 3 √ x + y + √ x − y = 4. Bài 1.194 : Tìm m để phương trình sau có nghiệm thực x3 + x2 + x − m(x2 + 1)2 = 0. Bài 1.195 : Giải bất phương trình 1 √ 2x2 + 3x − 5 > 1 2x − 1 . Bài 1.196 : Tìm m để phương trình √ 2x2 − mx + 13 = x − 2 có nghiệm thực. Bài 1.197 : Giải hệ phương trình 2x y + Ö 2y x = 3 x − y + xy = 3. Bài 1.198 : Giải hệ phương trình √ x + 1 + √ y − 1 = 4 √ x + 6 + √ y + 4 = 6. Bài 1.199 : Tìm các giá trị của tham số m để hệ phương trình sau có nghiệm thực x2 + y2 + 2(x + y) = 2 xy(x + 2)(y + 2) = 2m(2m+1 − 1). Bài 1.200 : Chứng minh rằng với mọi m ≥ 2010 hệ phương trình sau có không quá một nghiệm thực √ x + 27 − √ y + 1 = (m − 2010)y + 1 √ y + 27 − √ x + 1 = (m − 2010)x + 1. Bài 1.201 : Giải phương trình √ x + 1 + 1 = 4x2 + √ 3x. Bài 1.202 : Giải hệ phương trình x3y(1 + y) + x2y2(2 + y) + xy3 − 30 = 0 x2y + x(1 + y + y2) + y − 11 = 0. TRẦN ANH TUẤN - 0974 396 391 - (04) 66 515 343 Trang 34 www.VNMATH.com www.VNMATH.com
  • 35. WWW.VNMATH.COM CHUYÊN ĐỀ LUYỆN THI ĐẠI HỌC Bài 1.203 : Giải hệ phương trình x3 + 4y = y3 + 16x 1 + y2 = 5(1 + x2). Bài 1.204 : Giải hệ phương trình 2 + 6y = x y − √ x − 2y x + √ x − 2y = x + 3y − 2. Bài 1.205 : Giải bất phương trình x √ 2 − x ≤ x2 − x − 2 − √ 2 − x. Bài 1.206 : Giải hệ phương trình x3 + y3 = 1 x2y + 2xy2 + y3 = 2. Bài 1.207 : Tìm các giá trị của tham số m để bất phương trình x(4 − x) + m √ x2 − 4x + 5 + 2 ≤ 0 nghiệm đúng với mọi giá trị của x ∈ [2; 2 + √ 3]. Bài 1.208 : Giải hệ phương trình 2x2y + y3 = 2x4 + x6 (x + 2) √ y + 1 = (x + 1)2. Bài 1.209 : Giải hệ phương trình x − 2y − √ xy = 0 √ x − 1 + √ 4y − 1 = 2. Bài 1.210 : Giải hệ phương trình x √ x − 8 √ y = √ x + y √ y x − y = 5. Bài 1.211 : Tìm m để hệ phương trình x3 − y3 + 3y2 − 3x − 2 = 0 x2 + √ 1 − x2 − 3 2y − y2 + m = 0 có nghiệm thực. Bài 1.212 : Giải phương trình √ x + 3 + 2x √ x + 1 = 2x + √ x2 + 4x + 3. Bài 1.213 : Xác định các giá trị của tham số m để phương trình √ 2x2 + 2mx + m + 1 = 1 − x có đúng một nghiệm thực dương. Bài 1.214 : Giải hệ phương trình x2 + y2 + x2y2 = 1 + 2xy x + x2y + xy = xy2 + y + 1. Bài 1.215 : Giải phương trình √ 2x2 + 3x + 1 − √ 2x2 − 2 = x + 1. Bài 1.216 : Xác định các giá trị của tham số m để phương trình √ x − 1 − m √ x + 6√ x3 − x2 = 0 có nghiệm thực. Bài 1.217 : Tìm m để hệ phương trình sau có nghiệm thực x 3 + x4 + y 3 + y4 = 1 8 xy 9 + 3x4 + 3y4 + x4y4 = m. Bài 1.218 : Giải phương trình 1 x + 1 √ 2 − x2 = 2. Bài 1.219 : Giải hệ phương trình x2 + y2 = 5 √ y − 1(x + y − 1) = (y − 2) √ x + y. Bài 1.220 : Giải hệ phương trình x2 + 1 + y2 + xy = 4y x + y − 2 = y x2 + 1 . TRẦN ANH TUẤN - 0974 396 391 - (04) 66 515 343 Trang 35 www.VNMATH.com www.VNMATH.com
  • 36. WWW.VNMATH.COM CHUYÊN ĐỀ LUYỆN THI ĐẠI HỌC Bài 1.221 : Tìm m để phương trình √ x + √ x + 4 − m √ 4 − x = 3m có nghiệm thực. Bài 1.222 : Giải hệ phương trình x3y = 24 2 √ x3 + y = 6 3√ 3. Bài 1.223 : Giải hệ phương trình √ x − 1 + √ y − 1 = 3 x + y − √ (x − 1)(y − 1) = 5. Bài 1.224 : Tìm m để phương trình m √ x − 2 + 2 4√ x2 − 4 − √ x + 2 = 2 4√ x2 − 4 có nghiệm. Bài 1.225 : Giải hệ phương trình x2 + y2 + x + y = 18 x(x + 1)y(y + 1) = 72. Bài 1.226 : Giải hệ phương trình √ 7x + y + √ 2x + y = 5 √ 2x + y + 20x + 5y = 38. Bài 1.227 : Giải hệ phương trình xy + x2 = 1 + y xy + y2 = 1 + x. Bài 1.228 : Tìm các giá trị của tham số m để phương trình m + 2 3 √ x − x2 = √ x + √ 1 − x có nghiệm. Bài 1.229 : Giải bất phương trình 5 √ x + 5 2 √ x ≤ 2x + 1 2x + 5. TRẦN ANH TUẤN - 0974 396 391 - (04) 66 515 343 Trang 36 www.VNMATH.com www.VNMATH.com WWW.VNMATH.COM
  • 37. WWW.VNMATH.COM Chương 2 Bất đẳng thức 2.1 Phương pháp sử dụng bất đẳng thức Cauchy 2.1.1 Bất đẳng thức Cauchy - So sánh giữa tổng và tích Cho ba số không âm a, b, c, ta có : 1. a + b 2 ≥ √ ab, dấu bằng xảy ra khi a = b ; 2. a + b + c 3 ≥ 3 √ abc, dấu bằng xảy ra khi a = b = c. 2.1.2 Một số hệ quả trực tiếp Hệ quả 1 : So sánh giữa tổng nghịch đảo và tổng. Cho ba số dương a, b, c có : 1. 1 a + 1 b ≥ 4 a + b ; 2. 1 a + 1 b + 1 c ≥ 9 a + b + c . Hệ quả 2 : So sánh giữa tổng bình phương và tồng. Cho ba số thực a, b, c có : 1. 2(a2 + b2) ≥ (a + b)2 ; 2. 3(a2 + b2 + c2) ≥ (a + b + c). Hệ quả 3 : So sánh giữa tổng, tổng bình phương và tích. Cho ba số thực a, b, c có : 1. (a + b + c)2 ≥ 3(ab + bc + ca) ; 2. a2 + b2 + c2 ≥ ab + bc + ca. 2.1.3 Bài tập đề nghị Bài 2.1 : Cho a, b, > 0. Chứng minh rằng : ab(a + b) 2 ≤ a + b 2 3 ≤ (a + b)(a2 + ab + b2) 6 ≤ a3 + b3 2 ≤ (a2 + b2)3 (a + b)3 . Bài 2.2 : Cho a, b > 0 và a + b ≤ 1. Chứng minh rằng : 37
  • 38. WWW.VNMATH.COM CHUYÊN ĐỀ LUYỆN THI ĐẠI HỌC 1. 1 a + 1 b ≥ 4 ; 2. 1 a + 1 b + a + b ≥ 5. Bài 2.3 : Cho các số không âm a, b, c có a + b + c ≤ 3. Chứng minh rằng : 1. a + b + c ≥ ab + bc + ca ; 2. √ a + √ b + √ c ≥ ab + bc + ca. Bài 2.4 : Cho x, y > 0. Chứng minh rằng : (1 + x)(1 + y) ≥ (1 + √ xy)2. Bài 2.5 : Cho x, y > 0. Chứng minh rằng : x2 + y2 + 1 x + 1 y ≥ 2( √ x + √ y). Bài 2.6 : Cho x, y > 0 và x + y = 1. Tìm giá trị nhỏ nhất của P = 1 x2 + y2 + 1 xy . Bài 2.7 : Cho x, y, z > 0 và x + y + z = 1. Tìm giá trị lớn nhất của P = x x + 1 + y y + 1 + z z + 1 . Bài 2.8 : Cho a, b > 0 và a + b = 1. Chứng minh rằng : a2 a + 1 + b2 b + 1 ≥ 1 3 . Bài 2.9 : Cho các số thực dương a, b, c. Chứng minh rằng : 1 a + 3b + 1 b + 3c + 1 c + 3a ≥ 1 2a + b + c + 1 2b + c + a + 1 2c + a + b . Bài 2.10 : Chứng minh rằng với mọi a, b, c > 0 đều có : 1. 1 a(b + c) + 1 b(c + a) + 1 c(a + b) ≥ 27 2(a + b + c)2 ; 2. 1 a(a + b) + 1 b(b + c) + 1 c(c + a) ≥ 27 2(a + b + c)2 . Bài 2.11 : Cho a, b > 0 và a + b ≤ 1. Tìm giá trị nhỏ nhất của S = ab + 1 ab . Bài 2.12 : Cho a, b > 0. Tìm giá trị nhỏ nhất của biểu thức S = a + b √ ab + √ ab a + b . Bài 2.13 : Cho a, b, c > 0 và a + b + c ≤ 3 2 . Tìm giá trị nhỏ nhất của biểu thức S = a + b + c + 1 a + 1 b + 1 c . Bài 2.14 : Chứng minh rằng với mọi số dương x, y, z đều có : x2 + y2 + z2 ≥ √ 2(xy + yz). Bài 2.15 : Cho a, b, c > 0 và a + b + c = 4. Chứng minh rằng : ab a + b + 2c + bc b + c + 2a + ca c + a + 2b ≤ 1. Bài 2.16 : Cho a, b, c > 0. Chứng minh rằng : ab a + 3b + 2c + bc b + 3c + 2a + ca c + 3a + 2b ≤ a + b + c 6 . Bài 2.17 : Cho a, b, c > 0. Chứng minh rằng : 1. a + b c + b + c a + c + a b ≥ 6 ; 2. a b + c + b c + a + c a + b ≥ 3 2 ; 3. a2 b + c + b2 c + a + c2 a + b ≥ a + b + c 2 ; 4. a3 b + c + b3 c + a + c3 a + b ≥ a2 + b2 + c2 2 . Bài 2.18 : Cho a, b, c > 0 và abc = 1. Tìm giá trị nhỏ nhất của các biểu thức sau : 1. P = a2 b + c + b2 c + a + c2 a + b ; 2. Q = a3 b + c + b3 c + a + c3 a + b ; 3. R = a2 √ a b + c + b2 √ b c + a + c2 √ c a + b ; 4. S = bc a2b + a2c + ca b2c + b2a + ab c2a + c2b ; TRẦN ANH TUẤN - 0974 396 391 - (04) 66 515 343 Trang 38 www.VNMATH.com www.VNMATH.com
  • 39. WWW.VNMATH.COM CHUYÊN ĐỀ LUYỆN THI ĐẠI HỌC Bài 2.19 : Cho x, y, z, t > 0 và xyzt = 1. Tìm giá trị nhỏ nhất của biểu thức : P = 1 x3(yz + zt + ty) + 1 y3(zt + tx + xz) + 1 z3(tx + xy + yt) + 1 t3(xy + yz + zx) . Bài 2.20 : Cho a, b, c > 0. Tìm giá trị nhỏ nhất của biểu thức sau : 1. P = a b + 2c + b c + 2a + c a + 2b . 2. Q = a b + mc + b c + ma + c a + mb , m ∈ N, m > 2.1 Bài 2.21 : Cho a, b, c > 0. Chứng minh rằng : 1. (a + b)(b + c)(c + a) ≥ 8abc ; 2. bc a + ca b + ba c ≥ a + b + c. Bài 2.22 : Cho a, b, c là độ dài ba cạnh của một tam giác. Chứng minh rằng : 1. a b + c − a + b c + a − b + c a + b − c ≥ 3 ; 2. a2 b + c − a + b2 c + a − b + c2 a + b − c ≥ a + b + c. Bài 2.23 : 1. Cho a, b, c là độ dài ba cạnh của một tam giác, p là nửa chu vi của tam giác. Chứng minh rằng : (p − a)(p − b)(p − c) ≤ abc 8 . 2. Cho tam giác ABC có chu vi bằng 3 và độ dài ba cạnh của tam giác là a, b, c. Chứng minh rằng : 4(a3 + b3 + c3 ) + 15abc ≥ 27. Bài 2.24 : Cho a, b, c, d > 0 và a + b + c + d = 1. Chứng minh rằng : 1 a − 1 1 b − 1 1 c − 1 1 d − 1 ≥ 81. Bài 2.25 : Cho a, b ≥ 1. Chứng minh rằng : a √ b − 1 + b √ a − 1 ≤ ab. Bài 2.26 : Cho a, b, c ≥ 0 và a + b + c = 1. Chứng minh rằng : ab + bc + ca + abc ≤ 10 27 . Bài 2.27 : Cho a, b, c > 0. Chứng minh rằng : 2 a2 + bc ≤ 1 2 1 ab + 1 ac . Bài 2.28 : Cho a, b > 0 và a + b = 1. Chứng minh rằng : 3 ab + 2 a2 + b2 ≥ 16. Bài 2.29 : Cho a, b, c > 0 và 1 1 + a + 1 1 + b + 1 1 + c ≥ 2. Chứng minh rằng : abc ≤ 1 8 . Bài 2.30 : Cho a > b > 0 và ab = 1. Chứng minh rằng : a2 + b2 a − b ≥ 2 √ 2. Bài 2.31 : Tìm giá trị nhỏ nhất của A = (1 + x) 1 + 1 y + (1 + y) 1 + 1 x với x, y > 0 thỏa mãn x2 + y2 = 1. Bài 2.32 : Cho x, y, z > 1 thỏa mãn x + y + z = xyz. Tìm giá trị nhỏ nhất của : P = y − 2 x2 + z − 2 y2 + x − 2 z2 . Bài 2.33 : Cho a, b, c > 1. Chứng minh rằng : alogb c + blogc a + cloga b ≥ 3 3√ abc. 1 Một cách tổng quát, tìm giá trị nhỏ nhất của R = a xb + yc + b xc + ya + c xa + yb với a, b, c, x, y là những số dương TRẦN ANH TUẤN - 0974 396 391 - (04) 66 515 343 Trang 39 www.VNMATH.com www.VNMATH.com
  • 40. WWW.VNMATH.COM CHUYÊN ĐỀ LUYỆN THI ĐẠI HỌC Bài 2.34 : Cho a, b, c > 0 và a + b + c = 1. Chứng minh rằng : 1 + 1 a 1 + 1 b 1 + 1 c ≥ 64. Bài 2.35 : Cho a, b > 0. Chứng minh rằng : (a + b)2 + 1 a + 1 b 2 ≥ 8. Bài 2.36 : Cho a, b, c > 0. Chứng minh rằng : bc a2b + a2c + ca b2c + b2a + ab c2a + c2b ≥ 1 2 1 a + 1 b + 1 c . Bài 2.37 : Cho a, b, c > 0. Chứng minh rằng : ab a + b + bc b + c + ca c + a ≤ a + b + c 2 . Bài 2.38 : Cho a ≥ 3. Tìm giá trị nhỏ nhất của biểu thức S = a + 1 a . Bài 2.39 : Cho a ≥ 2. Tìm giá trị nhỏ nhất của biểu thức S = a + 1 a2 . Bài 2.40 : Cho a, b, c ≥ 0 thỏa mãn a2 + b2 + c2 = 1. Tìm giá trị nhỏ nhất của biểu thức S = a + b + c + 1 abc . Bài 2.41 : Cho x, y > 0 và x + y = 1. Tìm giá trị nhỏ nhất của biểu thức S = x √ 1 − x + y √ 1 − y . Bài 2.42 : Cho a, b, c ≥ 0 và a + b + c = 1. Tìm giá trị lớn nhất của biểu thức S = 3√ a + b + 3√ b + c + 3 √ c + a. Bài 2.43 : Cho a, b, c > 0 và a + b + c = 3. Tìm giá trị lớn nhất của biểu thức S = 3 a(b + 2c) + 3 b(c + 2a) + 3 c(a + 2b). Bài 2.44 : Cho a ≥ 2; b ≥ 6; c ≥ 12. Tìm giá trị lớn nhất của biểu thức S = bc √ a − 2 + ca 3√ b − 6 + ab 4 √ c − 12 abc . Bài 2.45 : Chứng minh rằng : a b + b c + c a 2 ≥ 3 2 a + b c + b + c a + c + a b với mọi a, b, c > 0. Bài 2.46 : Cho a, b, c > 0 và a + b + c = 3. Chứng minh rằng : a3 (a + b)(a + c) + b3 (b + c)(b + a) + c3 (c + a)(c + b) ≥ 3 4 . Bài 2.47 : Cho a, b, c > 0 và a + b + c = 3. Chứng minh rằng : a3 b(2c + a) + b3 c(2a + b) + c3 c(2b + c) ≥ 1. Bài 2.48 : Cho a, b, c > 0 và a2 + b2 + c2 = 1. Chứng minh rằng : a3 b + 2c + b3 c + 2a + c3 a + 2b ≥ 1 3 . Bài 2.49 : Cho a, b, c > 0 và a2 + b2 + c2 = 1. Chứng minh rằng : a3 a + b + b3 b + c + c3 c + a ≥ 1 2 . TRẦN ANH TUẤN - 0974 396 391 - (04) 66 515 343 Trang 40 www.VNMATH.com www.VNMATH.com
  • 41. WWW.VNMATH.COM CHUYÊN ĐỀ LUYỆN THI ĐẠI HỌC Bài 2.50 : Cho a, b, c > 0 và ab + bc + ca = 1. Chứng minh rằng : a √ 1 + a2 + b √ 1 + b2 + c √ 1 + c2 ≤ 3 2 . Bài 2.51 : Cho a, b, c > 0 và ab + bc + ca = 1. Chứng minh rằng : 1 a(a + b) + 1 b(b + c) + 1 c(c + a) ≥ 9 2 . Bài 2.52 : Cho a, b, c > 0 và a + b + c = 1. Chứng minh rằng : a (b + c)2 + b (c + a)2 + c (a + b)2 ≥ 9 4 . Bài 2.53 : Cho a, b, c > 0 và a2 + b2 + c2 = 3. Chứng minh rằng : ab c + bc a + ca b ≥ 3. Bài 2.54 : Cho a, b, c > 0 và a + b + c = 1. Chứng minh rằng : bc √ a + bc + ca √ b + ca + ab √ c + ab ≤ 1 2 . Bài 2.55 : Cho a, b, c > 0 và a + b + c = 2. Chứng minh rằng : bc √ 2a + bc + ca √ 2b + ca + ab √ 2c + ab ≤ 1. Bài 2.56 : Cho a, b, c > 0 và abc = 1. Chứng minh rằng : a3 (1 + b)(1 + c) + b3 (1 + c)(1 + a) + c3 (1 + a)(1 + b) ≥ 3 4 . Bài 2.57 : Cho a, b, c > 0 và abc = 1. Chứng minh rằng : 1 a3(b + c) + 1 b3(c + a) + 1 c3(a + b) ≥ 3 2 . Bài 2.58 : Cho a, b, c > 0. Chứng minh rằng : 1 a + 1 b + 1 c ≥ 2 1 a + b + 1 b + c + 1 c + a . Bài 2.59 : Cho a, b, c > 0 và a + b + c ≤ 1. Chứng minh rằng : 1 a2 + 2bc + 1 b2 + 2ca + 1 c2 + 2ab ≥ 9. Bài 2.60 : Cho a, b > 0 và a + b ≤ 1. Chứng minh rằng : 1 a2 + b2 + 1 ab ≥ 6. Bài 2.61 : Cho a, b > 0 và a + b ≤ 1. Chứng minh rằng : 1 a2 + b2 + 1 ab + 4ab ≥ 7. Bài 2.62 : Cho a, b, c > 0 và ab + bc + ca = abc. Chứng minh rằng : 1 a + 2b + 3c + 1 b + 2c + 3a + 1 c + 2a + 3b < 3 16 . Bài 2.63 : Tìm giá trị nhỏ nhất của : A = a 1 + b − a + b 1 + c − b + c 1 + a − c với a, b, c > 0 và a + b + c = 1. Bài 2.64 : Cho x, y, z > 0 và x2 + y2 + z2 = 1. Tìm giá trị nhỏ nhất của biểu thức : P = x y2 + z2 + y z2 + x2 + z x2 + y2 . Bài 2.65 : Cho x, y là hai số thực thay đổi. Tìm giá trị lớn nhất và giá trị nhỏ nhất của biểu thức : P = (x + y)(1 − xy) (1 + x2)2(1 + y2)2 . TRẦN ANH TUẤN - 0974 396 391 - (04) 66 515 343 Trang 41 www.VNMATH.com www.VNMATH.com
  • 42. WWW.VNMATH.COM CHUYÊN ĐỀ LUYỆN THI ĐẠI HỌC Bài 2.66 : Cho x, y, z là ba số thực thỏa mãn x + y + z = 0. Tìm giá trị nhỏ nhất của P = √ 2x + 3 + √ 2y + 3 + √ 2z + 3. Bài 2.67 : Cho các số thực x, y, z thỏa mãn x + y + z = 6. Chứng minh rằng : 8x + 8y + 8z ≥ 4x+1 + 4y+1 + 4z+1. Bài 2.68 : Cho 0 < a ≤ b ≤ c ≤ d ≤ e và a + b + c + d + e = 1. Chứng minh rằng : a(bc + be + cd + de) + cd(b + e − a) ≤ 1 25 . Bài 2.69 : Cho a, b, c là ba số dương thỏa mãn điều kiện ab + bc + ca = abc. Chứng minh rằng : a2 a + bc + b2 b + ca + c2 c + ab ≥ a + b + c 4 . Bài 2.70 : Cho a, b, c là các số thực dương, chứng minh rằng : b + c a + 3 4(b3 + c3) + c + a b + 3 4(c3 + a3) + a + b c + 3 4(a3 + b3) ≤ 2. Bài 2.71 : Cho a, b, c là các số thực dương, chứng minh rằng : 1 a3 + b3 + abc + 1 b3 + c3 + abc + 1 c3 + a3 + abc ≤ 1 abc . Bài 2.72 : Cho a, b, c là các số thực dương thỏa mãn abc = 1. Chứng minh rằng : a3 + b3 a2 + ab + b2 + b3 + c3 b2 + bc + c2 + c3 + a3 c2 + ca + a2 ≥ 2. Bài 2.73 : Cho ba số thực dương a, b, c. Chứng minh rằng : 2 √ a a3 + b2 + 2 √ b b3 + c2 + 2 √ c c3 + a2 ≤ 1 a2 + 1 b2 + 1 c2 . Bài 2.74 : Cho a, b, c > 0. Chứng minh rằng : 1 a2 + bc + 1 b2 + ca + 1 c2 + ab ≤ a + b + c 2abc . Bài 2.75 : Cho a, b, c là ba số dương sao cho ab + bc + ca ≥ 1. Chứng minh rằng : a3 b2 + 1 + b3 c2 + 1 + c3 a2 + 1 ≥ √ 3 4 . 2.2 Bất đẳng thức hình học Bài 2.76 : Cho a, b, c ∈ R. Chứng minh rằng : √ a2 + b2 + 4c2 + 4ac + √ a2 + b2 + 4c2 − 4ac ≥ 2 √ a2 + b2. Bài 2.77 : Với mọi a, b, c, d ∈ R. Chứng minh rằng : √ a2 + b2 + c2 + d2 + 2ac + 2bd ≤ √ a2 + b2 + √ c2 + d2. Bài 2.78 : Cho x, y, z > 0. Chứng minh rằng : √ x + 2 √ y + 3 √ z ≤ 14(x + y + z). TRẦN ANH TUẤN - 0974 396 391 - (04) 66 515 343 Trang 42 www.VNMATH.com www.VNMATH.com
  • 43. WWW.VNMATH.COM CHUYÊN ĐỀ LUYỆN THI ĐẠI HỌC Bài 2.79 : Cho bốn số a, b, c, d ∈ R thỏa mãn a2 + b2 = 1 và c + d = 3. Chứng minh rằng : ac + bd + cd ≤ 9 + 6 √ 2 4 . Bài 2.80 : Với mọi a, b, c ∈ R. Chứng minh rằng : √ a2 + ab + b2 + √ a2 + ac + c2 ≥ √ b2 + bc + c2. Bài 2.81 : Với mọi x, y ∈ R. Chứng minh rằng : 4 cos2 x cos2 y + sin2 (x − y) + 4 sin2 x sin2 y + sin2 (x − y) ≥ 2. Bài 2.82 : Với mọi x, y ∈ R. Chứng minh rằng : 4x2 + y2 + 12x + 9 + 4x2 + y2 − 4x − 6y + 10 ≥ 5. Bài 2.83 : Cho a + b + c = 1, ax + by + cz = 4 với a, b, c 0. Chứng minh rằng : √ 9a2 + a2x2 + 9b2 + b2y2 + 9c2 + c2z2 ≥ 5. Bài 2.84 : Cho a, b, c > 0. Chứng minh rằng : a2 − ab √ 2 + b2 + b2 − bc √ 3 + c2 ≥ Õ a2 − ac 2 − √ 3 + c2. Bài 2.85 : Cho a, b, c > 0 và abc + bc + ca = abc. Chứng minh rằng : √ b2 + 2a2 ab + √ c2 + 2b2 bc + √ a2 + 2c2 ac ≥ √ 3. Bài 2.86 : Cho x2 + y2 = 1. Chứng minh rằng : x2 √ 5 + 2xy − y2 √ 5 ≤ √ 6. Bài 2.87 : Cho x2 + xy + y2 = 3 y2 + yz + z2 = 16 và x, y, z là các số thực dương. Chứng minh rằng : xy + yz + zx ≤ 8. Bài 2.88 : Cho x, y, z là những số dương. Chứng minh rằng : x2 + xy + y2 + y2 + yz + z2 + z2 + zx + x2 ≥ √ 3(x + y + z). Bài 2.89 : Cho a + b + c = 12. Chứng minh rằng : 3a + 2 √ a + 1 + 3b + 2 √ b + 1 + 3c + 2 √ c + 1 ≥ 3 √ 17. Bài 2.90 : Cho các số dương x, y, z và x + y + z ≤ 2. Chứng minh rằng : 4x2 + 1 x2 + 4y2 + 1 y2 + 4z2 + 1 z2 ≥ √ 145 2 . Bài 2.91 : Giả sử x, y, u, v ∈ R thỏa mãn : x2 + y2 = 1; u2 + v2 + 16 = 8u + 4v. Tìm giá trị lớn nhất của biểu thức P = 8u + 4v − 2(ux + vy). Bài 2.92 : Cho x, y, z là các số dương thỏa mãn xy + yz + zx = 5. Tìm giá trị nhỏ nhất của P = 3x2 + 3y2 + z2. TRẦN ANH TUẤN - 0974 396 391 - (04) 66 515 343 Trang 43 www.VNMATH.com www.VNMATH.com
  • 44. WWW.VNMATH.COM CHUYÊN ĐỀ LUYỆN THI ĐẠI HỌC 2.3 Phương pháp sử dụng điều kiện có nghiệm của phương trình hoặc hệ phương trình - phương pháp miền giá trị Bài 2.93 : Tìm giá trị lớn nhất và nhỏ nhất của hàm số : f(x) = 2x2 + 7x + 23 x2 + 2x + 10 . Bài 2.94 : Tìm giá trị lớn nhất và giá trị nhỏ nhất của biểu thức P = x2 − (x − 4y)2 x2 + 4y2 , với x2 + y2 > 0. Bài 2.95 : Cho x là số dương, y là số thực tùy ý. Tìm giá trị lớn nhất, giá trị nhỏ nhất (nếu có) của biểu thức : P = xy2 (x2 + 3y2) x + x2 + 12y2 . Bài 2.96 : Tìm giá trị nhỏ nhất của biểu thức P = x2 + y2, với 2x2 + y2 + xy ≥ 1. Bài 2.97 : Cho các số thực x, y thỏa mãn điều kiện : 3 √ x( 3 √ x − 1) + 3 √ y( 3 √ y − 1) = 3 √ xy. Tìm giá trị lớn nhất, nhỏ nhất của biểu thức : P = 3 √ x + 3 √ y + 3 √ xy. Bài 2.98 : Cho x, y thỏa mãn điều kiện : x2 − xy+y2 = 3. Tìm giá trị lớn nhất và nhỏ nhất của biểu thức : P = x2 + xy−2y2. Bài 2.99 : Cho hai số thực x, y thỏa mãn điều kiện : x − 3 √ x + 1 = 3 √ y + 2 − y. Tìm giá trị lớn nhất, nhỏ nhất của biểu thức P = x + y. Bài 2.100 : Cho hai số thực x, y thỏa mãn : x2 + y2 = 2(x + y) + 7. Tìm giá trị lớn nhất, giá trị nhỏ nhất của biểu thức P = 3 √ x(x − 2) + 3 √ y(y − 2). Bài 2.101 : Cho các số thực x, y thỏa mãn : 4x2 − 3xy + 3y2 = 6. Tìm giá trị lớn nhất, giá trị nhỏ nhất của biểu thức P = x2 + xy − 2y2. Bài 2.102 : Cho các số thực x, y thỏa mãn : √ x + √ y = 4. Tìm giá trị lớn nhất, giá trị nhỏ nhất của biểu thức P = √ x + 1 + √ y + 9. Bài 2.103 : Cho các số thực x, y thỏa mãn : xy + x + y = 3. Tìm giá trị lớn nhất, giá trị nhỏ nhất của biểu thức P = 3x y + 1 + 3y x + 1 − x2 − y2. Bài 2.104 : Cho a, b ≥ 0 và a2 + b2 + ab = 3. Tìm giá trị nhỏ nhất và giá trị lớn nhất của biểu thức P = a4 + b4 + 2ab − a5 b5 . Bài 2.105 : Cho các số thực x, y thỏa mãn x + y = 2. Tìm giá trị lớn nhất của P = (x3 + 2)(y3 + 2). 2.4 Bất đẳng thức trong các kì thi tuyển sinh ĐH Bài 2.106 (CĐ08) : Cho hai số thực x, y thay đổi và thoả mãn x2 + y2 = 2. Tìm giá trị lớn nhất và giá trị nhỏ nhất của biểu thức: P = 2(x3 + y3) − 3xy. Bài 2.107 (CĐ10) : Cho hai số thực dương thay đổi x, y thỏa mãn điều kiện 3x + y ≤ 1. Tìm giá trị nhỏ nhất của biểu thức A = 1 x + 1 √ xy . Bài 2.108 (A03) : Cho x, y, z là ba số dương và x + y + z ≤ 1. Chứng minh rằng : x2 + 1 x2 + y2 + 1 y2 + z2 + 1 z2 ≥ √ 82. TRẦN ANH TUẤN - 0974 396 391 - (04) 66 515 343 Trang 44 www.VNMATH.com www.VNMATH.com
  • 45. WWW.VNMATH.COM CHUYÊN ĐỀ LUYỆN THI ĐẠI HỌC Bài 2.109 (A05) : Cho x, y, z là các số dương thoả mãn : 1 x + 1 y + 1 z = 4. Chứng minh rằng : 1 2x + y + z + 1 x + 2y + z + 1 x + y + 2z ≤ 1. Bài 2.110 (A06) : Cho hai số thực x 0, y 0 thay đổi và thoả mãn điều kiện : (x + y)xy = x2 + y2 − xy. Tim giá trị lớn nhất của biểu thức A = 1 x3 + 1 y3 . Bài 2.111 (A07) : Cho x, y, z là các số thực dương thay đổi và thoả mãn điều kiện xyz = 1. Tìm giá trị nhỏ nhất của biểu thức : P = x2(y + z) y √ y + 2z √ z + y2(z + x) z √ z + 2x √ x + z2(x + y) x √ x + 2y √ y . Bài 2.112 (A09) : Chứng minh rằng với mọi số thực dương x, y, z thỏa mãn x(x + y + z) = 3yz ta có : (x + y)3 + (x + z)3 + 3(x + y)(y + z)(z + x) ≤ 5(y + z)3 . Bài 2.113 (B05) : Chứng minh rằng với mọi x ∈ R, ta có : 12 5 x + 15 4 x + 20 3 x ≥ 33 + 4x + 5x. Khi nào đẳng thức xảy ra. Bài 2.114 (B06) : Cho x, y là các số thực thay đổi. Tìm giá trị nhỏ nhất của biểu thức : A = (x − 1)2 + y2 + (x + 1)2 + y2 + |y − 2|. Bài 2.115 (B07) : Cho x, y, z là ba số thực dương thay đổi. Tìm giá trị nhỏ nhất của biểu thức : P = x x 2 + 1 yz + y y 2 + 1 xz + z z 2 + 1 xy . Bài 2.116 (B08) : Cho hai số thực x, y thay đổi và thoả mãn hệ thức x2 + y2 = 1. Tìm giá trị lớn nhất và giá trị nhỏ nhất của biểu thức P = 2(x2 + 6xy) 1 + 2xy + 2y2 . Bài 2.117 (B09) : Cho các số thực x, y thay đổi thỏa mãn (x + y)3 + 4xy ≥ 2. Tìm giá trị nhỏ nhất của biểu thức : A = 3(x4 + y4 + x2 + y2 ) − 2(x2 + y2 ) + 1. Bài 2.118 (B10) : Cho các số thực không âm a, b, c thỏa mãn a + b + c = 1. Tìm giá trị nhỏ nhất của biểu thức M = 3(a2b2 + b2c2 + c2a2) + 3(ab + bc + ca) + 2 √ a2 + b2 + c2. Bài 2.119 (D05) : Cho các số dương x, y, z thoả mãn xyz = 1. Chứng minh rằng : 1 + x3 + y3 xy + 1 + y3 + z3 yz + √ 1 + z3 + x3 zx ≥ 3 √ 3. Bài 2.120 (D07) : Cho a ≥ b > 0. Chứng minh rằng : 2a + 1 2a b ≤ 2b + 1 2b a . Bài 2.121 (D08) : Cho x, y là hai số thực không âm thay đổi. Tìm giá trị lớn nhất và giá trị nhỏ nhất của biểu thức : P = (x − y)(1 − xy) (1 + x)2(1 + y)2 . Bài 2.122 (D09) : Cho các số thực không âm x, y thay đổi và thỏa mãn x + y = 1. Tìm giá trị lớn nhất và giá trị nhỏ nhất của biểu thức : S = (4x2 + 3y)(4y2 + 3x) + 25xy. Bài 2.123 (D10) : Tìm giá trị nhỏ nhất của hàm số y = √ −x2 + 4x + 21 − √ −x2 + 3x + 10. TRẦN ANH TUẤN - 0974 396 391 - (04) 66 515 343 Trang 45 www.VNMATH.com www.VNMATH.com
  • 46. WWW.VNMATH.COM CHUYÊN ĐỀ LUYỆN THI ĐẠI HỌC 2.5 Bài tập tổng hợp Bài 2.124 : Giả sử x, y là hai số dương thay đổi thoả mãn điều kiện x + y = 5 4 . Tìm giá trị nhỏ nhất của biểu thức : S = 4 x + 1 4y . Bài 2.125 : Giả sử a, b, c, d là bốn số nguyên thay đổi thoả mãn 1 ≤ a < b < c < d ≤ 50. Chứng minh a b + c d ≥ b2 + b + 50 50b và tìm giá trị nhỏ nhất của biểu thức : S = a b + c d . Bài 2.126 : Cho x, y, z là ba số thoả mãn x + y + z = 0. Chứng minh rằng : √ 3 + 4x + √ 3 + 4y + √ 3 + 4z ≥ 6. Bài 2.127 : Chứng minh rằng với mọi x, y > 0 ta có : (1 + x) 1 + y x 1 + 9 √ y 2 ≥ 256. Đẳng thức xảy ra khi nào. Bài 2.128 : Cho a, b, c là ba số dương thoả mãn a + b + c = 3 4 . Chứng minh rằng : 3√ a + 3b + 3√ b + 3c + 3√ c + 3a ≤ 3. Khi nào đẳng thức xảy ra? Bài 2.129 : Chứng minh rằng 0 ≤ y ≤ x ≤ 1 thì x √ y − y √ x ≤ 1 4 . Đẳng thức xảy ra khi nào ? Bài 2.130 : Cho x, y, z là ba số dương và xyz = 1. Chứng minh rằng : x2 1 + y + y2 1 + z + z2 1 + x ≥ 3 2 . Bài 2.131 : Cho x, y là các số thực thoả mãn điều kiện x2 + xy + y2 ≤ 3. Chứng minh rằng : −4 √ 3 − 3 ≤ x2 − xy − 3y2 ≤ 4 √ 3 − 3. Bài 2.132 : Cho các số thực x, y, z thoả mãn điều kiện 3−x + 3−y + 3−z = 1. Chứng minh rằng : 9x 3x + 3y+z + 9y 3y + 3z+x + 9z 3z + 3x+y ≥ 3x + 3y + 3z 4 . Bài 2.133 : Cho hai số dương x, y thay đổi và thoả mãn điều kiện x + y ≥ 4. Tìm giá trị nhỏ nhất của biểu thức A = 3x2 + 4 4x + 2 + y3 y2 . Bài 2.134 : Tìm giá trị nhỏ nhất của hàm số : y = x + 11 2x + Ö 4 1 + 7 x2 , x > 0. Bài 2.135 : Cho x, y, z là các số thực dương. Tìm giá trị nhỏ nhất của biểu thức : P = 3 4(x3 + y3) + 3 4(y3 + z3) + 3 4(z3 + x3) + 2 x y2 + y z2 + z x2 . Bài 2.136 : Cho a, b là các số dương thoả mãn ab + a + b = 3. Chứng minh rằng : 3a b + 1 + 3b a + 1 + ab a + b ≤ a2 + b2 + 3 2 . Bài 2.137 : Cho x, y > 0 và xy = 100. Hãy xác định giá trị nhỏ nhất của biểu thức P = x2 + y2 x − y . TRẦN ANH TUẤN - 0974 396 391 - (04) 66 515 343 Trang 46 www.VNMATH.com www.VNMATH.com